Download - Ευκλειδης Β 65

Transcript
Page 1: Ευκλειδης Β 65

[:1 ο

ο

ο Μαθηματική Εταιρεία

ο ο

Page 2: Ευκλειδης Β 65

-Άλγεβρα

Α Ενιαίου Λυκείου Θ. Τ σιούμας, Ι. Σιάχος

Μεθοδολογία Άλγεβρας Β' Ενιαίου Λυκείου

Γενικής Παιδείας

Ε. Πρωτοπαπάς

μαθηματικά t· t .. ιαlotJ l'wκclotJ

6')(0('//�Ο ιrος ?,OO=j--?,OOf

�ΕΚΔΟΣΕΙΣ ·�����.�

Μεθοδολογία Άλγεβρας Α Ενιαίου Λυκείου

Μ. Ευσταθίου, Ε. Πρωτοπαπάς

Γεωμετρία Β' Ενιαίου Λυκείου

Γ ενικής Παιδείας

Ε. Πρωτοπαπάς

Μαθηματικά Γ' Ενιαίου Λυκείου

Γενικής Παιδείας

Μ. Τσιλπιρίδης

Αρχέc; οικονομικής θεωρίας Γ Ενιαίου Λυκείου

Μάθημα επιλογής

για όλες τις κατευθύνσεις Π. Μηλίτσης

ΣΕ ΟΛΑ ΤΑ ΒΙΒΛΙΟΠΩΛΕΙΑ Αθήνα: 210.38.1 1.850, 21 0.38.31.078,

θεσσαλονίκη: 2310.70.63.54, 2310.70.67.15

web sίω: http://www.patakis.gr e-mall: 1.. . � [email protected]

rιιr Ελλqνιιιιjr

lιi•Βιιμ•rιιιιjr Εr•ι peί •r

,,;,., ,, .• ,. ,,_,;,,., ..

Πι..ριaδικι') Εnιστημονιοι.ή Εκδοαη

m>ΟΣ

·-�-ι ·--��--��----- -- - - --- Ελληνική Μαβι1ματικι') Εταιρε.ία

Κεντ:pηοj 6ni8ει;q Eλλqll'.;�ιj Ι!αlί8ηιμStΙτι;,:r;j E"f''rt'i.,;r;ι.:;:/q

Π•νεΉιοτqμfοu 3• - ���νq

Page 3: Ευκλειδης Β 65

ΕΛΛΗΝΙΚΗ ΜΑΘΗΜΑΤΙΚΗ ETAIPEIA Τεύχος 65 • ιούΑιοc; • Αύyουστοc; • Σεπτέμβριος 2007· Έτος Αη' • Ευρώ: 3,50

e-maiι: [email protected] www.hms.gr

ΜΑθΗΜΑΤΙΚΟ ΠΕΡιΟΔΙΚΟ ΓΙΑ ΤΟ ΛΥΚΕΙΟ

ΠΕΡΙΕΧΟΜΕΝΑ ./ Ο Πρόεδρος της Δημοκρατίας

και η Ολυμπιακή Ομάδα 2

./ Ιστορικές Μαθηματικές Αναφορές

Ο π υπάρχει Παντού 4

./ Μαθηματικές Ολυμπιάδες 2007 8

./ Homo Mathematicus 24

Μαθηματικέι Α� Τέι:ξης; ./ Άλγεβρα Ταυτότητες • Ισότητες και Ανισότητες 28

./ fεωμετρία 36

./ Απόδειξη 40

Μα:θηματικά Β' Τόξης, ./ Άλγεβρα 43

./ fεωμετρία 46

./ Κατεύθυνση 50

Μαθηματι.κά Γ' Τάξ,ης; ./ Μαθηματικά Κατεύθυνσης 53

./ Μαθηματικά fενικής Παιδεία 58

./ Πανελλαδικές εξετάσεις 65

./ Το βήμα του Ευκλείδη 66

./ Ο Ευκλείδης προτείνει ... 74

./ Τα Μαθηματικά μας Διασκεδάζουν 77

./ Μουσική και Μαθηματικά 79

Γ ράμμα της Σύνταξης Αγαπητοί συνάδελφοι

Σας euχόμαιιτe ολόψuχα καλή χρο­

νιά με uγeία και δημιουργικότητα για ιιας

και τοuς μαθητές ιιας. Πρόθεσn μας είναι

να παρακολουθούμε με ενδιαφέρον τις μαθη­

ματικές εργασίες που μας στέλνετε. Για τη

μαθηματικn ομοιομορφία των άρθρων θα μας

επιτρέψετε σε συνεννόηση μαζί σας να γίνο­

νται κάποιες επεμβάσεις στον τρόπο παρου­

σίασης. Επίσης για λόγους αντικειμενικών

δυσκολιών (περιορισμένος χώρος κλπ.! πιθα­

νόν να παραλειφθούν κάποια τμnματα της ερ­

γασίας χωρίς αυτό να αποβαίνει σε βάρος της

ουσίας. Ας μην ξεχνάμε ότι την κύρια ευθύνη

για την επιστημονικn ορθότητα κάθε επιστημο­

νικού άρθρου φέρει ο ίδιος ο aρθρογράφος.

Ενώνοuμe λοιπόν τις δuνάμeις μας

για την enίτeuξη τοu καλύτepοu δuνατού.

Με συναδελφικούς χαιρετισμούς

Ο Πρόεδρος της Συντακτικnς Επιτροπnς

Γ.Σ. Τασσόπουλος

••••••••••••••••••••••••••••••••••••••••••••••••••••••••••••••••••••••••••••••

ΕΚΔΟΣΗ ΤΗΣ ΜΑΘΗΜΑτΙΚΗΣ ΗΑΙΡΕΙΑΣ

ΠΑΝΕΠΙΠΗΜΙΟΥ 34 - 1 06 79 ΑΘΗΝΑ Τηλ.: 21 Ο 3617784 - 3616532 Fax: 21 03641 025 Εκδότης: Αλεξανδρής Νικόλαος Διευθυντής: Τυρλής Ιωάννης Κωδικός ΕΛ. ΤΑ.: 2055 ISSN: 1105- 7998

Επιμέλεια 'Εκδοσης:

Εκτε λεστική Γραμματεία

Πρόεδρος: Τασσόποuλος Γιώργος

Αντιπρόεδροι: Εuαταθίοu Βαγγέλης

Γραμματέας: Χριστόπουλος Παναγιώτης

Μέλη: Αργυράκης Δ.

Δρούτσας π. Λουρίδας Σ.

Ταπεινός Ν.

Συνταντική επιτροπή Αθανασόπουλο$ Γεώργιος Λουρίδας Σ�ρης Αναστασίου Για�ς Μαλϊχgιέκας Θαvάσης ΑνδρουλακάκηςΝικος Μανολ".άκου Σταματική Αντωνόπουλος Νίκος Μαυρογιαννάκης Λεωνίδας Αργυρgκης Δηιιήτpιος Μενδρινός_Γιάννης Βακαλόπουλος Κωστας Μεταξά_ς Νικόλαος Γράψας Κων/νος Μ�λωνας Δ1Jμήτρης Δρουτσας Παναγι,ώτης Μωκος Χρηστος Ευσταθίου Βαγγ€λης Πανουσάκης Νίκος Ζαχαρόπουλος Κων/νος Ρέyκλης _Δημήτρης Ζώiος Βαγγέλfις Σαιτη Ευα Καλίκας Σταμάtης . Στα"ίκος Κώστας ΚανέλλΟς Χρήστος Στά"ίκος Παναγιώτης Καραγκούνης Δημήτρης Στρατής Γιάννης Καρακατσάνης ΒασίΛης Ταπεινός Νικόλαος Καρκάτης Βασίλης Τασσόπουλος Γιώργος Κατσούλης Γιώργος Ί'ζιώτζιος Θανάσης Κερασα�ίδης Γιάννης Τριάντος Γεώργιος Καρδαμιτσης Σπύρος Τσαγκάρης Ανδρέας Κnπουρός Χρήστος Τσατούρα_ς Ευάγγελος Κλάδη Κατερίνα Τσικαλουδάκη� Γιώργος Κόντζιας Νίκος ΤσιουΜ' Θανασης

Τασσόπουλος Γιώργος Κοτσιφqκης Γιώργος, Τυρλ ς ,ωάννης Κυριακοπουλος Αντωνης Φανέ η Άννυ Σύνθεση εξωφύλλου:

Ευαταθίου Βαγγέλης Κυ�ιακ�πουλος Θανάσης Χαραλάμποπούλοu Λίνα Κυ ερ"!Ιτου Χ�στ. Χαραλά�υς Θάνος Λα αρίδης Χρηστος Χριστόπουλος Παναγιώτης

••••••••••••••••••••••••••••••••••••••••••••• Λ.�� ����Qlli ••••••••••••••••••• ••••

από τα τετράγωνα του Eϋler ( 1 7 7 9)

• Τα διαφημιζόμενα βιβλία δε σημαίνει ότι πρστείνονtαι ωτό την Ε.Μ.Ε. • Οι συνεργάτες, τα άρθρα, οι προτεινόμενες ασκήσεις, οι λύσεις ασκήσεων κτλ. πρέπει να σtέλνονtαι έγκαιρα, σtα γραφεία της

Ε.Μ.Ε. με την ένδειξη "Για τον Ευκλείδη β"'. Τα χειρόγραφα δεν επισι:ρέφονtαι. Τιμή Τεύχους ευρώ 3,50

Ετήοια συνδρομή (12,00 + 2,00 Ταχυδρομικά = ευρώ 14,00)

Ετήοια συνδρομή για Σχολεία ευρώ 12,00

Το αντίτιμο για τα τεύχη που παρα'Γ'(έλνονται σtέλνεται με ωτλή επιταγή σε διαταγή Ε.Μ.Ε. Τ αχ. Γραφείο Αθήνα 54 Τ.Θ. 30044 ή πληρώνεται σtα γραφεία της Ε.Μ.Ε.

Εκτύπωση: ιΝΓΕΡΠΡΕΣ Α.Ε. τηλ.: 210 8160330 YneιJBuνo� τunoγpαφefou: Β. Σωτηριάδης

Page 4: Ευκλειδης Β 65

Ο Πρόεδρος της Δημοκρατίας δέχεται τους μαθητές των Εθνικών Ολυμπιακών ομάδων στα Μαθηματικά

Ο πρόεδρος της Δημοκρατίας Κάρολος Παπούλιας δέχθηκε την Τετάρτη 4 Ιουλίου 2007

στο Προεδρικό Μέγαρο τους μαθητές μέλη των εθνικών ομάδων που συμμετείχαν στις Βαλ­

κανικές Μαθηματικές Ολυμπιάδες και την ομάδα που θα συμμετάσχει στη 48η Διεθνή Μαθη­

ματική Ολυμπιάδα (ΙΜΟ) στο Βιετνάμ.

Χαρακτηριστικά στιγμιότυπα από την υποδοχή της Ελληνικής Ομά­δας στο Προεδρικό Μέγαρο

Οι μαθητές που απο τελούν τις ομάδες είναι: Βαλκανική Μαθηματική Ολυμπιάδα Νέων Διεθνής Μαθηματική Ολυμπιάδα

Λ ιάμ πης Κ ωνσ ταντίνος Βαφείδης Αναστάσιος Λο γοθέτης Φώ της Μαυροειδή Άλκηστις Μπραζίτικος Κ ωνσ ταντίνος Μ πρ αζίτικος Σ ιλ ου ανός Ντρ ιάνκος Ι ωάνν ης Παπαδημ ητρ ίου Δημ ήτρ ης Παγωνάκης Δημ ήτριος Ρ ωμ ανίδης Νικόλ αο ς Π αντέλ α Σ τέλλ α Σ άββ ας Μ ιχ αήλ

ΕΥΚΛΕΙΔΗΣ Α' 65 τ.l/2

Page 5: Ευκλειδης Β 65

Χαρακτηριστικά στιγμι()τυπα Μαζ ί τους ήταν ο Πρόεδρος της Ελληνικής Μαθη­μ ατικής Εταιρείας, Καθηγητής κ. Νικόλαος Αλεξαν­δρή, ο Γενικός Γραμμ ατέας κ. Ιωάννης Τυρλής, ο Α­ναπληρωτής Καθηγητής κ. Ανάργυρος Φελλούρης Αρχηγός της Ελληνικής ομ άδας και ο Επίκουρος Κα­θηγητής κ. Γεώργιος Δημάκος, Υπαρχηγός της Ελλη­νικής ομ άδας.

Ο κ Παπούλιας δήλωσε την ιδιαίτερη χαρά του για τους Έλληνες μ αθητές που διαπρέπουν και είπε χαρα­κτηριστικά «μ ας κάνετε υπερήφανους» . Μάλιστα, a­στειευόμ ενος είπε ότι αισθάνεται δέος όταν βλέπει μα­θητές να διαπρέπουν σε μια τόσο δύσκολη επιστήμη, καθώς ο ίδιος δεν υπήρξε δυνατό ς στα μ αθημ ατικά.

Ο Πρόεδρος της Δημοκρατίας ευχήθηκε στα παι­διά καλή δύναμ η και καλή επιτυχία.

Από την πλευρά του ο Πρόεδρ ος της Ελληνικής Μαθημ ατικής Εταιρείας καθηγητής Νικόλαος Αλε­ξανδρής, πρόσφερε στον πρόεδρο της Δημ οκρατίας τις εκδόσεις της Εταιρείας, ευχαριστώντας τον θερμ ά, για­τί , όπως είπε, είναι ο πρώτος αρχηγός Ελληνικού κρά­τους που προσκαλεί την Ελληνική Μαθημ ατική Εται­ρεία και χάρη σε αυτόν για πρώτη φορά τα μ έλη της Εταιρείας και οι νεαροί μ αθητές περνούν την πόρτα του προεδρικού Μεγάρου.

«Από την αρχαιότητα έως τη σύγχρονη εποχή του διαδυκτύου και της τεχνολογικής προόδου, τα μ αθημα­τικά αποτελούν γέφυρα που ενώνει εποχές, έθνη , πολι­τισμ ούς και λαούς, έχοντας συμβάλλει καθοριστικά στην πρόοδο της aνθρωπότητας. Δεν είναι άλλωστε τυ­χαίο ότι τα μ αθηματικά είναι στη ουσία η πρώτη κοινή, παγκόσμια γλώσσα των α νθρώπων.

Τ α μ υστικά αυτού του συναρπαστικού κόσμ ου των Μαθηματικών έχετε τη χαρά και την τιμή να «ανακα­λύπτετε» .

Διαγωνισθήκατε σε ένα από τους ευγενέστερους στίβους: αυτόν του ανθρώπινου πνεύμ ατος. Και κατα­φέρατε να διακριθείτε γεμίζ οντας μ ας περηφάνια, αλλά πάνω από όλα μ ε αισιοδοξία για το μ έλλον του τόπου.

Τ α θερμ ά μου συγχαρητήρια.» ΚΑΡΟΛΟΣ ΠΑΠ ΟΥ ΛΙΑΣ

ΕΥΚΛΕΙΔΗΣ Β' 65 τ.l/3

Page 6: Ευκλειδης Β 65

ΚΥΚΛ ΟΙ� ΣΦΑΙΡΕΣ .. ΤΥΧΑΙΑ ΓΕΓΟΝΟΤΑ. Ο π ΥΠΆΡΧΕΙ ΠΑΝΤΟΥ.

Γιώργος Κ. Πούλος

Ο τετραγωνισμός του κύκλου είναι ένα από τα γνωστά άλυτα προβλήματα της αρχαιότητας και έχει την εξής διατύπωση: Να κατασκευαστεί τετράγωνο με εμβαδόν ίσο με το εμβαδόν ενός κύκλου με κανόνα και διαβήτη όχι προσεγγιστικά αλλά ακριβώς. Ο τετραγωνισμός του κύκλου είναι άμεσα συνδεδεμένος με τον γνωστό αριθμό π. Ο αριθμός π συναντάται σε όλους τούς πρακτικούς υπολογισμούς του κύκλου. Για να υπολογιστεί το μήκος της περιφέρειας πολλαπλασιάζεις την διάμετρο με τον αριθμό π. Το εμβαδόν του κύκλου προκύπτει με πολλαπλασιασμό τού τετραγώνου της ακτίνας με το π. Η ερώτηση για τον τετραγωνισμό του κύκλου μπορεί να διατυπωθεί και με τον εξής τρόπο: Να κατασκευασθεί ένα ευθύγραμμο τμήμα με κανόνα και διαβήτη μήκους ίσου με π. Βέβαια ο ακριβής προσδιορισμός του π είναι συνδεδεμένος με την ιστορία των Μαθηματικών. Ήδη ο αριθμός π ήταν γνωστός στους Βαβυl.ωνίους και τους Αρχαίους Αιγύπτιους γύρω στο 2. 000 πΧ

Ο Αρχιμήδης όμως είναι ο πρώτος ο οποίος δεν προσπάθησε να υπολογίσει ακριβώς το π αΙJ.ά έδωσε ' ' ' ' ' ' 310 31 ενα ανω και ενα κατω φραγμα της τιμης του - < π. < - .

71 7 Το παγκόσμιο ρεκόρ στον υπολογισμό του π. έχει ο γ ιαπωνέζος Υ asumasa Kanada από το

πανεπιστήμιο του Τόκιο το 1997 υπολόγισε 515 δισεκατομμύρια ψηφία του π σε περίπου 29 ώρες. Το μυστικό του π. βρίσκεται στο γεγονός ότι δεν μπορεί να προβλεφθεί η ακολουθία των ψηφίων του. Δεν υπάρχει κανένα πρότυπο γι' αυτό. Στην αρχή ο π. θεωρήθηκε από αυτήν την άποψη μη πεπερασμένος και παρά τα 515 δισεκατομμύρια δεκαδικά ψηφία δεν υπάρχει κανένα τέλος γι' αυτά. Έτσι η προσπάθεια για περισσότερα ψηφία καταλήγει να γίνεται ένα σισύφειο έργο.

Το π είναι άρρητος: Αυτό σημαίνει ότι τα άπειρα δεκαδικά ψηφία δεν επαναl.αμβάνονται όχι επειδή δεν έχουμε μέθοδο γι αυτό αλλά γιατί ακριβώς δεν υπάρχει τέτοια. Αυτό έχει αποδειχθεί το 1761 από Johann Heίnrίch Lambert.

Το π είναι υπερβατικός: Αυτό σημαίνει ότι ο π δεν μπορεί να εκφραστεί ακριβcίχ; με τη βοήθεια μιας μιγαδικής εξίσωσης. Δεν υπάρχει πολυώνυμο με ρητούς συντελεστές το οποίο να έχει ;•ια ρίζα του τον αριθμό π: Εν' γένει είναι πολύ δύσκολο να αποδειχθεί ότι ένα συγκεκριμένο στοιχείο είναι υπερβατικό. Ωστόσο αποτελεί ένα σαφές παράδειγμα ή απόδειξη της υπερβατικότητας του π. Αυτό αν και φαίνεται να είναι ένα τεχνικό αποτέλεσμα έχει ως συνέπεια του το ότι ο τετραγωνισμός του κύκl.ου είναι αδύνατος με κανόνα και διαβήτη. Το συμπέρασμα αυτό μας επιστρέφει στο 1882 και στον Ferdίnand Lίndemann ο όποιος στηριζόμενος στην απόδειξη της υπερβατικότητας του e από τον Σαρλ Ερμίτ (Sarl Hermίt) απέδειξε ότι ο π είναι υπερβατικός αριθμός (για την απόδειξη αυτή βλέπε « Το θεμελιώδες πρόβλημα της Άλγεβρας » Benjamίn Fίne - Gerhard Rosenberger Εκδόσεις Leader books Αθήνα 2001 )

Το π. μας δίνει άλυτα προβλήματα μέχρι σήμερα. Πολλοί πιστεύουν ότι τα ψηφία του π. είναι τυχαία κατανεμημένα. Τα ψηφία δεν είναι φυσικά τυχαία, αλλά κάθε ένα είναι κατά μοναδικό τρόπο ορισμένο. Αν στην ανάπτυξη των ψηφίων μετά την υποδιαστολή δεν εμφανίζεται το μηδέν, δεν θα ήταν τυχαία η ακολουθία των ψηφίων μετά το κόμμα. Όμως το πρώτο μηδέν εμφανίζεται για πρώτη φορά στην 3 1η θέση. Επίσης θα μπορούσε να εμφανιστεί το διψήφιο τμήμα 00, ή το 007, δηλ. εν ολίγοις: θα μπορούσε να

ΕΥΚΛΕΙΔΗΣ Β ' 65 τ.l/4

Page 7: Ευκλειδης Β 65

--------------- Ιστορικές Μαθηματικές Αναφορές -------------­

εμφανιστεί κάθε πεπερασμένη ακολουθία ψηφίων. Ο Αύγουστος Ντε Μόργκαν(Αugust D Morgan) το 19° αιώνα για παράδειγμα έκανε μια πρόχειρη μελέτη για τα πρώτα 600 ψηφία και επισήμανε ότι υπάρχουν λιγότερα επτά από όσα περίμενε κανείς. θα περίμενε γύρω στα 60 αλλά είναι δέκα λιγότερα Οι Μαθηματικοί ονομάζουν αυτήν την ιδιότητα (" normal") κανονική. Η ερώτηση όμως είναι εάν ο π. είναι "κανονικός" ααριθμός, μία υπόθεση αναπόδεικτη μέχρι σήμερα. Η έννοια κανονικός δεν είναι ίδια με την έννοια άρρητος. Ένας αριθμός είναι κανονικός, όταν κάθε ψηφίο ή συνδυασμός ψηφίων έχει την ίδια πιθανότητα όπως και κάθε άλλο ψηφίο ή συνδυασμός ψηφίων. Ο χαουαρντ Ιβς στο mathematίcal cίrcles Reνίsίted 1971- γράφει «Το ζήτημα της κανονικότητας ή μη του π δεν θα λυθεί βέβαια ποτέ από ηλεκτρονικούς υπολογιστές. Πρόκειται για περίπτωση θεωρητικού προβλήματος που απαιτεί ιδιαίτερα χαρισματικούς μαθηματικούς και δεν μπορεί να λυθεί μόνο με υπολογισμούς»

Στην πορεία της ιστορίας αναπτύχθηκαν πολλές μέθοδοι για τον υπολογισμό του π. Ένας τύπος ο οποίος προσεγγίζει καλύτερα από άλλους την τιμή του π είναι ο μαθηματικός τύπος

π __ 9,801 CΣοο (4n)!(1103+ 26300n)

)_1

J8 n=O (n!)43964n τον οποίο διατύπωσε η μεγάλη μαθηματική μορφή ο ινδός

schrίnίνosa Ramannjan το 19 14.

Ορθογώνια σε κύκλους Μια δυνατότητα για να υπολογίσουμε το εμβαδόν ενός κύκλου

και με αυτό την τιμή του π, στηρίζεται στο να εγγράψουμε εσωτερικά του κύκλου, ορθογώνια παραλληλόγραμμα. Η μέθοδος αυτή διατυπώθηκε τον 17 αιώνα στην Ιαπωνία.

Παίρνουμε έναν κύκλο ακτίνας όσης με την μονάδα Τότε το εμβαδόν του ισούται με π. Θεωρούμε το τέταρτο του κύκλου το οποίο θα έχει εμβαδόν έσο με π/4. Διαιρούμε την ακτίνα σε ίσα τμήματα π.χ. σε 20 (είκοσι) και σε κάθε τμήμα ορίζουμε τα ορθογώνια παραλληλόγραμμα τα οποία προσεγγίζουν εσωτερικά τον κύκλο. Το εμβαδόν των παραλληλογράμμων υπολογίζεται απλά.

-

r-r-- ---r-_ ι--- r-

'1'-.r--r'-,,.-'\

� 1\ I

Το τμήμα οριζόντια είναι γνωστό από την αρχική επιλογή και το κάθετο υπολογίζεται από τον τύπο χ2+/=1 προσθέτοντας το εμβαδόν των ορθογωνίων υπολογίζεται μία προσέγγιση του π

Ο ορισμός του μαγικού αριθμού Περίμετρος προς διάμετρο Εάν μετρήσουμε την περιφέρεια ενός κύκλου με την

βοήθεια της διαμέτρου προκύπτει ότι η περιφέρεια είναι πάντα μεγαλύτερη κατά έναν σταθερό αριθμό. Εάν πάρουμε την διάμετρο ώση με 1 μονάδα τότε η περίμετρος είναι ίση με π.Η πρώτη προσέγγιση της τιμής του π έγινε από τον Έλληνα Μαθηματικό και Φυσικό Αρχιμήδη ( 2 8 7- 212 π xJ Ο Αρχιμήδης ήταν ο πρώτος ο οποίος διατύπωσε έναν συστηματικό μαθηματικό τρόπο για την προσέγγιση της τιμής του π. Η μέθοδος του αποτελείται από τα εξής:

3.14m Σχ. 2

Σ' έναν κύκλο εγγράφουμε και περιγράφουμε κανονικά πολύγωνα και στην συνέχεια παίρνουμε την

' 3 10 3 1 /RΊ ' λ ' ' 'δ β'1 ' ' άλ ανισοτητα - <π.< - ψΑεπε ο υπο ογισμος του π υπο του Αρχιμη ους στο Βι Αιο Τα περιφημα υτα 71 7

προβλήματα της αρχαιότητας Μπρίκας Μ Αθήνα 1970 καθώς επίσης και στο Απειροστικός Λογισμός τόμος Ι Νεγρεπόντης - Γιωτόπουλος - Γιανyακούλιας εκδόσεις Αίθρα Αθήνα 1987)

Ο όγκος μιας σφαίρας- Το π μας πληροφορεί Αυτή η εικόνα πρέπει να κοσμεί τον τάφο του Αρχιμήδη. Ο Αρχιμήδης

μπορεί να μείνει γνωστός μόνο από τον υπολογισμό του όγκου της σφαίρας. Ο όγκος της σφαίρας είναι ακριβώς το 113 που όγκου του περιγεγραμμένου κυλίνδρου. Αυτό το απέδειξε ο Αρχιμήδης με την βοήθεια ενός τρίτου στερεού, ενός διπλού κώνου τον οποίο εγγράφει στον κύλινδρο. Ο Αρχιμήδης οδηγήθηκε στο συμπέρασμα ότι ο όγκος της σφαίρας με ακτίνα r ισούται με 4πl 113 (βλέπε Απαντά Αρχιμήδους τόμος Α Μέρος Β Εκδόσεις τεχν. Επιμελ. Ελλάδος Αθήναι 1970 ) Σ χ. 4

ΕΥΚΛΕΙΔΗΣ Β ' 65 τ.l/5

Page 8: Ευκλειδης Β 65

--------------- Ιστορικές Μαθηματικές Αναφορές --------------

Το π βοηθά την ταξινόμηση κύκλων Στο ερώτημα ποιό ποσοστό της επιφάνειας ενός επίπεδου τραπεζιού μπορούμε να καλύψουμε με ίσα

κέρματα χωρίς το ένα κέρμα να πατάει το άλλο, η απάντηση είναι σε διάταξη εξαγώνου έχουμε την μεγαλύτερη κάλυψη. Με τα κέρματα θα καλύψουμε το 90% της επιφάνειας.

Ακριβέστερα το συνολικό εμβαδόν καλύπτεται με κύκλους του ιδίου μεγέθους Το μέρος του συνολικού

π εμβαδού που θα καλυφθεi με αυτόν το τρόπο της διάταξης θα είναι ακριβώς ίση με r;; Το ποσοστό αυτό

είναι πολύ καλύτερο από το να τοποθετηθούν τα κέρματα σε άλλη διάταξη π.χ. σε «τετραγωνική».

Επίσης αν τα κέρματα εφάπτονται μεταξύ των παίρνουμε ένα ποσοστό κάλυψης περίπου 78%. Δεν είναι εύκολο να απεδείχθη ότι ο εξαγωνικός τρόπος της διάταξης είναι ο καλύτερος δυνατός αυτό έγινε κατορθωτό στις αρχές του μισού του 20 αιώνα.

2ν3

Σχ. 5

Διάταξη σφαιριbν. Πως τοποθι:τοι5vται τα μιίλα; Ό πως στην αγορά πιστει5ει ο Gαιι .... , ..

Το ερώτημα για την διάταξη των σφαιρών είναι πολύ δυσκολότερο από αυτό των κύκλων στο επίπεδο. Το πρόβλημα μας οδηγεί πίσω στον J. Kepler (1571-1630) ο οποίος σε μια πραγματεία για τις νιφάδες του χιονιού αναφέρθηκε στο πόσο πυκνά μπορούν να τοποθετηθούν τα κουκούτσια των φρούτων π.χ. των μήλων. Ο ίδιος όντας Μαθηματικός εμπνεύστηκε μεν από το συγκεκριμένο ερώτημα αλλά το έθεσε στην γενικότερη μορφή δηλ. ποια είναι η πυκνότερη διάταξη των σφαιρών. Γνωστό ως Εικασία Kepler Είχαν περάσει αιώνες χωρίς να αποδειχθεί το πρόβλημα αυτό. Στο διεθνές συνέδριο των Μαθηματικών του 1900 πού έγινε στο Παρίσι ο Γερμανός μαθηματικός Daνίd Hίlbert παρουσίασε στην μαθηματική κοινότητα μια λίστα άλυτων προβλημάτων με την εικασία του Kep/er στην 18η θέση.

Η Πυκνότητα της διάταξης είναι ώση με � �Ο, 74 ν18

Σχ.6 Στην περίπτωση αυτή πάλι η λύση είναι η «εξαγωνική διάταξη» Αρχίζουμε με μια σειρά σφαιρών που

τοποθετούνται σε εξαγωνικό σχήμα και ο τρόπος αυτός είναι ο καλύτερος. Στην δεύτερη σειρά τοποθετούνται οι σφαίρες στα κενά της πρώτης και συνεχίζουμε με τον ίδιο τρόπο για την Τρίτη κ.ο.κ.θ .. Μπορούμε να υπολογίσουμε την πυκνότητα αυτής της διάταξης με την βοήθεια του π Ο τύπος της είναι

� . Έτσι μπορούμε να καλύψουμε το 7 4% του χώρου. ν18

Δεν βρέθηκε καλύτερη λύση για την διάταξη και για μεγάλο διάστημα δεν μπορούσε να αποδειχθεί ότι υπάρχει πυκνότερη διάταξη. Ένα πρώτο ρήγμα δημιούργησε ο Μαθηματικός Car/ Gauss ο οποίος το 1831 απέδειξε αυτήν την υπόθεση με την δεσμευτική προϋπόθεση ότι οι σφαίρες της διάταξης δημιουργούν ένα κανονικό σχήμα.

Το πρόβλημα γενικά, παρ' όλα αυτά έμενε άλυτο και συνέχιζε να γοητεύει για την απλή διατύπωση από

ΕΥΚΛΕΙΔΗΣ Β' 65 τ.l/6

Page 9: Ευκλειδης Β 65

--------------- Ιστορικές Μαθηματικές Αναφορές --------------­

την μία, αλλά και για την δυσκολία μιας απόδειξής του από την άλλη. Κάτι τέτοιο εμφανίζεται συχνά στα Μαθηματικά αρκεί να αναφερθούμε στην διάσημη πρόταση του Fermat. (xv +yv =zv) Έτσι οι μανάβηδες στις λαίκές αγορές τοποθετούν τα φρούτα μήλα ή πορτοκάλια σε εξάγωνα λες και έχουν διαβάσει τον Ganss.

Μόλις το 1998 ο Thomas Hales κατάφερε να αποδείξει ότι αυτή η μέθοδος συσκευασίας είναι η πυκνότερη. (Βλέπε Thomas Hales )

Ο αριθμός π, εκτός του ότι μπορεί να χρησιμοποιηθεί για τον υπολογισμό του μήκους της περιφέρειας ενός κύκλου, μπορεί επίσης να χρησιμοποιηθεί και για τον υπολογισμό του εμβαδόν του. Ένας κύκλος με ακτίνα r = 1m έχει εμβαδά ίσο με π τετραγωνικά μέτρα. Ε=π /. Αυτό σημαίνει ότι το εμβαδόν του κύκλου προκύπτει από ένα τετράγωνο r πλευράς, του οποίου το εμβαδά στην συνέχεια πολλαπλασιάζεται με το π. Ο πρώτος ο οποίος εξέτασε τη σχέση κύκλου και τετραγώνου ήταν ο Αναξαγόρας ο Κλαζομενεύς (500 -428 π.Χ ) δάσκαλος και φίλος του Περικλή. Λίγο καιρό αργότερα οι σοφιστές Αντιφών και Βρύσων από την Ηράκλεια και οι δύο σύγχρονοι του Σωκράτη(469-399 π. χ.) επιχείρησαν να μετρήσουν το

3,14159m2

Σχ. 7

εμβαδόν ενός κύκλου χρησιμοποιώντας την ιδέα της « εξάντλησης ».Αν πάρουμε ένα κανονικό εξάγωνο εγγεγραμμένο σε κύκλο και διπλασιάσουμε τις πλευρές του και στην συνέχεια τις διπλασιάσουμε ξανά και συνεχίσουμε την μέθοδο του διπλασιασμού θα πάρουμε ένα πολύγωνο που θα ισούται προσεγγιστικά με τον κύκλο.

nίa βιβΑίa Μaθ�μaτικώv 1\ΩU dιaφίρΩUV

Μαθηματικά Γιο τn Θετική· Τεχνολογικn Κοτεύθuνσn Β' Λυκείου Π. Δοuλnγέρnς

Δ. Α. Κωοτόnοuλος

Μαθnμcιτικά Γιο τn Θετικn­Τεχνολογικn Κοτεύθuνσn Γ' Λυκείου Ι. Γοpοτζιώτnς Π. Μόοτοκος

Η Άλyeβρα στις Μοθnμοτικές

' Ολυμnιόδες Σ. Ρωμονfδnς

Εισογωγn στους

Αλγόριθμους Ανόnτυξn Εφαρμογών σε Προγραμματιστικό Περιβόλλον Γ' Λuκειοu

4 Α. Γ. Ντόφλος Δ. Α Κωοτόnοuλος �-!!!!!!!!!!!------<� Η. Ε. Χρnοτόκnς

�t �.:�t.r.�.� ΕΚΔΟΙΕΙΙ ΚΕΔΡΟΙ: Γ. ΓΕΝΝΑΔΙΟΥ 3, ΑΘΗΝΑ 106 78 • ΤΗΛ.: 21 Ο 380971 2 • e-mail: [email protected] -www.kedrosedu.com

ΕΥΚΛΕΙΔΗΣ Β ' 65 τ.Ι/7

Page 10: Ευκλειδης Β 65

Η�.ιι.ι�μ., -

Επιτροπή Διαγωνισμών της Ε.Μ.Ε.

24η Βαλκανική Μαθηματική Ολυμπιάδα 27 Απριλίου - 2 Μαϊου 2007

I

Η 24η Βαλκανική Μαθηματική Ολυμπιάδα διοργανώθηκε φέτος με μεγάλη επιτυχία στην Ελλάδα (Ρόδος) από το Παράρτημα Δωδεκανήσου της Ελληνικής Μαθηματικής Εταιρείας.

Συμμετείχαν συνολικά 1 4 χώρες: Αλβανία, Βουλγαρία, Ελλάδα, Κύπρος, Μολδαβία, Μαυροβούνιο, ΠΓΔΜ, Ρουμανία, Σερβία, Τουρκία καθώς και οι φιλοξενούμενες χώρες: Αρζεμπαιζάν, Καζακστάν, Η­νωμένο Βασίλειο, Ιταλία.

Βαφείδης Νικόλαος, χάλκινο μετάλλιο, Σάββας Μιχαήλ, χάλκινο μετάλλιο, Η Ελληνική ομάδα συμμετίχε με τους εξείς μαθlτές:

Ράπανος Νικόλαος, χάλκινο μετάλλιο, Παπαδημητρίου Γεώργιος, εύφημη μνεία, Ρωμανίδης Νικόλαος, χάλκινο μετάλλιο, Μαυροειδή Άλκηστις.

Αρχηγός της ελληνικής ομάδας ήταν η μαθηματικός Αγγελική Βλάχου και υπαρχηγοί οι μαθηματι­κοί Ανδρέας Τριανταφύλλου και Φωτεινή Αρβανίτη.

Πρόβλημα 1. Έστω ABCD ένα κυρτό τετράπλευρο με ΑΒ = BC = CD, AC -φ BD και έστω Ε το σημείο το-

μής των διαγωνίων του. Να αποδείξετε ότι ΑΕ= DE , αν, και μόνον αν, LBAD + LADC = 120°.

Λύση Έστω LBAC = LBCA = α,LCBD = LCDB =β και S το σημείο το-

μής των ευθειών ΑΒ και DC. Αν AE=DE, τότε ΑΕ ΑΒ - CD DE

sin(2α +β) sin( α+ β) sin( α+ β) sin( α+ 2β) Τώρα είναι φανερό ότι: sin(2a+ β) =sin(a+2β), σ <2α+β<180"' 0° < α+2β < 180°'

α"Φβ (αφού ΑC"ΦΒD),οπότε 2α+β+ α+2β=180° ή α+ β=60°.

Άρα είναι φανερό ότι LBAD + LADC = 120°.

Α

Α ντίστροφα, αν υποθέσουμε ότι LBAD + LADC = 120°, τότε έχουμε

LAEB = LBAE + LEDC, LAEB = LEAD + LEDA ,

ΕΥΚΛΕΙΔΗΣ Β' 65 τ.l/8

s

Β

β α· ,,c α+β . α+β

( 1 ) (2)

Ε

D

Page 11: Ευκλειδης Β 65

--------- Μαθηματικοί Διαγωνισμοί - Μαθηματικές Ολυμπιάδες --------­

από τις (1) και (2) με πρόσθεση κατά μέλη προκύπτει ότι: 2(LAEB) = LBAD + LADC = 1 20° . Επίσης από το τρίγωνο ADS και την υπόθεση , προκύπτει ότι LBSC = 60° .

Επομένως έχουμε ότι LAEB = 60° = LBSC

και ότι το τετράπλευρο SBEC είναι εγγράψιμο . Άρα είναι LBSE = LBCE = LBAE και AE=ES.

Ομοίως αποδεικνύεται και ότι ES=ED.

Β

Στο τρίγωνο ABD έχουμε : Α, +κ+<Ρ+Dι =180". Στο τρίγωνο ACD έχουμε : λ +λ+ ω+ Dι = 180°

Στο τετράπλευρο ABCD έχουμε : Αι +λ+κ+2<Ρ+2rο+Dι =360°

� ε πρόσθΓ ε���������-----. 3(Α + D ) + 2(λ+ κ )= 4·180° (1)

Ευθύ

Έστω ΕΑ = ED , τότε τα τρίγωνα ΕΑΒ και ECD έχουν {�: ;� Ει = Ez

Άρα ή θα είναι ίσα ή θα έχουμε κ+λ=1rο'. Αν ήταν ίσα τότε θα είχαμε: ΕΒ = EC οπότε DB = AC (άτοπο) . Άρα κ+ λ= 180°' οπότε απο τη σχέση (1) έχουμε: 3(A+D)+2·180" =4·180" δηλαδή A+D=1200 ο

Έστω ότι Α+ D = 120°. Τότε από την σχέ­ση (1) έχουμε: κ+ λ= 180°. /

I

/ /

' < '

' '

I I

' I I Άρα τα τρίγωνα ΕΑΒ και ECD έχουν: Α Α οπότε κ+λ = 180°} \ \ Ει =Ez

ΕΑ = ΑΒ = 1 � ΕΑ = ED ED DC Ε

ω ' • __ __ 1 ', ' / 1 /

Α -- - - - - "f'-: :: _-_ =-��. Ό

' I

I

Θεωρούμε τους ίσους κύ κλους (B,BC) και (C,BC), Σχήμα 1

που κόβονται στο F . Αν το F ανήκει στην AD τότε προφανώ ς Αι= Dι (Διότι το τρίγωνο BCF είναι ισόπλευρο). Άρα το

τρίγωνο AED είναι ισοσκελές (Σχήμα 1).

Σχήμα 2

Έστω τώ ρα ότι το τρίγωνο AED είναι ισοσκελές (Σχήμα 2). Θα αποδείξουμε ότι F ε AD .

Α ν F � AD , τότε BG = CH οπότε το τετράπλευρο ABCD ισοσκελές τραπέζιο οπότε AC = CD . (Άτοπο)

Αποδείξαμε λοιπόν ΕΑ = ED αν και μόνο αν F εAD.

Αν F ανήκει στην AD τότε (Σχήμα 1) έχουμε: Αι = Dι =30° και Αι +Dι =ΒΕΑ= rο+ψ.

Άρα Α+D= Αι +Dι +m+<P=120°.

ΕΥΚΛΕΙΔΗΣ Β' 65 τ.l /9

Page 12: Ευκλειδης Β 65

--------- Μαθηματικοί Διαγωνισμοί - Μαθηματικές Ολυμπιάδες --------

Έστω ότι το τρίγωνο AED είναι ισο­σκελές.Στρέφουμε το τρίγωνο ECD ώστε ή πλευρά ED να ταυτιστεί με την ΕΑ. Τότε το τετράπλευρο ABEF είναι εγγράψι­μο. (διότι στο τρίγωνο BEF η διχοτόμος της BEF και η μεσοκάθετος της BF κόβ ονται στο Α(* )) (* ) αν η διχοτόμος της BEF και η μεσοκά­θετος της BF ταυτίζονται, τότε το ABCD είναι ισοσκελές τραπέζιο . ( άτοπο )I. Απο το

'

ω

'8ο;:-""'"""'" Α '-,_

-�...,_-.:--------------' χ c

ω •

χ

χ

τρίγωνο BCE έχουμε: ω+ χ =ψ . Απο το τρίγωνο AED έχουμε: I EA.D + EDA =ψ ι. Από το εγγρ άψιμο τετράπλευρο ABEF έχουμε: ω+ χ + 2ψ = 180°

--��--:----�----- ω c

_.

_)( 1

D

Έστω ότι Α + f> = 120° . Θα αποδείξουμε ότι EA = ED .

Θεωρούμε τα ευθύγραμμα τμήματα EF και AF ώστε AEF =ψ και EAF = χ .

Ι σχύουν οι σχέσεις: EAD +ω+ χ + f>l = 120° EAD +D 1 = ω+ χ =Ψ . Άρα ψ =60°

Άρα το τετράπλευρο ABEF είναι εγγράψιμο . Οπότε ΑΒ = AF = CD και τα τρίγωνα AEF και

DEC είναι ίσα.

Να προσδιορίσετε όλες τις συναρτήσεις f: 1R � 1R που είναι τέτοιες ώστε

Ι(!( χ) + y} =Ι(!( χ ) -y } + 4f(x)y, για κάθε x,y ε JR.

Είναι φανερό ότι η συνάρτηση f = Ο ικανοποιεί τη δεδομένη συνθήκη . Υποθέτουμε ότι f * Ο . Επιλέγουμε χ0 Ε JR τέτοιο ώστε f ( χ0 ) * Ο και θέτουμε y = ( )

, για 4f Χο

κάθε y Ε JR. Τότε από τη δεδομένη σχέση, για χ0 αντί του χ και για y αντί του y, λαμβ άνουμε Υ = f ( f ( Χο ) + Υ} -f ( f ( Χο ) -Υ) ·

Για κάθε y1,y2 Ε JR θέτοντας στην (1) Υι-Yz αντί του y λαμβ άνουμε 2

y , ;y, = ι(ι(χ, ) + �)-ι(ι(χ, ) -�} (1)

δηλαδή για κάθε y1, y2 Ε 1R υπάρχουν χ 1 (y 1 ,y 2 ) = f( χ0 ) + Υι - Yz και χ 2 (y 1 ,y2 ) = f( χ0 ) - Υι -Yz έτσι 2 2

ώστε Υι-Υz =f(χ 1 ) -f{χ2 ), δηλαδή 2f(χ1)-Υι =2f(xz)-Yz· (2) 2

Επιπλέον, αντικαθιστώντας το y με το f (χ)-y στη δεδομένη σχέση λαμβ άνουμε f (2f(x ) -Υ ) = f(y ) + 4f(x ) (f(x ) -Υ ) �

ΕΥΚΛΕΙΔΗΣ Β ' 65 τ. l/10

Page 13: Ευκλειδης Β 65

-------- Μαθηματικοί Διαγωνισμοί - Μαθηματικές Ολυμπιάδες --------f(y)-Υ2 = f(2f(x)-y)-(2f(x)-y)2 . (3)

Α ν τώρα αντί των Υ ι, y 2 θέσουμε στην (3) τα χ 1 (Υ ι, y 2 ) , χ 2 (Υ ι, y 2 ) , αντίστοιχα, λαμβάνουμε

f(ΥΙ)-Υ12 = !( 2f(χΙ )-Υ1) -( 2f(χΙ)-Υ1 )2

και f(y2)-Υ;= f( 2f( χ2)-Υ2 ) -( 2f(x2)-Υ2 )2 ,

από τις οποίες και τη (2) λαμβάνουμε f (Υι ) -y� = f (y2 )-y;. Επειδή η σχέση αυτή αληθεύει για τυ­χόντα y1,y2 ε R συμπεραίνουμε ότι f(x )-χ

2 = c ε R, για κάθε χ ε R <=> f(x) = χ 2 + c,c ε R. Εύκολα επαληθεύουμε ότι η παραπάνω συνάρτηση ικανοποιεί τη δεδομένη συνθήκη.

Είναι φανερό ότι η συνάρτηση f =Ο ικανοποιεί τη δεδομένη συνθήκη.

Υποθέτουμε ότι f * Ο . Για y = f (χ) η δεδομένη σχέση γίνεται

f ( 2 f {χ)) = ( 2 f {χ) )2 + f {Ο), για κάθε χ Ε JR. => f {χ) = χ2 + f {Ο), για κάθε χ Ε 2 Im f , όπου Im f είναι το σύνολο τιμών της f.

Αν θεωρήσουμε τη συνάρτηση g(x) = f(x)-x2-f(O), χ ε R ,

τότε θα έχουμε ότι g(x) =Ο, για κάθε χ ε 2 1m/ , g ( f (χ) + Υ ) = g ( f (χ) - Υ ), για κάθε χ, y ε R . ( 1 )

Στη συνέχεια, αν θέσουμε στη σχέση ( 1 ), y = f (χ) - 2 f ( z ) , χ, z ε R , τότε λαμβάνουμε

g ( 2(/{ χ)-f( z )) ) = g( 2f(z )) =Ο, για κάθε x,z Ε JR.. (2)

Από την υπόθεση προκύπτει άμεσα ότι 4 f (χ) y = f ( f {χ) + y) -f ( f {χ) -Υ ) ,

για κάθε χ, y ε R , οπότε , για χ ε R με f (χ) * Ο , καταλήγουμε στην ισότητα συνόλων

lmf -lmf =R. (3)

Επομένως, από τις (2) και (3) προκύπτει ότι {ο} = {g ( 2(f(x)-f(z))) :x,zE1R} = g (21R.) = g (JR.) ,

οπότε θα είναι g (χ) = Ο, για κάθε χ ε R , δηλαδή f (χ) = χ2 + f (Ο), για κάθε χ ε R .

Να προσδιορίσετε όλους τους θετικούς ακέραιους η που είναι τέτοιοι ώστε να υπάρχει μετάθεση

σ του συνόλου {1 ,2, . . . ,n} για την οποία ο αριθμός �σ(l) + �σ(2) + �···+�σ( η) είναι ρητός.

Μία μετάθεση του συνόλου {1,2, . . . ,n} είναι μία 1-1 απεικόνιση του συνόλου αυτού στον

εαυτό του.

'Εστω ότι για κάποιο n ε Ν* ισχύει ότι � σ(l) + �σ(2) + �· ·· + �σ(n) = 'ί Ε Q. (1) Θεωρώντας τα τετράγωνα των δύο μελών της (1) λαμβάνουμε ότι και ο αριθμός � σ(2) + � σ(3) + �· · · + Γσ(1ί) είναι ρητός. Ομοίως λαμβάνουμε ότι, για κάθε k ε { 1, 2, ... , n} , ο αριθμός

rk := � σ(k)+ �σ(k+1)+�···+Γσ(!ί) EQ.

Στη συνέχεια ορίζουμε ak : = � n + � n + �· · · + J;ι , για κάθε k ε Ν* •

k

ΕΥΚΛΕΙΔΗΣ Β ' 65 τ. l/1 1

Page 14: Ευκλειδης Β 65

--------- Μαθηματικοί Διαγωνισμοί - Μαθηματικές Ολυμπιάδες --------­

Εύκολα μπορούμε με επαγωγή να αποδείξουμε ότι ak < .J;; + 1 , για κάθε k Ε Ν*, οπότε θα έχουμε

'ί= � σ(1)+�σ(2)+�···+�σ(n) <an<Fn+1. Αν f. ο θετικός ακέραιος που ικανοποιεί τις ανισότητες: f.

2 � n < ( f. + 1 ) 2 , τότε για κάποιο ί Ε { 1 ,2, ... ,n} θα έχουμε σ(ί) = f.2• Διακρίνουμε δύο περιπτώσεις:

!'η ί * n Τότε έχουμε .e < �Γ.e2-+ι�=σ=(ί=+ =1)=+=�=;·Ξ· Ξ. +ΞΓσ(n}ΞΞσΞ( ΞnΞ) < Fn + 1 < .e + 2,

από την οποία έπεται ότι � .f.2 + �σ(ί + 1) + �· · · + �σ(n) = .e + 1 . (2) Με ύψωση στο τετράγωνο των δύο μελών της (2) λαμβ άνουμε 2e + 1 =�Γσ(_ί_+ -1)-+-�-;:.= .. =+=JΞσ(ΞnΞ) < .J;ι +Ι< .e + 2, από την οποία έπεται ότι f. < 1 , που είναι άτοπο.

,-'- .. �-, --�η ί = n Αν είναι f. > 1 , τότε f.2 -1 Ε { σ(1 ) , ... ,σ( n -1)} . 'Εστω τώρα j < n τέτοιο ώστε σ(J) = f.2

-1.

Ομοίως, όπως στην πρώτη περίπτωση λαμβ άνουμε .e < � .f.2 -1 + � σ(j + 1) + �· · · + .Ji2 < Fn + 1 < .e + 2 � � f2 -1 + � σ(j + 1) + �· · · + .Ji2 = .e + 1

� 2.e + 2 =�σ(}+ 1)+�··· + .Ji2 < Fn + 1 < .e + 2, που είναι άτοπο. Αν f. = 1 , τότε n ε {1,2,3} . Με έλεγχο όλων των περιπτώσεων είναι εύκολο να διαπιστώσουμε ότι για n = 1 και για n = 3 υπάρχουν με-ταθέσεις που ικανοποιούν τη ζητούμενη σχέση. Αναλυτικότε-ρα, για n = 1 έχουμε Jϊ = 1 , ενώ για n = 3 έχουμε: J2 + �3 + .Jϊ = 2. Για n = 2 δεν υπάρχει τέ­τοια μετάθεση .

ΓΊ! ,,, -i! Για ένα δεδομένο θετικό ακέραιο n > 2, έστω cl' c2' c3 είναι το σύνορο τριών κυρτών n-

γώνων στο επίπεδο έτσι ώστε τα σύνολα C1 n C2, C2 n C3, C3 n C1 είναι πεπερασμένα. Να βρείτε

το μεγαλύτερο δυνατό αριθμό σημείων του συνόλου C1 n C2 n C3• • \:_ �)�'Fηc Έστω Α και Β δύο διαδοχικά σημεία του συνόλου C, n C2 n C3 , παρατηρούμενα κατά τη φορά κί-

νησης των δεικτών του ωρολογίου από ένα σημείο εσωτερικό και των τριών πολυγώνων. Εξετάζουμε τώρα για κάθε σύνορο Ci το τμήμα του κατά τη φορά κίνησης των δεικτών του ωρολογίου μεταξύ των Α και Β εξαιρουμένων των δύο αυτών σημείων. Αν δύο από αυτά τα μέρη δεν περιέχουν κάποιες κορυ­φές των αντίστοιχων πολυγώνων, τότε το ευθύγραμμο τμήμα ΑΒ ανήκει και στα δύο πολύγωνα, που εί­ναι άτοπο. Έτσι δύο τουλάχιστον από αυτά τα τμήματα έχουν μία τουλάχιστον κορυφή το καθένα και επιπλέον δεν περιέχουν το ευθύγραμμο τμήμα. Προφανώς δύο τέτιες κορυφές υπάρχουν. Επειδή υπάρ­χουν ιc, n C2 n C3 1 διαδοχικά σημεία Α και Β του C, n C2 n C3, πρέπει να υπάρχουν τουλάχιστον 2IC, n c2 n c3ι διαφορετικές κορυφές στα τρία πολύγωνα. Έτσι έχουμε 2IC, n c2 n c3ι s 3n' δηλαδή

ιc, n c2 Γ'ι c3ι s ι 3; J' n;::.: 3' ( αφού ιc, Γ'ι c2 n c3ι είναι ακέραιος).

Η επίτευξη του ανωτέρω φράγματος μπορεί να αποδειχθεί ως εξής: Κατασκευάζουμε δύο κανονικά n- γωνα C1, C2 με το ίδιο κέντρο και τέτοια ώστε τα σημεία τομής

ΕΥΚΛΕΙΔΗΣ Β ' 65 τ.l/12

Page 15: Ευκλειδης Β 65

--------- Μαθηματικοί Διαγωνισμοί - Μαθηματικές Ολυμπιάδες

τους να σχηματίζουν ένα κανονικό 2n- γωνο. Ονομάζουμε τις κορυφές αυτού του 2n- γώνου Ρι, Pz, . . . , ?zn με κυκλική σειρά και C τον περιγεγραμμένο κύκλ ο του. Στη συνέχεια θεωρούμε ως πολύ-γωνο C3 , το πολύγωνο με n πλευρές που φράσσεται από τις ευθείες Ρι �, f>sPι , �Ρι 1•••• (περιέχεται και η πλευρά �Ρι , αν ο n είναι περιττός) και από τις εφαπτόμενες του κύκλ ου C στα σημεία �, Pg, Ρι2 , •••

. Για τα πολύγωνα που θεωρήσαμε είναι φανερό ότι ισχύει η ισότητα I c, Γ'ι c2 (1 c3ι = ι 32n J' n � 3 ο

'''·'''"'''Ι;c Βιετν άμ (Αν όϊ - Χά λογ κ), 1 9- 3 1 Ιουλίου 2007 . . :νμi':•·•ι> \ιι·:· X·i\ •'·'' Ha Huy Khoai

) ηιψ;. . •'λ(ι 93 χώρες και 2 μόν ο με παρατηρητές. Ι ι.f'·i χ-ι.:'.ι,: 1 •') : .. Ρ ωσία 184, Κ ίν α 184, Βιετ νάμ κ αι Ν. Κο ρέα 168, ΗΠΑ 155, Ιαπωνία κα ι Ουκρα­

νία 154, Β. Κορέα 1 5 1, Βουλγαρία κ αι Τ αϊβ άν 1 49. Η Ελληνική ομάδα κατέλαβ ε την 33η θέση με 89 β αθμούς και την αποτελούσαν οι μαθητές:

. ,

αργυρό μετάλλιο, χάλκινο μετάλλιο, χάλκινο μετάλλιο,

χάλκινο μετάλλιο, εύφημη μνεία, εύφημη μνεία .

Σ υνοδοί της αποστολής ήταν ο Αναπληρωτής καθηγητής του Ε. Μ. Πολυτεχν είου Ανάρ -yυρος Φε λλούρης

ως αρχηγός και ο Επί κουρος καθηγητής του Πανεπιστημίου Αθηνών Γεώργιος Δημάκος ως υπαρχηγός.

Δίνονται οι πραγματικοί αριθμοί a1 ,a2 , • • • ,an . Για κάθε i,(l::;; ί::;; n) ορίζουμε d; =max{a1 :l::;;j::;;i}-min{a1 :i::;;j::;;n} και έστω d=max{d; : ι::;;;::;;n} . (α) Για οποιουσδήποτε πραγματικούς αριθμούς Χι, χ2, • • • , xn με Χι ::;; χ2 ::;; • • • ::;; xn , να αποδείξετε ότι

max {/χ; -α;/ : 1 ::;; ί ::;; n} � � . (*) (β) Να αποδείξετε ότι υπάρχουν πραγματικοί αριθμοί x1,x2, ••• ,xn με χ1 ::;;χ2::;; • • • ::;; xn έτσι ώστε

να ισχύει η ισότητα στη σχέση (*) .

'Ε 1 < < < < ' δ ' ' ' d d - { ·1< "<"} - · { · "< "< } στω _p_q_r_ n ειναι εικτες τετοιοι ωστε = q' αΡ- ΠΒΧ α1. -J-l , α,- mn α1.ι_;_n ,

οπότε d = α Ρ - αq. ( Οι δείκτες αυτοί δεν είναι αναγκαστικά μοναδικοί.) Για τυχαίους πραγματικούς αριθμούς x1,x2 , ••• ,xn με χ1 ::;;χ2 ::;; ••• � xn θεωρούμε τις ποσότητες

ΙχΡ - α ΡI και /χ, - α, / . Επειδή (α Ρ -χ Ρ ) + (χ, -α, ) = ( α Ρ -α, ) + (χ, -χ Ρ ) � α Ρ -α, = d ,

θ , , d , dE

, θ ' α εχουμε οτι α!' -χΙ' � - η χ,. -α, � - . πομενως α ειναι 2 2 max {/x; -α; / : 1::;; ί ::;; n} � max {lxP -αΡ I,/χ,. -α,. /} � max {αP -χΡ,χ, -α,} � � .

) : ' '' · ; Ορίζουμε τους πραγματικούς αριθμούς χ1, χ2 , ••• , xn ως εξής:

χ1 =α1- � και xk = max {xk_1,αk - �} . για 2 � k�n.

ΕΥΚΛΕΙΔΗΣ Β ' 65 τ.l/13

Page 16: Ευκλειδης Β 65

--------- Μαθηματικοί Διαγωνισμοί- Μαθηματικές Ολυμπιάδες

Θα αποδείξουμε ότι για τους παραπάνω αριθμούς ισχύει η ισότητα στη σχέση (*) . Πράγματι, σύμφωνα

με τον ορισμό τους θα ισχύουν Χι �χ2 � ... � xn και xk -αk �-d

, για 1 � k � n. 2

/ d Στη συνέχεια θα αποδείξουμε ότι xk -αk � - , για 1 � k � n . 2 Θεωρούμε ένα τυχαίο δείκτη k με 1 � k � n . Έστω R � k ο ελάχιστος δείκτης που είναι τέτοιος ώστε

xk = Xe· Θα έχουμε ότι R = 1 ή R � 2 και xt � Χ e-ι . Και στις δύο περιπτώσεις έχουμε d xk = Xe = ae -2. ( 1 )

Επειδή αe -αk � max {αj: 1 �j � k} -max {αj: k � j � n} = dk � d,

d d d από την ισότητα ( 1 ) έπεται ότι χ -α = α -α -- � d--=-. k k e k 2 2 2

'Ετσι έχουμε βρει μέχρι τώρα ότι - d � xk -αk �

d, για κάθε k = 1, 2, ... , n δηλαδή θα είναι

2 2

{I I 1 . } d

, , , , , I I d

max xi -αi : � ι � n � - , οποτε η ισοτητα επεται απο τη σχεση Χι -αι =-. 2 2

Γ 'θ . 1 2 ' Μ { 1 < . < ·} . { . < . < } ια κα ε ι= , , ... ,n, εστω i = αj: _ J _ι και mi = mιn αj: ι_ J _ n .

Για ί = 1,2, ... ,n- 1 έχουμε Mi =maχ {αι, ... ,αi } �maχ {αι, ... ,αi,αi+ι } =Μi+ι } mi = min {αi,αi+ι, ... ,αn } � min {αi+ι, ... ,αn } = mi+ι

·

Από τον ορισμό των Mi και mi έχουμε ότι mi � αί � Mi , ί = 1, 2, ... , n.

Για να επιτύχουμε να ισχύει η ισότητα στη σχέση (*), θέτουμε Mi +mi . 1 2 xi= ,ι= , , ... ,n . 2

(2)

Πράγματι, λόγω των ( ) ισχύει ότι Χι � χ2 � ... � xn και επιπλέον από την ισότητα di = M i -mi

παρατηρούμε ότι d m.-M M-m. d _

_ ι= ι ι =χ.-Μ. �χ.-α. �x.-m. = ι ι=____!_ 2 2 ι ι ι ι ι ι 2 2'

οπότε {I - ι · 1 < . < } < { di · 1 < . < } - d max xi αi . _ ι _ n _ max

2 . - ι - n -

2 .

d Λόγω του (3) και του (α) έπεται ότι: max { lxi -ai I : 1 � ί � n} =-.

2

Θεωρούμε πέντε σημεία A,B,C,D και Ε, τέτοια

ώστε το τετράπλευρο ABCD είναι παραλληλό­

γ ραμμο και το τετράπλευρο BCED είναι εγγράψι­

μο. Έστω R μία ευθεία που περνάει από το Α. Υ πο­

θέτουμε ότι η R τέμνει το εσωτερικό του ευθύ­

γραμμου τμήματος DC στο σημείο F και την ευθεία

BC στο G. Υποθέτουμε επίσης ότι EF=EG=EC. Να αποδείξετε ότι η ευθεία R είναι διχοτόμος

της γωνίας DAB. -

Επειδή DAG = AGB, για να είναι η AG διχο-τόμος της γωνίας DAB αρκεί να ισχύει

--- --- ---- ----

BAG=AGB ή CFG=AGB ή CF=CG.

ΕΥΚΛΕΙΔΗΣ Β ' 65 τ. l/14

Α Σχήμα 1

(3)

D

Page 17: Ευκλειδης Β 65

--------- Μαθηματικοί Διαγωνισμοί - Μαθηματικές Ολυμπιάδες -------­

Θεωρούμε ΕΚ l_ CF και EL l_ CG . Επειδή τα τρίγωνα ADF και GCF είναι όμοια, έχουμε AD = DF

::::} BC = DF

::::} BC = DF

::::} BC + CL = DF + FK

::::} BL = DK

::::} BL = CL . (1)

GC CF GC CF CL CK CL CK CL CK DK CK Επειδή το τετράπλευρο BCED είναι εγγράψιμο, έχουμε ότι CBE = EDC , οπότε τα ορθογώνια

τρίγωνα BLE και DKE είναι όμοια. Άρα BL EL

-

= -DK ΕΚ

(2)

Α , ( 1) (2) , , CL EL , πο τις και επεται οτι - = - , οποτε CK ΕΚ

τα τρίγωνα CLE και CKE είναι όμοια και επειδή έχουν κοινή υποτείνουσα θα είναι CL = CK και CG=CF .

Όπως παραπάνω θα αποδείξουμε ότι CG = CF . Επειδή το σημείο Ε ανήκει στον περιγε­γραμμένο κύκλο του τριγώνου BCD, αν φέρουμε τις ΕΚ l_ CD και EL l_ BG , τότε η ευθεία KL

Β c

Α είναι η ευθεία Simpson από το σημείο Ε του τρι- Σχήμα 2 γώνου BCD.

L

D

Όμως στο τρίγωνο CFG η ΚL συνδέει τα μέσα δύο πλευρών του, οπότε είναι ΚL 11 FG και ΚL 11 Α G . Επομένως στο τρίγωνο Α CG η ευθεία KL περνάει από το μέσο Ο της πλευράς Α C, άρα και από το μέσο Ο της διαγωνίου BD. Άρα ΕΟ l_ BD, αφού η ευθεία ΚL είναι η ευθεία Simpson από το σημείο Ε του τριγώνου BCD.

Επομένως στο τρίγωνο EBD η ΕΟ είναι ύψος και διάμεσος, οπότε θα έχουμε ΒΕ = ED . Τότε τα τρίγωνα BLE και DKE είναι ίσα με συνέπεια την ισότητα πλευρών ΚΕ = EL , οπότε θα είναι CL = CK και CG = CF .

--- ----Έστω DAG=θ, ΒΑG=φ, οπότε θα είναι DFA=CFG=CEL=LEG=φ και -- -- --AGC = CEK = KEF =θ. Για απλοποίηση των πράξεων θεωρούμε ότι EC = EF = EG = 1 (η ακτίνα του περιγεγραμμένου κύκλου του τριγώνου CFG ). Έχουμε

CL =LG =sinφ, LE = cosφ, CK = ΚF =sinθ, ΚΕ = cosθ . AD Επειδή τα τρίγωνα ADF και CGF είναι όμοια με λόγο ομοιότητας, έστω -= λ , έπεται ότι: CG

BC = AD = λCG = 2λsinφ και DF = λCF = 2λsinθ, οπότε λαμβάνουμε BL = ( 2λ + 1 ) sin φ και DK = ( 2λ + 1 ) sin θ .

Επειδή το σημείο Ε βρίσκεται στον περιγεγραμμένο κύκλο του τριγώνου ---- -- ---- -- EL ΕΚ cos φ EBL = EDK , οπότε θα είναι tan EBL = tan EDK � -=- �

( ) BL DK 2λ+ 1 sinφ � tanφ = tanθ � φ =θ . (αφού σ <θ, φ<1800).

BCD έπεται ότι cosθ

(2λ+ 1) sinθ

Θεωρούμε ορθοκανονικό σύστημα συντεταγμένων με αρχή Α (Ο, Ο) και τέτοιο ώστε να είναι D (α, Ο), Β ( b, c) με α >Ο, c -:f:. Ο. Επειδή το τετράπλευ-ρο ABCD είναι παραλληλόγραμμο έπεται ότι C(α+b, c) και G(b+d, c) , όπου Ο< α < d.

ΕΥΚΛΕΙΔΗΣ Β ' 65 τ.l/15

Page 18: Ευκλειδης Β 65

--------- Μαθηματικοί Διαγωνισμοί- Μαθηματικές Ολυμπιάδες --------

0(0, 0)

Α(Ο,Ο) D(α , Ο) Σχήμα 4

Σχήμα 3

Αρκεί να αποδείξουμε ότι: AB=HJ� =d. Κατ' αρχήν εύκολα αποδεικνύουμε το επόμενο λήμμα: Σε τρίγωνο με κορυφές Ο (Ο, Ο) , S ( χ1 , y) και Τ ( χ2, y), το κέντρο Μ του περιγεγραμμένου

κυκλου του εχει συντεταγμενες 1 2 , 1 2 •

, , , ( χ + χ y2 - χ χ )

2 2y Επειδή το κέντρο Μ του περιγεγραμμένου κύκλου ανήκει στη μεσοκάθετη της πλευράς ST, θα

είναι της μορφής Μ( Χι � χ2 , w) , όπου ΟΜ = SM. Επομένως έχουμε

ΟΜ2 = SM2 <:::::> (Χι �χ2 )2 + w2 = ( χ2 ;Χι )2 + ( w- yγ <:::::> 2yw-y2 = -χιχ2 <:::::> w = y2 ;;ιχ2.

Επειδή το Ε είναι το κέντρο του περιγεγραμμένου κύκλου του τριγώνου BCD, εύκολα υπολογί­ζουμε τις συντεταγμένες του εφαρμόζοντας το λήμμα 1 στο τρίγωνο BCD μετά από κατάλληλη παράλ-ληλη μετατόπιση , οι οποίες είναι (α� 2b, ab-�; + c2 ) •

Άρα η εξίσωση του περιγεγραμμένου κύκλου του τριγώνου BCD είναι (χ- a�2b )' +- ab-� +c' )' � ( -a;2b )' +( ab-�: +c' )' Αν Ε' είναι το κέντρο του περιγεγραμμένου κύκλου του τριγώνου ABG, τότε σύμφωνα με το λήμ-

μα 1, θα είναι Ε' ( 2b; d , -b2 + ;: -bd) . Επιπλέον ισχύει ΑΕ = � [ ( d - α ) ΑΕ' + α Α G J , οπότε θα είναι

Ε ( α + 2b + d -bd2 + ( αb - b2 + c2 )d + α ( b2 + c2 )J. 2 ' 2cd

Επειδή το σημείο Ε βρίσκεται στον περιγεγραμμένο κύκλο του τριγώνου BCD θα ισχύει

( �)' +( -bd' +:c�' + c') J' � (-α; 2b )' +( ab -�: +c' )' ΕΥΚΛΕΙΔΗΣ Β' 65 τ. l/16

Page 19: Ευκλειδης Β 65

--------- Μαθηματικοί Διαγωνισμοί - Μαθηματικές Ολυμπιάδες

b2 2 b2 2 <::::> 4c; ;2 [ d4 -2abd2 + α2 ( b2 + c2) J = 4;z

c ( α2 + bz + c2 -2ab) b2 2 b2 2

<::::> 4c; ;2 [ d4 -( α2 + b2 + c2) d2 + α2 ( bz + cz) J = Ο <::::> 4c; ;2 ( dz - α2) [ dz - ( bz + cz) J = Ο '

από την οποία έπεται ότι d = .J b2 + c2 , αφού Ο < α < d .

'ι'

Σε ένα μαθηματικό διαγωνισμό μερικοί διαγωνιζόμενοι είναι φίλοι. Η φιλία είναι πάντοτε αμοι­

βαία. Ονομάζουμε μία ομάδα διαγωνιζομένων παρέα, αν κάθε δύο μέλη της είναι φίλοι. (Ειδικά,

κάθε ομάδα με λιγότερα από δύο μέλη αποτελεί μια παρέα). Ο αριθμός των μελών μιας παρέας ονο­

μάζεται μέγεθος της παρέας. Δίνεται ότι, σε αυτόν το διαγωνισμό, το μεγαλύτερο μέγεθος μιας παρέας είναι άρτιος αριθμός.

Να αποδείξετε ότι οι διαγωνιζόμενοι μπορούν να κατανεμηθούν σε δύο δωμάτια έτσι ώστε το μεγα­

λύτερο μέγεθος από τις παρέες που βρίσκονται στο ένα δωμάτιο να είναι ίσο με το μεγαλύτερο μέ­

γεθος από τις παρέες που βρίσκονται στο άλλο δωμάτιο.

Θα παρου σιάσου με έναν αλγόριθμο για την κατανομή των διαγωνιζόμεν ων. Ονομάζου με τα δύο δωμάτια ως δωμάτιο Α και δωμάτιο Β. Θα αρχίσου με με μια αρχική τυ χαία κατανομή των διαγωνιζόμεν ων και θα την τροποποιήσου με αρκετές φορές στέλνοντας ένα άτομο από το έν α δωμάτιο στο Ci)..).s). Σ ε κάθε βήμα του αλ­γόριθμου , θα σημειώνου με με Α και Β το σύνολο των διαγωνιζόμενων στο δωμάτιο Α και Β, αντίστοιχα, και με c (Α ) και c (Β) το μέγιστο μέγεθος από τις παρέες που βρίσκονται στο δωμάτιο Α και Β, αντίστοιχα.

Έστω Μ μία παρέα μέγιστου μεγέθους με lM I = 2m. Τοποθετούμε όλα τα μέ).η της Μ στο δωμάτιο

Α και όλους τους υπόλοιπους διαγωνιζόμε-νους στο δωμάτιο Β.

Επειδή η Μ είναι παρέα μέγιστου μεγέθου ς, έχου με c (Α) = lM I � c (Β) . , ' ' Καθόσον είναι c (Α ) > c (Β) στέλνουμε ένα άτομο από το δωμάτιο Α στο δωμάτιο Β.

Από τη σχέση c( Α ) > c(B) έπεται ότι το δωμάτιο Α δεν μένει κενό. Σ ε κάθε μετακίνηση ενός ατόμου ο αριθμός c(A) μειώνεται κατά 1, ενώ ο αριθμός c(B) αυ ξάνεται το πολύ κατά Ι . Έτσι, στο τέλος αυ -τού του βήματος θα έχου με c( Α ) ::; c(B)::; c( Α ) + 1 .

Δωμάτιο Α Δωμάτιο Β

00 ο ο �ο

ο 00 ο

ο

ο

Επιπλέον, στο τέλος της διαδικασίας του βήματος αυ τού θα έχου με ότι c ( Α ) = !ΑΙ� m.

Πράγματι, αν ήταν c (Α ) = IAI < m, τότε θα είχαμε του λάχιστον m + 1 μέλη της παρέας Μ στο δω­μάτιο Β και το πολύ m -1 μέλη της παρέας Μ στο δωμάτιο Α με συ νέπεια c (B) -c (A)�(mt1)-(�1) =2. : · Έστω k = c ( Α ). Αν είναι c (B ) = k , τότε η διαδικασία τελειώνει, αφού τότε c(A) =c (B) = k. Διαφορετικά θα έχου με c(B) = k+ 1 και από τα παραπάνω προκύ πτει ότι k= IAI = IA� �m και IBnM1 � m.

ΕΥΚΛΕΙΔΗΣ Β ' 65 τ. l/17

Page 20: Ευκλειδης Β 65

--------- Μαθηματικοί Διαγωνισμοί- Μαθηματικές Ολυμπιάδες --------­

Βήμα 4. Αν υπάρχει διαγωνιζόμενος χ ε Β n Μ και παρέα C c Β έτσι ώστε ιcι = k + 1 και χ� C, τότε μετακινούμε το διαγωνιζόμενο χ στο δωμάτιο Α και σταματάμε.

BnM ο AnM ο

00 ο ο c 0 0 00 ο ο ο ο

Μετά τη μετακίνηση του χ στο δωμάτιο Α θα έχουμε k + 1 μέλη της παρέας Μ στο δωμάτιο Α. ο­πότε c( Α ) = k + 1 . Επειδή χ� C, ο αριθμός c(B) = ιcι δεν μειώνεται, οπότε c( Α ) = c(B) = k + 1 .

Αν δεν υπάρχει τέτοιος διαγωνιζόμενος, όπως παραπάνω, τότε όλες οι παρέες μεγέθους k + 1 στο δωμάτιο Β περιέχουν το σύνολο Β n Μ ως υποσύνολο. Βήμα 5. Όσο ισχύει c (Β)= k + 1 , επιλέγουμε μία παρέα C c Β τέτοια ώστε ιcι = k + 1 και μετακινούμε

ένα μέλος της παρέας C- Μ στο δωμάτιο Α.

Σημειώνουμε ότι ιcι = k + 1 > m � IB (\ Ml ' οπότε το σύνολο c- Μ είναι διάφορο του κενού συνόλου. Δωμάτιο Α Δωμάτιο Β

AnM c 0 0 BnM0

�ο ο Κάθε φορά που μετακινούμε ένα άτομο από το δωμάτιο Β στο δωμάτιο Α, ο αριθμός c(B) μειώνε­

ται το πολύ κατά Ι . Επομένως στο τέλος αυτής της διαδικασίας θα έχουμε c(B) = k Στο δωμάτιο Α έχουμε τώρα την παρέα Α n Μ με μέγεθος IA n Ml = k, οπότε c (Α )� k . Θα α­

ποδείξουμε ότι δεν υπάρχει στο δωμάτιο αυτό παρέα μεγαλύτερου μεγέθους. Έστω Q c Α τυχούσα

παρέα του δωματίου Α. Θα αποδείξουμε ότι IQI � k . Δωμάτιο Α Δωμάτιο Β

AnM Q BnM ο ο ο ο

ο ο ο 00 ο ο ο

ΕΥΚΛΕΙΔΗΣ Β' 65 τ.l/18

Page 21: Ευκλειδης Β 65

--------- Μαθηματικοί Διαγωνισμοί - Μαθηματικές Ολυμπιάδες

Στο δωμάτιο Α και ειδικότερα στη παρέα Q υπάρχουν δύο τύποι διαγωνιζόμενων: • Μερικά μέλη της παρέας Μ. Όλοι αυτοί είναι φίλοι με όλα τα μέλη του συνόλου (παρέας) Β n Μ . • Διαγωνιζόμενοι που μετακινήθηκαν στο δωμάτιο Α στο βήμα 5 . Καθένας από αυτούς έχει βρεθεί σε

μία παρέα με το σύνολο Β n Μ , οπότε αυτοί είναι επίσης φίλοι με όλα τα μέλη της παρέας Β n Μ . Επομένως, όλα τα μέλη της παρέας Q είναι φίλοι με όλα τα μέλη της παρεας Β n Μ , οπότε και το

σύνολο Q υ (Β n Μ) είναι επίσης παρέα. Επειδή η παρέα Μ είναι μέγιστου μεγέθους, θα έχουμε

Οπότε IM I � JQ υ (Β nM)J = IQ I + IB nMI = IQ I + IM I - IA nMI ,

IQ I � IA nMI = k . Τελικά, μετά το βήμα 5 θα έχουμε c ( Α ) = c(B) = k .

Π αρατή ρηση . Είναι προφανές ότι

το

ζητούμενο δεν ισχύει όταν το μέγιστο μέγεθος από τις υπάρχουσες

παρέες είναι αριθμός περιττός.

Π ρόβλη μα 4 Σε τρίγωνο ABC η διχοτόμος της γωνίας

BCA τέμνει ξανά τον περιγεγραμμένο κύκλο του

στο R, τη μεσοκάθετη της πλευράς BC στο Ρ και

τη μεσοκάθετη της πλευράς AC στο Q. Το μέσον

της πλευράς BC είναι το σημείο Κ και το μέσον

της πλευράς AC είναι το σημείο L. Να αποδείξετε

ότι τα τρίγωνα RPK και RQL έχουν ίσα εμβαδά .

. \ ΊίJση Αν είναι AC = BC , τότε το τρίγωνο ABC εί­

ναι ισοσκελές, οπότε τα τρίγωνα RPK και RQL είναι συμμετρικά ως προς τη διχοτόμο CR και προ­φανώς έχουν ίσα εμβαδά.

Αν είναι AC 7:- BC, τότε χωρίς βλάβη της γενι-κότητας μπορούμε να υποθέσουμε ότι AC < BC . Σχήμα 5

Έστω Ο το κέντρο του περιγεγραμμένου κύκλου του τριγώνου ABC . Τα ορθογώνια τρίγωνα CLQ και CKP έχουν ίσες τις γωνίες της κορυφής C , οπότε είναι όμοια.

Άρα έχουμε QL CQ - = - (1) ΡΚ CP

Έστω R η μεσοκάθετη της χορδής CR , η οποία βεβαίως περνάει από το Ο . --- ---- ---- ---

Επειδή OPQ = KPC = LQC = OQP το τρίγωνο OPQ είναι ισοσκελές με ΟΡ = OQ . Επομένως η ευθεία R είναι άξονας συμμετρίας του τριγώνου OPQ , οπότε RP = CQ και RQ = CP . (2)

Τα τρίγωνα RQL και RPK έχουν ίσες τις γωνίες τους στις κορυφές Q και Ρ , οπότε θα ισχύει Ι -

(RQL) _

2_ RQ ·QL · sin RQL

(RPK ) -}_ RP ·PK · sin fiPΚ 2

(3)

οπότε από τις ( 1) και (2) η σχέση (3) γίνεται: ( RQL)

=

RQ · QL =

CP · CQ = Ι . οπότε ( RQL) = ( RPK) . ( RPK) RP ΡΚ CQ CP

2"' τρόπος.

Έστω r η ακτίνα του περιγεγραμμένου κύκλου του τριγώνου ABC.

ΕΥΚΛΕΙΔΗΣ Β ' 65 τ.l/19

Page 22: Ευκλειδης Β 65

--------- Μαθηματικοί Διαγωνισμοί - Μαθηματικές Ολυμπιάδες

Όπως προηγουμένως αποδεικνύουμε ότι ΟΡ = OQ , οπότε θεωρώντας τις δυνάμεις των σημείων Ρ και Q ως προς τον περιγεγραμμένο κύκλο του τριγώνου ABC λαμβάνουμε

Όμως έχουμε

. CQ · QR = r2 - OQ2 = r2 - ΟΡ2 = CP · PR . (4)

(RPK) =!_PR - PK · sinΚiR =!_PR · PC · sin C · sin (9oo + C) =!_PR · PC · sin C · cos C =!_PR - PC · sin C 2 2 2 2 2 2 2 4

και ομοίως ( RQL) = .!_ CQ · QR · sin C , 4

οπότε λόγω της (4) προκύπτει η ζητούμενη ισότητα.

'Εστω α και b θετικοί ακέραιοι. Να αποδείξετε ότι, αν ο 4ab - 1 διαιρεί τον { 4a2 - 1 }2 , τότε

a = b .

Ονομάζουμε ένα ζεύγος (α, b ) θiτικών ακεραίων κακό, αν ο 4αb - 1 διαι-ρεί τον ( 4α2 - 1 )2 και

ισχύει α :�; b . Για να αποδείξουμε ότι δεν υπάρχουν τέτοια κακά ζεύγη , θα παρουσιάσουμε δύο ιδιότητες αυτών των ζευγών οι οποίες οδηγούν σε μία άπειρη κάθοδο (infinite descent) . Ιδιότητα (i). Αν (α, b ) είναι ένα κακό ζεύγος με α < b , τότε υπάρχει θετικός ακέραιος c < α έτσι ώστε το

ζεύγος (α, c) να είναι επίσης κακό.

Πράγματι, αν ( 4α2 - 1 )2 = r ( 4αb - 1) , τότε r = -r · ( -1) = ( -r )( 4αb - 1) = - ( 4α2 - 1 )2 = -1 ( mod4α) ,

(4d -1)2 οπότε θα είναι r = 4αc - 1 , για κάποιο θετικό ακέραιο c . Για α < b έχουμε 4ac-1 < 4d -1 ,

4ab-1 οπότε θα είναι c < α . Από τον ορισμό του ο αριθμός 4αc - 1 διαιρεί τον αριθμό ( 4α2 - 1 )2 , οπότε το

ζεύγος (α, c) είναι κακό.

Αν (α, b ) είναι ένα κακό ζεύγος, τότε και το ζεύγος ( b, α ) είναι επίσης κακό.

Πράγματι, επειδή είναι 1 = 12 = ( 4αb )2 ( mod ( 4αb - 1 ) ) , έχουμε

(4b2 - 1 )2 = (4b2 - (4αbγ )2 = 1 6b4 (4α2 - 1 )2 = 0 (mod (4αb - 1 ) ) ,

οπότε ο αριθμός 4αb - 1 διαιρεί και τον αριθμό ( 4b2 - 1 )2 . Τώρα υποθέτουμε ότι υπάρχει ένα τουλάχιστον κακό ζεύγος. Θεωρούμε ένα τέτοιο ζεύγος (α, b) τέ­

τοιο ώστε το άθροισμα 2α + b να είναι ελάχιστο και διακρίνουμε τις περιπτώσεις:

Αν α < b , τότε η ιδιότητα (ί) μας δίνει ένα ακόμη κακό ζεύγος (α, c) με c < α < b , οπότε 2α + c < 2α + b , που είναι άτοπο.

Αν b < α , τότε από την ιδιότητα (ίί) το ζεύγος (b, α ) είναι επίσης κακό, ενώ ισχύει

2b + α < 2α + b , που είναι επίσης άτοπο.

ΕΥΚΛΕΙΔΗΣ Β ' 65 τ. l/20

Page 23: Ευκλειδης Β 65

--------- Μαθηματικοί Διαγωνισμοί - Μαθηματικές Ολυμπιάδες

Επομένως δεν μπορεί να υπάρχει κακό ζεύγος, οπότε όταν ο αριθμός 4αc - 1 διαιρεί τον αριθμό

( 4α2 - 1 )2 έπεται ότι α = b.

ότι

Ονομάζουμε, όπως και προηγουμένως, ένα ζεύγος (α, b ) θετικών ακέραιων κακό, αν ο 4αb - 1

διαιρεί τον ( 4α2 - 1 )2 και ισχύει α -:F- b .

'Εστω ( α, b ) ένα κακό ζεύγος, οπότε ο 4αb - 1 1( 4α2 - 1 )2 • Τότε

4αb - 1 1b ( 4α2 - 1 ) 2 - ( 4αb - 1 ) α ( 4α2 - 1 ) = (α - b ) ( 4α2 - 1 ) . Επειδή είναι ( b, 4αb - 1 ) = 1 , ισχύει επίσης και το αντίστροφο. Ομοίως, μπορούμε να αποδείξουμε

4αb - 1 1 (α - b ) ( 4α2 - 1 ) � 4αb - 1 1 (α - b )2 ( 4α2 - 1 ) , οπότε η αρχική υπόθεση είναι ισοδύναμη με την 4αb - 1 1 (α - b )2 ( 4α2 - 1 ) .

Η τελευταία σχέση είναι συμμετρική ως προς α και b , οπότε (α, b ) είναι κακό ζεύγος, αν και μόνον αν, ( b, α ) είναι κακό ζεύγος.

Επομένως, μπορούμε να υποθέσουμε χωρίς βλάβη της γενικότητας για το κακό ζεύγος (α, b ) , ότι

α > b και ότι το κακό ζεύγος που έχουμε επιλέξει έχει το πρώτο στοιχείο του ελάχιστο .

'Εστω (α - b )2 = m (4αb - 1 ) , όπου m θετικός ακέραιος. Τότε α2 + (-2b - 4mb ) α + (b2 + m ) = 0 . Επειδή η παραπάνω δευτεροβάθμια εξίσωση έχει ακέραια ρίζα, η διακρίνουσά της

Δ = ( 2b + 4mb )2 - 4 ( b2 + m ) = 1 6mb2 + 1 6m2 b2 - 4m ,

πρέπει να είναι τέλειο τετράγωνο. Ας υποθέσουμε ότι Δ= 16mb2 +16nfb2 -4m= (2mb+t)2 , όπου O <t < b .

Αν θέσουμε s = b - t , τότε θα έχουμε 1 6mb2 + 1 6m2 b 2 - 4m = 4m2 b 2 + 4mb + t2 � m ( 4b2 - 4bt - 1 ) = t2 � m ( 4b2 - 4b ( b - s ) - 1 ) = ( b - s )2 � m ( 4bs - 1 ) = ( b - s )2 .

Επομένως το ζεύγος ( b, s ) είναι κακό με b < α , που είναι άτοπο.

'. ι:

'Εστω n ένας θετικός ακέραιος. Θεωρούμε το σύνολο S = {(x,y,z) : x,y,z ε {0,1,2, ... ,n} ,x+ y + z > O} ως ένα σύνολο που αποτελείται από { n + 1 )3 - 1 σημεία του τρισδιάστατου χώρου. Να προσδιορίσε­

τε τον ελάχιστο δυνατό αριθμό επιπέδων των οποίων η ένωση περιέχει το σύνολο S, αλλά δεν περιέ­χει το σημείο { 0, 0, 0) .

Είναι εύκολο να βρούμε 3n τέτοια επίπεδα, όπως για παράδειγμα τα επίπεδα με εξισώσεις : χ = ί, ί = 1 , 2, . . . , n (κάθετα στον άξονα των χ στα σημεία ( ί, Ο, Ο) ) y = ί, ί = 1 , 2, . . . , n (κάθετα στον άξονα των y στα σημεία (Ο, ί, Ο ) ) z = ί, ί = 1 , 2, . . . , n (κάθετα στον άξονα των z στα σημεία (Ο, Ο, ί ) )

Πράγματι, η ένωση αυτών των επιπέδων περιέχει το σύνολο S , ενώ κανένα ιιπό αυτά δεν περιέχει το σημείο (0, 0, 0) .

Επίσης τα επίπεδα με εξισώσεις χ+ y + z = k, k = 1, 2, . . . , 3n , ικανοποιούν το ζητούμενο του προβλή­

ματος.

ΕΥΚΛΕΙΔΗΣ Β' 65 τ. l/21

Page 24: Ευκλειδης Β 65

--------- Μαθηματικοί Διαγωνισμοί - Μαθηματικές Ολυμπιάδες --------­

Θα αποδείξουμε ότι 3n είναι ο ελάχιστος αριθμός επιπέδων των οποίων η ένωση περιέχει το σύνολο S , ενώ κανένα από αυτά δεν περιέχει το σημείο (Ο, Ο, Ο) . Για το σκοπό αυτό θα αποδείξουμε πρώτα το

ακόλουθο λήμμα.

Λ.ήμμα I . Έστω Ρ( Χι , • • • , xk ) ένα μη μηδενικό πολυώνυμο k μεταβλητών. Έστω ότι το πολυώνυμο

Ρ( χι , ••• , xk ) μηδενίζεται στα σημεία (Χι , ••• , xk ) που είναι τέτοια ώστε Χι , ••• , xk ε { O, l, ... , n} και

Χι+ χ2 + • • • + xk > Ο , ενώ είναι Ρ( 0, 0, ... , 0) * Ο . Τότε ισχύει ότι deg P � kn .

Απόδειξη . Θα χρησιμοποιήσουμε επαγωγή ως προς το k. Η περίπτωση για k = Ο είναι φανερή , αφού P =F- 0 .

Στη συνέχεια για λόγους ευκολίας συμβολισμού γράφουμε y = xk . Έστω τώρα R ( χ1 , • • • , χk-ι , y) το

υπόλοιπο της διαίρεσης του πολυωνύμου Ρ με το πολυώνυμο Q (y) = y (y - 1 ) · · · (y - n ) . Το πολυώ­

νυμο Q (y) μηδενίζεται για y = 0, 1 , . . . , n , οπότε P(x1 , • • • , xk-ι • Y) = R (x1 , • • • , xk-ι • Y ) , για κάθε

x1 , • • • , xk_1 , y Ε {0, 1 , . . . , n} .

Επομένως και το πολυώνυμο R ( χ1 , • • • , χk- ι , y) ικανοποιεί τη συνθήκη του λήμματος και επιπλέον

degY R (x1 , • • • , xk_ 1 , y) � n . Όμως deg R � deg P , οπότε αρκεί να αποδείξουμε ότι deg R � kn .

Εκφράζουμε τώρα το πολυώνυμο R ως προς τις δυνάμεις του y και έστω

R( χ1 , • • • , χk-Ι • Υ) = Rn ( χ1 , • • • , χk-ι ) yn + Rn-Ι { χ1 , • • • , χk-Ι ) y

n-Ι + . . . + Ro { χ1 , • • • , χk-Ι ) .Θα αποδείξουμε ότι το

πολυώνυμο Rn (χ1 , • • • , χk-Ι ) ικανοποιεί τη συνθήκη της υπόθεσης της επαγωγής. Θεωρούμε το πολυώνυ­

μο Τ (y) = R (Ο, . . . , Ο, y) με βαθμό deg Τ (y) � n . Το πολυώνυμο αυτό έχει n ρίζες y = 1 , 2, . . . , n , ενώ

είναι Τ (y) j. Ο , αφού Τ (Ο) =F- Ο . Άρα είναι deg Τ = n και ο συντελεστής του μεγιστοβάθμιου όρου του

είναι ο Rn ( Ο, Ο, . . . , Ο) * Ο . (Ειδικά για k = 1 ο συντελεστής Rn είναι μη μηδενικός).

Ομοίως, θεωρούμε τυχαίους αριθμούς α1 , • • • , αk-ι Ε {Ο, 1 , . . . , n} έτσι ώστε α1 + . . . + αk- Ι > Ο . Αντικαθι­

στώντας Χ; = α; στο πολυώνυμο R ( χ1 , • • • , χk-Ι , y ) λαμβάνουμε ένα πολυώνυμο μεταβλητής y το οποίο μη­

δενίζεται για y = O, l, . . . , n και έχει βαθμό � n . Άρα, σύμφωνα με γνωστό θεώρημα, το πολυώνυμο

R ( α1 , • • • , αk-Ι , y) είναι το μηδενικό πολυώνυμο, οπότε θα έχουμε R; ( α1 , • • • , αk-Ι ) = Ο , για κάt:ιε ί = Ο, 1 , . . . , n .

Επομένως το πολυώνυμο Rn ( χ1 , • • • , χk-ι ) ικανοποιεί τη συνθήκη της υπόθεσης της επαγωγής, οπότε

θα έχουμε deg Rn � ( k - 1 ) n και deg Ρ � deg R � deg Rn + n � kn . D

Στη συνέχεια θα προσπαθήσουμε να τελειώσουμε τη λύση του προβλήματος, υποθέτοντας ότι υπάρ-χουν Ν επίπεδα με εξισώσεις α;x + b;y + c;z + d; = Ο, ί = 1 , 2, . . . , Ν, που καλύπτουν όλα τα σημεία του συνόλου S , αλλά κανένα δεν περιέχει την αρχή (Ο, Ο, Ο) . Θεωρούμε

το πολυώνυμο Ν

P(x, y, z ) = Π (α;x + b;y + c;z + d; ) ί=l

το οποίο έχει βαθμό Ν . Για το πολυώνυμο αυτό ισχύουν Ρ ( χ0 , y0 , z0 ) = Ο , για κάθε { χ0 , y0 , z0 ) Ε S , ενώ είναι Ρ ( Ο, Ο, Ο) =F- Ο . Επομένως, σύμφωνα με το λήμμα Ι , θα έχουμε Ν = deg Ρ � 3n .

ΕΥΚΛΕΙΔΗΣ Β ' 65 τ. l/22

Page 25: Ευκλειδης Β 65

--------- Μαθη ματικοί Διαγωνισμοί - Μαθηματικές Ολυμπιάδες --------

Η l l η Βαλκανική Μαθηματική Ολυμπιάδα Νέων έγινε στο Σούμεν της Βουλγαρίας από 25 μέχρι και 30 Ιουνίου 2007 .

Η διοργάνωση ήταν άψογη και οι διοργανωτές κατέβαλαν κάθε δυνατή προσπάθεια για την όσο το δυνατόν καλύτερη και πληρέστερη οργάνωση του διαγωνισμού .

Ενδεικτικό είναι το γεγονός ότι ο Υφυπουργός Παιδείας και Επιστημών της Βουλγαρίας παρευρίσκε­το στον τόπο της διοργάνωσης καθ' όλη τη διάρκεια της διοργάνωσης.

Συμμετείχαν συνολικά 1 3 ομάδες. Επίσημα οι ομάδες 1 1 χωρών, Αλβανία, Βοσνία και Ερζεγοβίνη , Βουλγαρία, Ελλάδα, Κύπρος, Μαυροβούνιο, Μολδαβία, ΠΓΔΜ, Ρουμανία, Σερβία, Τουρκία, και ανεπί­σημα οι χώρες: Βουλγαρία 2 , (Σούμεν και Βάρνα), Καζακστάν.

Η Ελληνική ομάδα συμμετείχε με τους μαθητές : Λιάμπη Κωνσταντίνο, Λογοθέτη Φώτιο , Μπραζιτί­κο Κωνσταντίνο, Ντριάνκο Ιωάννη , Παγωνάκη Δημήτριο και Παντέλα Στυλιανή .

Η Ελληνική ομάδα, συνεχίζοντας και φέτος την παράδοση των επιτυχιών των Ελλήνων μαθητών στις Μαθηματικές Ολυμπιάδες, είχε τις παρακάτω διακρίσεις :

Γ Γυμνασίου Γ Γυμνασίου Β ' Γυμνασίου

Αργυρό Μετάλλιο Αργυρό Μετάλλιο Χάλκινο Μετάλλιο

Γ Γυμνασίου Χάλκινο Μετάλλιο Γ Γυμνασίου Χάλκινο Μετάλλιο

Συνοδοί των μαθητών ήταν ο κ. Ανάργυρος Φελλούρης, Αναπληρωτής Καθηγητής του Ε. Μ. Πολυ­τεχνείου ως αρχηγός της Ελληνικής ομάδας και η μαθηματικός Σύλβια Κατσαμένη ως υπαρχηγός.

Το Δ ιοικητικό Συμβούλιο της Ελληνικής Μαθηματικής Εταιρείας ευχαριστεί τις διοικήσεις

του Ελληνικού Κολλεγίου Θεσσαλονίκης, των Εκπαιδευτηρίων Κωστέα Γείτονα και

του παραρτήματος της ΕΜΕ του Νομού Τρικάλων καθώς και τους γονείς κ. Θεόδωρο Λ ογοθέτη και Κωνσταντίνο Παγωνάκη για το ιδιαίτερο ενδιαφέρον τους και την οικονο­

μική προσφορά τους για την πραγματοποίηση του ταξιδιού της Ελληνικής ομάδας στο Shumen της Βουλγαρίας, προκειμένου να συμμετάσχει στην Βαλκανική μαθηματική

Ολυμπιάδα Νέων.

«0 Θαλής» «0 Ευκλείδης» «0 Αρχιμήδης»

.' Ί . · . ι ·

24 Νοεμβρίου 2007 1 9 Ιανουαρίου 2008 23 Φεβρουαρίου 2008

ΕΥΚΛΕ ΙΔΗΣ Β ' 65 τ. l /23

Page 26: Ευκλειδης Β 65

Η Homo Mathematicus είναι μια στήλη στο περιοδικό μας, με σκοπό την ανταλλαγή απόψεων και την ανάπτυξη προβληματισμού πάνω στα εξής θέματα: 1 ) τι είναι τα Μαθηματικά, 2) Πρέπει ή όχι να διδάσκονται, 3) Ποιοι είναι οι κλάδοι των Μαθηματικών και ποιο το αντικείμενο του καθενός, 4) Ποιες είναι οι εφαρμογές τους, 5) Ποιες επιστήμες ή κλάδοι επιστημών απαιτούν καλή γνώση των Μαθηματικών για να μπορέσει κάποιος να τους σπουδάσει. I ιυ τ ο υ .: συ"ψγι!τ ι : .:; τη.: ση]ί .η .: : παράκληση ! τα κείμενα της στήλης αυτής, ως προς το περιεχόμενό τους και ως προς το επίπεδό τους, θα πρέπει να είναι συμβιβαστά με τα ενδιαφέροντα και το επίπεδο κατανόησης από μέρους των παιδιών.

I . "τι ι:ίι ·ω τα Ηα(�ημrι.τtfαί. : " δ . Η αλεξανδ ρ ιν 1] πr: ρ ίοδο.; τ ω ν υ u Ο η μ υ η κι;}�' Η «αλεξανδρινή περίοδος των Μαθηματικών»,

ξεκινά με την ακμή του Ευκλείδη (300 π.Χ.) και τελειώνει με το θάνατο της Υπατίας (4 1 5 μ.Χ.) . Μετά έχουμε τη μακριά νύχτα του Μεσαίωνα. Κατά τη γνώμη μας, το χαρακτηριστικό της «αλεξανδρινής περιόδου των Μαθηματικών», δεν είναι οι θεωρητικές αναζητήσεις πάνω στη φύση των Μαθηματικών (όπως συνέβη, προγενέστερα, την «κλασική περίοδο»), αλλά στο πως και που μπορεί να μας χρησιμεύσουν.

Αυτή την περίοδο έχουμε ένα γαλαξία λαμπρών μαθηματικών-μηχανικών, μαθηματικών­γεωγράφων, μαθηματικών-αστρονόμων. Να ορισμένοι απ' αυτούς [πηγή : www.telemath.gr, Ελληνική Μαθηματική Πύλη] : Ευκλείδης,

1 1 . ".-/ ιπr) το .;..;μα τι: : "

Αρίσταρχος ο Σάμιος, Αρχιμήδης ο Συρακούσιος, Ερατοσθένης ο Κυρηναίος, Απολλώνιος ο Περγαίος, Νικομήδης, Ίππαρχος ο Νικαεύς, Ποσειδώνιος από την Απάμεια, Διονυσόδωρος ο Μήλειος, Διοκλής, Ήρων ο Αλεξανδρινός, Μενέλαος ο Αλεξανδρινός, Πτολεμαίος Κλαύδιος, Διόφαντος ο Αλεξανδρινός, Πάππος ο Αλεξανδρινός, Υπατία. Την ίδια εποχή έχουμε την ακμή της «Βιβλιοθήκης της Αλεξάνδρειας» που ήταν το εκδοτικό κέντρο του τότε γνωστού κόσμου, υπερσκελίζοντας ως προς τον πλούτο των χειρογράφων της κάθε άλλη γνωστή βιβλιοθήκη της εποχής της και του παρελθόντος.

Η «Βιβλιοθήκη της Αλεξάνδρειας» υπήρξε το πρώτο, ίσως, πανεπιστήμιο του κόσμου.

Ι ι ιί: � · ,, · η .; ι..:. ιρ υ σιφ ί i1 η .:

Ποιος θα μπορούσε να κατασκευάσει (στην πράξη), ένα τέλειο κυρτό πολύεδρο, με έδρες κανονικά πολύγωνα; [η απάντηση στο τέλος της στήλης]

1 1 1 . "οι σ υ ι ·ι.ρ; •rί. τι:.; τη .; στψ.η ... - ; •μ rί. φ ο ι; ι ·-ι :{ΗΙJ το ι ! ι · " Σ ' αυτό το τεύχος θα παραβιάσουμε, για μια ακόμα φορά, τη γνωστή σειρά ύλης. Ο λόγος είναι πως

έφτασαν στη στήλη μας αρκετές εργασίες, μερικές από τις οποίες ήταν μεγάλος πειρασμός για μας. Απ' αυτές παρουσιάζουμε τρεις · από τις άλλες θα επιλέγουμε, κάθε φορά, και θα δημοσιεύουμε.

Ι " Θi: μ υ Από το νέο συνάδελφο Κ. Π. Τραϊανό (Λάρισα) λάβαμε και ευχαρίστως δημοσιεύουμε μια εργασία

με τίτλο <<Κατασκευή τριγώνων από τρία στοιχεία» . Όπως μας γράφει, η εργασία αυτή είναι ευρεία περίληψη της ομότιτλης εργασίας του George Berzsenyi που δημοσιεύθηκε στο περιοδικό "QUANTUM" (τόμ. Ι , τεύχ. 3, σελ. 43) . Σε μας ο George Berzsenyi ήταν γνωστός και από μια άλλη παρόμοια εργασία του με τίτλο <<Κατασκευή τριγώνων από τρία δεδομένα σημεία» .

« Κυτασκ: : ιη) τρ ιγ ι;η·ι •Η' ιι.π(J τ ρ ί ιι. στωμ: ίω> , τ ο υ ( ; ι' ο Γgι· Β ιτ z � ι ιη i «Στην τελευταία συνάντηση της Αμερικάνικης Rabinowitz (υπεύθυνο έκδοσης της σειράς Indexes

Μαθηματικής Ένωσης (ΜΑΑ) και της to Mathematical Problems) τον Curtis Cooper, τον Αμερικάνικης Μαθηματικής Εταιρείας (AMS) στο Robert Kennedy ( ο πρώτος είναι συντονιστής Σινσινάτι, απόλαυσα μια καταπληκτική βραδιά με σύνταξης και ο δεύτερος επιμελητής προβλημάτων τέσσερις φίλους μου μαθηματικούς, τον Stanley στο θαυμάσιο Missouri Journal of Mathematics

ΕΥΚΛΕΙΔΗΣ Β' 65 τ.l/24

Page 27: Ευκλειδης Β 65

#�MIIJ NιiT#eNAr/t'l/f -------------­

Sciences), και τον Leroy (Roy) Meyers (επιμελητή Vίsίtor, από το 1 877 ως το 1 896, οι διαγωνισμοί προβλημάτων για πολλά χρόνια στο Mathematics NYSML-ARML, από το 1 989 ως το 1 994, και δύο Magazine ). Επειδή ο Roy συνεργαζόμαστε με τη ακόμη τόμοι από το Jndex to Mathematίcal συντακτική επιτροπή που διευθύνει ο Stanley , Problems, που καλύπτουν τις περιόδους 1 97 5- 1 979 όπως ήταν φυσικό, συζητήσαμε αρκετά για τα και 1 985- 1 989. Μια κι ο Stan και ο Roy βιβλία που πρόκειται να εκδώσει στο άμεσο μέλλον ενδιαφέρονται ιδιαίτερα για τη Γεωμετρία, η η εταιρεία του, η MathProPress. Ανάμεσα σ' αυτά συζήτησή μας στράφηκε σε ορισμένα προβλήματα περιλαμβάνονται οι Μαθηματικές Ολυμπιάδες του αυτής της περιοχής, κι έτσι έμαθα ότι ο Roy είναι Λένινγκραντ, από το 1 987 ως το 1 99 1 , το ένας διακεκριμένος ειδικός στα προβλήματα Προβλήματα και Λύσεις από το Mathematίcal κατασκευασιμότητας τριγώνων.

ο Leroy (Roy) Meyers (για πολλά χρόνια πλευρές α β γ επιμελητής προβλημάτων στο Mathematics γωνίες Α Β Γ Magazine) είναι διακεκριμένος ειδικός στα ύψη hα hβ hγ προβλήματα κατασκευασιμότητας τριγώνων από δεδομένα στοιχεία τους. Συγκεκριμένα ανακάλυψε ότι υπάρχουν 1 86 μη ισομορφικά προβλήματα που προκύπτουν αν επιλέξουμε τρία στοιχεία από τα 1 8 του διπλανού καταλόγου : . . . . . . . . . . . . . . . . . . . . . . . . . . . . . . . . . . . . . . . . . . . . . . . . . . . . . . . . . . .

Βασικά, τα προβλήματα χωρίζονται σε τέσσερις κατηγορίες :

].Πλεονάζουσες τριάδες όπου δύο οποιαδήποτε από τα τρία στοιχεία καθορίζουν το τρίτο. Από τα 1 86 προβλήματα, μόνο τα (Α,Β,Γ), (Α,Β, hy) , (α, Α, R) ανήκουν σ' αυτή την κατηγορία. 1.Μη επιλύσιμα προβλήματα, τα οποία δεν επιλύονται με ευκλείδεια μέσα (δηλ. , με κανόνα και διαβήτη) . Υπάρχουν 27 τέτοιες τριάδες. 3.Επιλύσιμα προβλήματα (με κανόνα και διαβήτη) . Υπάρχουν 1 28 τέτοια προβλήματα. 4.Ανοιχτά προβλήματα. Αυτά παρατίθενται στη συνέχεια, με την αρίθμηση που τους έχει δώσει ο Roy

Για να αποδείξει τη μη επιλυσιμότητα ορισμένων απ' αυτά τα προβλήματα, ο Roy ανακάλυψε το ακόλουθο συχνά χρήσιμο πόρισμα ενός θεωρήματος του Gauss:

Είναι αδύνατο να κατασκευάσουμε με κανόνα και διαβήτη ένα ευθύγραμμο τμήμα του οποίου το μήκος είναι ρίζα ή το αντίθετο της ρίζας μιας κυβικής εξίσωσης με ρητούς συντελεστές που δεν έχει ρητές ρίζες, όταν το μοναδιαίο μήκος είναι δεδομένο.

Για να χρησιμοποιήσει αυτό το θεώρημα, ο Roy aντιστοίχισε προσεκτικά επιλεγμένες ρητές τιμές στα δεδομένα, παρήγαγε μια κυβική εξίσωση με ρίζες τις πλευρές του τριγώνου (ή άλλα θεμελιώδη στοιχεία του) και κατασκευάσιμους συντελεστές, και κατόπιν απέδειξε ότι αυτή η κυβική εξίσωση δεν έχει ρητές ρίζες. Σε κάποια άλλα προβλήματα, τα δεδομένα του τον οδήγησαν

διάμεσοι μα μβ μγ διχοτόμοι δα δ Β δ γ ακτίνα περιγεγραμμένου κύκλου R ακτίνα εγγεγραμμένου κύκλου r ημιπερίμετρος s

7 2 . α μ β δα 1 3 1 . α μβ r

8 1 . hα μα δ β 1 35 . hα μβ r

8 2 . hα μβ δα 1 38 . α δ β r

83. hα μβ δ β 1 42 . hα δ β r

84. hα μ β δy 1 43 . μα δα r

8 6 . μα μ β δy 1 44 . μα δ β r

8 8 . α δ β δy 1 49 . μα R r

8 9 . Α δα δy 1 50 . δα R r

90. Α δ β δy 1 65 . hα μβ s

1 1 ο . hα μβ R 1 72 . hα δ β s

1 1 7 . hα δ β R 1 73 . μα δα s

1 1 8 . � δα R 1 74 . μα δ β s

1 1 9 . μα δ β R 1 79 . μα R s

1 20 . δα δ β R 1 80 . δα R s

στην κατασκευασιμότητα μιας γωνίας 20° ή σε άλλες αδύνατες περιπτώσεις. Πρέπει, πάντως, να επισημάνουμε ότι σε όλες τις περιπτώσεις, ακόμα και όταν η κατασκευή δεν μπορεί να γίνει με ευκλείδεια μέσα, πρέπει να υπάρχει ένα τρίγωνο που να ικανοποιεί τα δεδομένα. Στην πραγματικότητα, μπορούμε να αντιμετωπίσουμε και τα 1 86 προβλήματα ως ανακατασκευή ενός τριγώνου από τα δεδομένα στοιχεία.

Το ενδιαφέρον του Leroy Meyer γι αυτό το είδος των προβλημάτων ξεκινά από την εποχή του λυκείου, όταν όπως και πολλοί άλλοι μαθηματικοί στο ξεκίνημά τους - γοητεύτηκε από την ποικιλία αυτών των προβλημάτων και τις έξυπνες τεχνικές επίλυσής τους. Αργότερα, το ενδιαφέρον του αναθερμάνθηκε από ένα άρθρο του William Wenick στο Mathematics Magazine»

ΕΥΚΛΕΙΔΗΣ Β' 65 τ. l/25

Page 28: Ευκλειδης Β 65

------------------------------ HdArd Af�THEAfAT/CPf -----------------------------

2" Θέμα

Π ρολεγόμενα : Ο μαθητής Κώστας Ελευθερίου (Ηράκλειο), μας έστειλε μια εργασία του Νίκου Δαπόντε, την οποία αλίευσε από την «Εκπαιδευτική Πύλη Νοτίου Αιγαίου» (www.epyna.gr) . Επειδή ο χώρος μας είναι περιορισμένος, δημοσιεύουμε περίληψη αυτής της εργασίας. Ευχαριστούμε τον Ν. Δαπόντε.

«Η αυτοομοιότητα : Μ ια ιδ ιότητα των t"ntct�ι l s» , του Ν . Δαπ()\ιπι:; Η αυτοομοιότητα στην Ευ κλείδεια Γεωμετρ ί α . Στην Ευκλείδεια Γεωμετρία έχει καθιερωθεί η έννοια "ομοιότητα" . Δύο ευθύγραμμα γεωμετρικά

σχήματα είναι όμοια αν έχουν την ίδια μορφή ανεξάρτητα από το μέγεθός τους. Όμως, οφείλουν να έχουν τις πλευρές τους ανάλογες και τις γωνίες που σχηματίζονται από ομόλογες πλευρές τους ίσες μία προς μία.

Αν παρατηρήσουμε την ακόλουθη σειρά γεωμετρικών σχημάτων:

συμπεραίνουμε ότι το καθένα από τα συνιστώντα μέρη είναι αντίγραφο του μεγαλύτερου. Μπορούμε να λέμε ότι τα παραπάνω σχήματα χαρακτηρίζονται από την ιδιότητα της αυτοομοιότητας. Όλα τα μέρη του είναι όμοια μεταξύ τους καθώς και με το αρχικό. Τα παραδείγματα δείχνουν ότι η έννοια που μας ενδιαφέρει εμφανίζεται σε γεωμετρικά σχήματα τα οποία δεν είναι φράκταλ.

Η αυτοομο ιότητα στη ψίJση .

Τρία χαρακτηριστικά παραδείγματα φυσικών αντικειμένων που εκδηλώνεται η αυτοομοιότητα είναι το κουνουπίδι . η φτέρη και οι ακτογραμμές.

α. Η φτέρη ανήκει στην κατηγορία των φυτών που εκδηλώνουν την ιδιότητα της αυτοομοιότητας με τον καλύτερο τρόπο. Μια φτέρη αποτελείται από φύλλα καθένα από τα οποία αποτελείται από πολλά μικρότερα. Και αυτά ακόμα τα μικρά φύλλα αποτελούνται από ακόμα μικρότερα που διατηρούν την ίδια δομή με τη φτέρη β. Αν από ένα κουνουπίδι αποσπάσουμε ένα κομμάτι θα διαπιστώσουμε ότι αυτό θα μοιάζει με το αρχικό, θα είναι ένα μικρότερο αντίγραφο. Αν από το πρώτο αποσπάσουμε ένα κομμάτι θα διαπιστώσουμε ότι είναι ακόμα μικρότερο αλλά εξακολουθεί να μοιάζει με το αρχικό . γ. Ας παρατηρήσουμε χάρτες που περιγράφουν ακτογραμμές σε διαφορετικές κλίμακες όπως αυτή που ακολουθεί (παρμένη από το βιβλίο Fractals for the classroom) : Αυτό που μας αποκαλύπτεται είναι μια όμοια κατανομή κόλπων και ακρωτηρίων. Μπορούμε να θεωρήσουμε ότι μια ακτογραμμή παρουσιάζει ·· φράκταλ δομή με την έννοια ότι αν μεγεθύνεται εμφανίζονται νέοι κόλποι και --:i;::JII8!111PIC ακρωτήρια και παρόλα αυτά εξακολουθεί να μοιάζει με ακτογραμμή

Η αυτοομοιότητα στα μαθη ματικά φράκταλ.

Η στοιχειώδης μοντελοποίηση μιας φτέρης μπορεί να πραγματοποιηθεί με τη χρήση ενός υπολογιστικού περιβάλλοντος. Ένα μικρό πρόγραμμα που περιλαμβάνει πολλαπλές αναδρομικές κλήσεις, συνήθως, είναι αρκετό για να μοντελοποιήσουμε ορισμένα ενδιαφέροντα μαθηματικά φράκταλ όπως για παράδειγμα η φτέρη , το τρίγωνο του Sierpinski, τα φράκταλ δέντρα και η χιονονιφάδα του Koch. Η μοντελοποίηση στον υπολογιστή τέτοιων φράκταλ θα μας βοηθήσει να πειραματιστούμε με αντικείμενα που δεν ανήκουν στην περιοχή μελέτης της Ευκλείδειας Γεωμετρίας.

3" Θέμα

Π ρολεγόμενα : Μόλις είχε κλείσει η ύλη της στήλης μας (για το τεύχος 64), όταν, ο εκ των επιμελητών της στήλης Γιάννης Κερασαρίδης, έλαβε στο ηλεκτρονικό του ταχυδρομείο ένα σύντομο σημείωμα του Νίκου Λυγερού με τίτλο «0 Ξεχασμένος Δίσκος». Στο σημείο αυτό πρέπει να πούμε πως ο Νίκος Λυγερός είναι μαθηματικός καθηγητής στο πανεπιστήμιο της Λυών, επισκέπτης καθηγητής σε μια σειρά πανεπιστήμια, λογοτέχνης και εικαστικός. Είναι επιστημονικός σύμβουλος του «Συνδέσμου Φίλων Κ. Καραθεοδωρή». Με τη δική του συμβολή ήρθαν στο φως της δημοσιότητας αρκετές σπάνιες εργασίες

ΕΥΚΛΕΙΔΗΣ Β' 65 τ.l/26

Page 29: Ευκλειδης Β 65

------------------------------ #,Af, Af�T#EAfAr/CVf ----------------------------­

του διάσημου έλληνα μαθηματικού Κώστα Καραθεοδωρή . Πολύ σύντομα η στήλη μας θα έχει αφιέρωμα στην προσωπικότητα και το έργο του Κ. Καραθεοδωρή .

Το σημείωμα του Ν. Λυγερού αναφέρεται στον Κ. Καραθεοδωρή και γράφτηκε για να θυμηθούμε το <<'Έτος Leonard Euler» (300 χρόνια από τη γέννησή του). Απολαύστε το ιδιότυπο λογοτεχνικό ύφος του Ν. Λυγερού.

«0 ξi:zασμi:νος δίσκος». '\ ί κος Λυγερός, πανεπιστήμιο Λυών Όταν μελετούσε τα Άπαντα του Euler, άκουγε

τον αγαπημένο του δίσκο. Έτσι ένιωθε ότι τα lίeder του δυστυχισμένου συνθέτη τον βοηθούσαν να κατανοήσει τις επινοήσεις του δασκάλου. Η γυναικεία φωνή που συνόδευε τόσο τρυφερά το πιάνο της ανάγκης τον γέμιζε με το παράξενο αίσθημα της χαράς. Μία περίεργη χαρά που ζούσε από τότε που ήξερε πως θα πέθαινε μόνο μετά την έκδοση του βιβλίου του Ελβετού μαθηματικού . Είχε δαπανήσει νύχτες ολόκληρες για ν' αγγίξει την ουσία του έργου του. Ήξερε πόσα του χρωστούσε για την ανάπτυξη των θεωριών του στο Λογισμό Μεταβολών. Κανείς δεν μπορούσε να δει τα κρυφά του σχόλια. Δεν ήθελε να πεθάνει η μνήμη του δασκάλου δίχως να δώσει μία τελευταία μάχη. Έπρεπε να τη σώσει από τη λήθη όπως ο δίσκος είχε σώσει την αθάνατη φωνή που έκλαιγε δίχως δάκρυα. Κάθε κλεμμένη νότα από τη λήθη ήταν μία μάχη που κέρδισε ο συνθέτης. Εξέταζε τα κείμενα του δασκάλου κι ένιωθε την υπευθυνότητά του. Αν πέθαινε πριν το τέλος της εργασίας του, ποιος θα μάθαινε για τα θεωρήματα της ομορφιάς; Άκουγε λοιπόν κάθε στιγμή της αθανασίας για να δώσει κουράγιο στον εαυτό του.

Όλοι έλεγαν ότι είχε γεράσει και δεν μπορούσε πια να προσφέρει στα μαθηματικά, ακόμα και οι φίλοι του. Όμως εκείνος ζούσε την αλήθεια. Κι έπρεπε ν' αφήσει ένα έργο για τους επόμενους, που δεν είχε γράψει ακόμα. Εκείνοι οι άνθρωποι που δεν είχαν γεννηθεί ακόμα, εκείνοι οι αρχαιολόγοι του μέλλοντος έπρεπε να παραλάβουν τη μνήμη

l l α . "Λ ιπδ το ξ{ρατε,, " I η υπάντηση !

του και το έργο του δασκάλου. Μόνο έτσι θα μπορούσαν να το διασώσουν και να το διαδώσουν. Ήλπιζε ότι στο μέλλον θα υπήρχε ένα μουσείο, ένα ίδρυμα ή ένα ινστιτούτο, δεν μπορούσε να το καθορίσει εκείνη τη στιγμή , που θα αναλάμβανε να μη σβήσει η μνήμη ενός άλλου ανύπαρκτου λαού. Προς το παρόν όμως πάλευε με τους τυπογράφους για να εξασφαλίσει την αυθεντικότητα του έργου . Πρόσεχε κάθε σημείο σαν να ήταν η νότα μιας αόρατης παρτιτούρας που δεν έπρεπε να χαθεί. Αλλιώς το έργο της ζωής του δεν θα είχε πια το ίδιο νόημα.

Άκουγε την ανθρωπιά της γυναικείας φωνής και έβλεπε μέσα της ολόκληρη την ανθρωπότητα. Μόνο που εκείνη τη στιγμή για όλο το έργο του δασκάλου, η ανθρωπότητα δεν ήταν παρά μόνο ένας άνθρωπος. Και αυτός ο άνθρωπος ήταν ο εαυτός του. Ένας μαθηματικός που άργησε ν' αρχίσει τη σταδιοδρομία του. Δεν είχε αντιληφθεί τη βαρύτητα της ζωής του παρά μόνο όταν αντίκρισε τις πυραμίδες. Εκεί τον είχε αγγίξει η ομορφιά της λιτότητας. Και από τότε είχε αποφασίσει ότι δεν θα σταματούσε τον αγώνα του για τα μαθηματικά. Κι ενώ οι άλλοι θεωρούσαν ότι ήταν ήδη αργά, εκείνος απέδειξε ότι ήταν πολύ νωρίς. Το πιάνο δεν έπαψε ούτε μία στιγμή να συνοδεύει τη φωνή . Έτσι κι εκείνος δεν θα άφηνε το έργο του δασκάλου παρά μόνο μετά την έκδοσή του ακόμα και αν πέθαινε μετά, για να ζήσει μέσα στον ανύπαρκτο λαό.

Είναι φανερό πως κάτι τέτοιο μόνο η φύση θα το μπορούσε. Το 1 985 , ο H.W. Κroto (Μ. Βρετανία) και οι J. Heath, S. O 'Brien, R. Curl, R Smalley (ΗΠΑ) ανακάλυψαν πως όταν εξαερώνεται ο γραφίτης υπό την επίδραση μιας δέσμης λέιζερ μέσα σε ρεύμα ηλίου δημιουργούνται μόρια, αρκετά σταθερά, που αποτελούνται από ένα μεγάλο (32-90) και πάντοτε άρτιο αριθμό ατόμων άνθρακα (κοίτα διπλανό σχήμα)

Το σταθερότερο ήταν το C60, και οι μπάλα του Bucky, προς τιμήν του Buckminster επιστήμονες που το ανακάλυψαν θεώρησαν ότι είχε Fuller. Το όνομα αυτό εξελίχθηκε γρήγορα στο τη μορφή μιας μπάλας ποδοσφαίρου. Οι ερευνητές επισημότερο μπακμινστερφουλερένιο [πηγή : ονόμασαν το μόριο μπάκυμπωλ (buckyball), δηλ, QUANTUM, τόμ. l , τεύχ.2] Π ρος τους φ ίλους της ση)λης : Για μια ακόμη φορά η στήλη είναι υποχρεωμένη να υπενθυμίσει α) δεν είναι δυνατό να δημοσιεύονται όλες οι εργασίες (ο αριθμός τους είναι μεγάλος), β) οι εργασίες σας να είναι περιορισμένης έκτασης και περιεκτικές, αφού και ο χώρος μας είναι περιορισμένος.

ΕΥΚΛΕΙΔΗΣ Β' 65 τ.l/27

Page 30: Ευκλειδης Β 65

�ωCDqJfP0UUm& l}J0(1J fJf[JfJJ !iJ() u(/)�f[J ΊJ(f)fJJ ιtflrJJωe5tJ(!)tJ

Βαγγέλης Ευσταθίου - Θάνος Χαραλάμπους

Οι ασκήσεις που ακολουθούν βασίζονται στις ταυτότητες που αναφέρονται στο σχολικό βιβλίο και στις

συνέπειες αυτών που στη συνέχεια παραθέτουμε.

1 . Από τις γνωστές ταυτότητες προκύπτουν : i) α2 + β2 = ( α + β )2 - 2αβ ii) α2 + β2 = ( α - β )2 + 2αβ iii) ( α + β )2 - ( α - β )2 = 4αβ ίν) ( α + β )2 + ( α - β )2 = 2 ( α2 + β2 ) ν) ( α + β )3 = α3 + β3 + 3αβ( α + β) νί) ( α - β )3 = α3 - β3 - 3αβ(α - β) νίί) α3 + β3 = ( α + β )3 - 3αβ ( α + β ) viii) α3 - β3 = ( α - β )3 + 3αβ ( α - β )

Άμεση συνέπεια των ταυτοτήτων (i), (ii), (iii), (ίν) εί ναι οι παρακάτω ανισότητες: α2 + β2 � -2αβ , α2 + β2 � 2αβ , (α + β)2 � 4αβ , 2 ( α2 + β2 ) � ( α + β )2 , όπου το ίσον για μεν την πρώτη ισχύει μόνο όταν α=-β για δε τις υπόλοιπες μόνο όταν α=β.

i) (-α + β γ = (β - α)ν , για κάθε ν ε Ν* i i) (-α - β γ = (α + β) ν , για κάθε άρτιο φυ­

σικό ν :;t: Ο iii) (-α - β γ = - (α + β γ , για κάθε περιττό

φυσικό ν 2. Να αποδειχτούν οι ταυτότητες :

i) ( χ - α ) ( χ - β ) = χ2 - ( α + β ) χ + α β ii) ( χ - α ) ( χ - β ) ( χ - γ ) =

= χ3 - ( α + β + γ ) χ2 + ( α β + βγ + γα ) χ - αβγ

i) ( χ - α ) ( χ - β ) = χ2 - αχ - βχ + αβ = = χ 2 - ( α + β ) χ + α β

ii) ( χ - α ) ( χ - β ) ( χ - γ ) = = [ χ 2 - ( α + β ) χ + α β ] ( χ - γ ) =

χ 3 - yx 2 - ( α + β ) χ 2 + γ ( α + β ) χ + αβχ - αβγ = χ 3 - ( α + β + γ ) χ 2 + ( α β + βγ + γα ) χ - αβγ .

από την (i) με α=β=-y παίρ­νουμε (x+y)2 =x2 +2xy+y και από την (ii) με α=β=γ=-y παίρνουμε : ( χ +y)3 = χ3 +3x2y+3xi +1 . 3. Να αποδείξετε ότι ισχύει :

{α + β + γγ = α2 + β2 + γ2 + 2αβ + 2αγ + 2βγ .

(α + β + γγ = [(α + β) + rΤ = (α + β )2 + γ2 + 2γ (α + β) = α2 + β2 + γ2 + 2αβ + 2αγ + 2βγ Ομοίως: (α + β + Υ + δ)2 = α2 + β2 + γ2 + δ2 + 2αβ + +2αγ + 2αδ + 2βγ + 2βδ + 2γδ και (α - β + γ - δ)2 = α2 + β2 + γ2 + δ2 - 2αβ +

+2αγ - 2αδ - 2βγ + 2βδ - 2γδ . Προκειμένου να εξασκηθούν οι μαθητές σε

μετασχηματισμούς ή παραγοντοποιήσεις παρα­στάσεων, τις περισσότερες από τις ταυτότητες που ακολουθούν θα τις αποδεί ξουμε ξεκινώντας από το 1 ο μέλος (Α) και καταλήγοντας στο 2° μέλος (Β), παρόλο ότι μερικές από αυτές αποδεικνύονται ευ­κολότερα με εκτέλεση των πράξεων και στα δύο μέλη ή ακολουθώντας αντί στροφη πορεία από το Β στο Α. 4. Να αποδείξετε ότι ισχύει:

ΕΥΚΛΕΙΔΗΣ Β' 65 τ. l/28

Page 31: Ευκλειδης Β 65

Μαθηματικά για την Α ' Λυκείου

α) (α + β + γ )3 = α3 + β3 + γ3 + 3α2 { Ρ + γ) + + 3β2 (γ + α) + 3γ2 (α + Ρ) + 6αβγ

β) { α+β+γ )3 =α3 +β3 +γ3 +3 { α+β ) { β+γ ) { γ+α)

Α = ( α + β + γ )2 ( α + β + γ ) = ( α2 + β2 + γ2 + 2αβ + 2αγ + 2βγ ) ( α + β + γ ) = . . . = α3 +β3+γ3 +3α2β+3αβ2 +3γα2 + +3γ2α+3γβ2 +3βγ2 +6αβγ=

= α3 + β3 + γ3 + 3αz (β + γ) + 3βz (γ + α) + + 3γ2 (α + β) + 6αβγ = Β

Δείξαμε λοιπόν ότι: Α = α3 + β3 + γ3 + 3Κ , όπου Κ=α2β+αβ2 +γα2 +γ2α+γβ2 +βγ2 +2αβγ= α β (α + β ) + γ2 (α + β) + αγ2 + β 2γ + αβγ + αβγ = αβ (α + β) + γ2 (α + β) + αγ (α + β ) + βγ (α + β ) = = (α + β ) ( αβ + γ2 + αγ + βγ) = = (α + β ) [ α(β + γ) + γ (β + γ)] = = (α + β ) (β + γ) (γ + α) , οπότε Α = α3 + β3 + γ3 + 3 (α + β) (β + γ) (γ + α) = Β

( α + β + γ )3 = [ ( α + β ) + γ ]3 = ( α + β )3 + γ3 + +3 ( α + β ) γ [ ( α + β ) + γ ] =

= α3 + β3 + 3αβ ( α + β ) + γ3 + 3γ ( α + β ) ( α + β = α3 + β3 + γ3 + 3 ( α + β ) [ αβ + γ ( α + β + γ ) ] = = α3 + β3 + γ3 + 3 ( α + β ) ( αβ + αγ + βγ + γ2 ) = = α3 + β3 + γ3 + 3 ( α + β ) [ β ( γ + α ) + γ ( γ + α ) ] = α3 + β3 + γ3 + 3 ( α + β ) ( β + γ ) ( γ + α )

2 β2 2 s 3 β3 3 Αν S1 = α + β + γ, S2 = α + + γ , 3 = α + + γ τότε:

ί) s; = S2 + 2 (αβ + βγ + γα) ί ί) s: = 3S,S2 - 2S3 + 6αβγ ί ί ί) s: = S3 + 3S, ( αβ + βγ + γα) - 3αβγ

ί) Είναι άμεση συνέπεια της άσκησης 3 . ί ί) Από (4α) έχουμε

s: = S3 + 3α2 (S, - α) + 3β2 (S, - β) +

+ 3γ2 ( s, - γ) + 6αβγ = = S3 + 3S1 ( α

2 + β2 + γ2 ) - 3( α3 + β3 + γ3 ) + 6αβγ = S3 + 3S1S2 - 3S3 + 6αβγ = = 3S, S2 - 2S3 + 6αβγ . ί ί ί) Από ( 4β) έχουμε :

(2 ) s: = S3 + 3 (S, - α) (s, - β ) (s, - γ) = = S3 + 3s: - 3 (α + β + γ) s; + +3 ( αβ + βγ + γα ) S, - 3αβγ = = S3 + 3S: - 3s: + 3 ( αβ + βγ + γα)S, - 3αβγ = = S3 + 3S, ( αβ + βγ + γα) - 3αβγ

5. Να αποδειχτεί η ταυτότητα : α2 + β2 + γ2 - αβ - βγ - γα = = l[ ( α - β γ + ( β - γ γ + ( γ - α )2 J

Α = _!_( 2α2 + 2β2 + 2γ2 - 2αβ - 2βγ - 2γα ) = 2 =_!_( α2 + α2 + β2 + β2 + γ2 + γ2 - 2αβ - 2βγ - 2γα ) = 2 = }[ ( α - β / + ( β - γ )2 + ( γ - α )2 J = Β Επειδή το 2° μέλος είναι μη αρνητικός αριθμός

προκύπτει η βασική ανισότητα: α2 + β2 + γ2 � αβ + βγ + γα .

6. Να αποδειχτεί η ταυτότητα: (Cauchy -Euler) α3 + β3 + γ3 - 3αβγ =

=( α + β + γ ) { α2 + β2 + γ2 - αβ - βγ - γα ) = γ L�( α + β + γ ) [ ( α - β )2 + ( β - γ )2 + ( γ - α γ J

Με βάση την ταυτότητα ( 1 νίί) έχουμε : Α = ( α + β )3 - 3αβ ( α + β ) + γ3 - 3αβγ =

= ( α + β )3 + γ3 - 3αβ ( α + β + γ ) = [ ( α + β ) + γ ]3 - 3 ( α + β ) γ [ ( α + β ) + γ ] ­-3αβ ( α + β + γ ) = = ( α + β + γ )3 - 3 ( α + β ) γ ( α + β + γ ) ­-3αβ ( α + β + γ ) = = ( α + β + γ ) [ ( α + β + γ )2 - 3 ( α + β ) γ - 3αβ J = = ( α + β + γ ) ( α2 + β2 + γ2 + 2αβ + 2βγ + 2γα - 3αγ - 3βγ - 3αβ ) = = ( α + β + γ ) ( α2 + β2 + γ2 - αβ - βγ - γα ) = Β Και σύμφωνα με την προηγούμενη άσκηση . Β = } ( α + β + γ ) [ ( α - β )2 + ( β - γ / + ( γ - α / J

Προφανώς η πορεία από το Β στο Α είναι α­πλούστατη .

ί) α3 + β3 + γ3 = 3αβγ <:::> α+β+γ = Ο ή α = β = γ ί ί) αν α + β + γ > Ο τότε α3 + β3 + γ3 � 3αβγ .

ΕΥΚΛΕΙΔΗΣ Β ' 65 τ.l/29

Page 32: Ευκλειδης Β 65

Μαθηματικά για την Α' Λυκείου

iii) s3 - 3αβγ = sl [ s2 - ( αβ + βγ + γα) J ' όπου sl = α + β + γ, s2 = α2 + β2 + γ2 , s3 = α3 + β3 + γ3

7. Να αποδειχτεί η ταυτότητα : ( α + β )3 - 3αβ ( α + β - ι ) - ι = = ( α + β - ι ) ( α2 + β2 - αβ + α + β + ι )

Απόδειξη ( ! ν )

Α = α3 + β3 + 3αβ( α+ β) - 3αβ( α+ β) + 3αβ - 1 = = α3 + β3 - 1 + 3αβ = α3 + β3 + ( -1 )3 - 3αβ( -1 ) = = ( α+β- 1 )[ α2 +β2 + ( -1 )2 -αβ-α( -1 ) -β( -1 ) ] = = ( α + β - 1 ) ( α2 + β2 + 1 - αβ + α + β ) = Β

8. Α ν α + β + γ = 6, α3 + β3 + γ3 = 36, αβγ = 6 , τότε να υπολογίσετε το άθροισμα

S = α2 + β2 + γ2 . Λ ύ ση Αν θέσουμε sl = α + β + γ = 6, s2 = s = α2 + β2 + γ2 = χ ' S3 = α3 + β3 + γ3 = 36 και αβ+βγ+γα=y, τότε s� = s2 + 2 ( αβ + βγ + γα) και από την συνέ­πεια (iii) της άσκησης 6 έχουμε : s3 - 3αβγ = sl [ s2 - ( αβ + βγ + γα) J ' δηλαδή

62 = χ + 2y } 36 - 3 · 6 = 6 ( χ - y) .

Τελικά , οπότε . x + 2y = 36} χ = 1 4} x - y = 3 y = 1 1

Β ' τρόπος ( α + β + γ )3 = α3 + β3 + γ3 + 3α2 ( β + γ ) + +3β2 ( γ + α ) + 3γ2 ( α + β ) + 6αβγ :::::> 63 =36+3α2 ( 6-α) +3β2 ( 6-β) + 3y ( 6-γ) +6 · 6=:> 63 = 36 + 1 8 ( α2 + β2 + γ2 ) - 3 ( α3 + β3 + γ3 ) + 36=:> 6 · 36=36+ 1&S-3 · 36+36:::::> 12=2+S-6+ 2=:>S= 14 Άμεση λύ ση βέβαια έχουμε με χρήση της συ­νέπειας (ii) της ταυτότητας (4) . Π ρ άγματ ι : Si = 3S1 S2 - 2S3 + 6αβγ :::::> :::::> 63 = 3 · 6 · S - 2 · 36 + 6 · 6 =:> S = 1 4 Τέλος μια ακόμη λύση μπορεί να δοθεί με βά­ση τις συνέπειες (i) και (iii) των ταυτοτήτων (3) και (4). Πράγματι: ( i ) =:> 62 = x + 2y και ( ii i ) =:> 63 = 36 + 3 · 6y - 3 · 6 =:> y = 1 1 , οπότε : ( i ) =:> x = 14 .

9. Να αποδείξετε ότι : "

) Γ ζ

' θ ' (σ, β) ' ι ια τα ευγη των αρι μων (χ, � εχουμε :

( α2 + β2 ) { χ2 + ψ2 ) _ ( αχ + β ψ )2 =

= ( αψ - βχ )2 = I � : 1 2 *

" " ) Γ ' δ (α, β, γ) ' ιι ια τις τρια ες εχουμε : (χ, ψ, ω)

( α2 + β2 + γ2 ) ( χ2 + � + ω2 ) - ( αχ+ βψ + γω)2 =

= ι : :ι2 + ι : : ι2 + ι : : ι2 Ανάλογη ταυτότητα ισχύει για δύο ν-αδες α­ριθμών (ταυτότητα Lagrange) Απόδειξη ί ) Α = α2χ2 + α2ψ2 + β2χ2 + β2ψ2 _

- ( α2χ 2 + β2ψ2 + 2αχβψ ) = α2χ2 + α2� + β2χ2 + β2� _ α2χ2 _ β2� _ 2αχβψ =

α2ψ2 + β2χ2 - 2αψβχ = ( αψ - βχ )2 = I : �� 2 = Β i i ) Α = α2χ2 + α2ψ2 + α2ω2 + βzχ2 + β2ψ2 +

+β2ω2 + γ2χ2 + γ2ψ2 + γ2ω2 _

( α2 χ2 + β2ψ2γ2ω2 + 2αχβψ + 2βψγω + 2γωαχ ) = α2ψ2 + α2ω2 + β2χ 2 + β2ω2 + γ2χ2 + γ2ψ2 _

-2αχβψ - 2βψγω - 2γωαχ = ( αψ - βχ )2 + ( αω - γχ )2 + ( βω - γψ / = ι : � � 2 + ι : : ι 2 + ι � : ι 2 = Β

Βασικές συνέπειες : Ανισότητες : Cauchy - Schwartz : i ) (α2 + β2 )(χ 2 + ψ2 ) � (αχ + βψ)2 i i ) (α2 + β2 + γ2 )(χ2 +� + ω2 ) � (αχ + βψ+ γω)2 Α ν αβγ =ι:. Ο τότε το ίσο στην (i) ισχύει μόνο

• χ ψ ( " " ) . . χ ψ ω οταν - = -β και στην η μονο οταν - = -β = - . α α γ ι ο. Να αποδειχτεί η ταυτότητα :

( α2 + β2 + γ2 γ - ( αβ + βγ + γα )2 = = ( α2 _ βγ )2 + ( β2 _ αγ γ + ( γ2 _ αβ )2

Απόδε ιξη Με εφαρμογή της ταυτότητας Lagrange για τις ' δ (α, β, γ) . τρεια ες εχουμε: (β, γ, α)

Α = ( α2 + βz + γ2 ) ( β2 + γ2 + α2 ) _ ( αβ + βγ + γα )2 =

• Γενικά με το σύμβολο �� �� παριστάνουμε τη διαφο­

ρά (αδ - βγ) και τη λέμε ορίζουσα 2ης τάξης με γραμμές (α,β) και (γ, δ) ή με στήλες (α, γ) και (β, δ) .

ΕΥΚΛΕΙΔΗΣ Β' 65 τ.l/30

Page 33: Ευκλειδης Β 65

Μαθηματικά για την Α ' Λυκείου

= ι ; � � 2 + ι ; : ι 2 + ι � : ι2 = ( αγ _ β2 )2 + ( α2 _ βγ )2 + ( αβ _ γ )2 = = ( α2 _ βγ )2 + ( β2 _ γα )2 + ( γ2 _ αβ )2 = Β

1 1 . Ν α αποδειχτεί ότι ισχύει : χ3 = ( χ(\+ 1) )2 - ( χ( χ 2- 1) )2 '

για κάθε χ ε JR (1 )

χ2 χ2 Β = 4(χ + 1)2 -4 (χ - 1)2 = χ2 χ2 4[ (χ + 1)2 - (χ - 1)2 ] = 4 . 4χ = χ3 = Α

fiJ �R ρ i[η u� μηιr'η ; Από την ( 1 ) για χ Ε z• προκύπτει ότι ο κύβος

κάθε ακεραίου αριθμού είναι διαφορά τετραγώνων δύο ακεραίων, αφού με χ Ε z• οι αριθμοί χ( χ + 1 )

και χ(χ - 1) είναι άρτιοι ακέραιοι αριθμοί (ως γι­νόμενα διαδοχικών ακεραίων). π.χ. 1 73 = ( 1 7� 1 8 )2 - ( 1 7�1 6 ) 2 = ( 1 7 · 9 )2 - ( 1 7 · 8 )2 =

= 1 532 - 1 362 Ταυτότητες που περιέχουν την ποσότητα

α + β + γ τ =

2 (Συνήθως για α,β,. γ πλευρές τρι-

γώνου)

12. Να αποδειχτεί η ταυτότητα : ( τ - α )3 + ( τ - β )3 + ( τ - γ )3 + 3αβγ = τ3 ,

α + β + γ αν τ = 2 . /'\πόδπξη Από την ταυτότητα (4β) έχουμε :

χ3 + y + ω3 = ( χ + y + ω)3 - 3( x + y)( y + ω)( ω+ χ )

Αν θέσουμε χ=τ - α, y= τ - β, ω= τ - γ, παίρ­νουμε: ( τ - α )3 + ( τ - β )3 + ( τ - γ )3 =

[ ( τ - α ) + ( τ - β ) + ( τ - γ ) ]3 --3[ ( τ-α) +( τ-β) ] [ ( τ-β) +( τ-γ) ] [ ( τ-γ) +( τ-α) ] = [ 3τ - ( α + β + γ ) ]3 - 3 [ 2τ - ( α + β ) ] [ 2τ - ( β + γ ) ] [ 2τ - ( γ + α ) ] = ( 3τ - 2τ )3 - 3γαβ = τ3 - 3αβγ. Αρα: ( τ - α )3 + ( τ - β )3 + ( τ - γ )3 + 3αβγ = τ3

13. Σε κάθε τρίγωνο ΑΒΓ να δείξετε ότι: τ ( τ - α ) = ( τ - β ) ( τ - γ ) � Α = 90° .

Λπ6δειξη α + β + γ β + γ - α Έχουμε ότι τ - α = - α = '-------'-----2 2 '

α + γ - β α + β - γ ομοίως τ - β = 2 , τ - γ = 2 . Οπότε: τ ( τ - α ) = ( τ - β ) ( τ - γ ) �

( α + � + γ

)(β + �- α

) = ( α+�-β

)( α+�-γ

) � � ( β + γ )2 - α2 = α2 - (β - γ)2 � � ( β + γ )2 + (β - γ)2 = 2α2 �

� 2β2 + 2γ2 = 2α2 � βz + γ2 = α2 � Α = 90ο Ταιπδτητες υπδ συνΟψα:ς

14. Α ν α + β + γ = Ο , να αποδειχτεί ότι ισχύουν οι σχέσεις : i) α2 + αγ = β2 + βγ

ii) α2 - βγ = l( α2 + β2 + γ2 )

, \πόδtιξη Η συνθήκη α+β+γ=Ο μπορεί ν επιλυθεί ως

προς ένα γράμμα π. χ. γ = -α - β . Σ' αυτή την πε-ρίπτωση η αποδεικτέα ανάγεται σε απλούστερη με δύο αγνώστους. Πράγματι:

i) α ' τρόπος Αφού γ = -α - β θα έχουμε: Α = α2 + α ( -α - β ) = α2 - α2 - αβ = -αβ και Β=β2 +β( -α-β) =β2 -αβ-β2 =-αβ. Άρα Α=Β β ' τρόπος α + β + γ = Ο :::::> α + β = -γ => ( α - β ) ( α + β ) = -γ ( α - β ) :::::> α2 - β2 = -αγ + βγ :::::> α2 + αγ = β2 + βγ i i ) Α. ' τρόπος Έχουμε : Α = α2 - β ( -α - β ) = α2 + αβ + β2 και Β=� [ α2 +β2 + (-α-β)2 ] =�[ α2 +β2 +(α+β)2 ] = =_!_(2α2 + 2αβ+ 2β2 ) = α2 + αβ + β2 , οπότε Α=Β. 2 Β ' τρόπος α + β + γ = Ο � α = -β - γ � α2 = ( β + γ )2 �

=> α2 = β2 + 2βγ + γ2 =>2α2 = α2 + β2 +τ + 2βγ :::::> 2α2 - 2βγ = α2 + βz + γ2 :::::> 2 ( αz _ βγ ) = αz + β2 + γ2

1 :::::> α2 _ βγ = - ( α2 + βz + γz ) 2 1 5. Άν ισχύει } +-β

1 +! = Ο (1 ), με αβγ 'Φ Ο τότε:

α γ i) Ν α αποδείξετε ότι : α β + βγ + γα = Ο

ΕΥΚΛΕΙΔΗΣ Β ' 65 τ.l/31

Page 34: Ευκλειδης Β 65

Μαθηματικά για την Α · Λυκείου

ii) Ν α υπολογίσετε την παράσταση :

Α = β + γ + γ + α+

α + β . α β γ

Απόδειξη Ϊ) Έχουμε : ( 1 ) ::::::> αβ + βγ + γα = Ο ::::::> αβ + βγ + γα = Ο αβγ i i ) Α 'τρόπος ( 1 ) ::::::> _.!_ + _!_ = _ _.!_ ::::::> α + β = -_!_ :;t: O :::::> α β γ αβ γ :::::> α + β :;t: Ο και γ = -�, οπότε: α + β

β - � α -� Α = α + β + α + β + α + β = α β αβ

α + β β2 α (α + βγ = + - = α (α + β ) β (α + β ) αβ

α3 + β3 - (α + β )3 { Ι ν) -3αβ (α + β) __ -,------'-----:-'---'---- - - -3 αβ (α + β ) - αβ (α + β) -Β 'τρόπος αβ + βγ + γα = Ο ::::::>

γ ( α + β ) = -α β ::::::> α + β = - α β γ γ2

Ομοίως κυκλικά αντικαθιστώντας , έχουμε : Α = _

βγ _

γα _

αβ α2 β2 γ2

= - ( βγ + γα + αβ ) = α3β3 + β\3 + γ3α3 α2 β2 γ2 α2β2γ2

(Χρησιμοποιούμε την Ταυτότητα Cauchy -Euler) = - 3 ( αβ ) ( βγ ) ( γα ) = -3 α2βzγ2

16. Ά ν αβγ = 1 και αβ + α + 1 :;t: Ο τότε να δει­χτεί ότι ορίζεται, και να υπολογιστεί η πα­ράσταση :

Α = α + β + γ

αβ + α + 1 βγ + β + 1 γα + γ + 1

Απόδειξη 1 αβγ = 1 ::::::> γ = - ::::::> βγ + β + 1 = α β

= _.!_ + β + 1 = 1 + αβ + α :;t: 0 α α Ι Ι α + Ι + αβ και γα + γ + Ι = - +- + Ι = :;t: Ο . β αβ αβ

Άρα ορίζεται η Α και

Ι Α = α + β + αβ = α + αβ + Ι = 1 αβ + α + Ι αβ + α + 1 αβ + α + Ι αβ + α + Ι

α αβ

17. Ά ν α + β + γ = Ο , και α :;t: β :;t: γ :;t: α τότε να δείξετε ότι:

1 1 1 -----:-- + + = ο . 2α2 + βγ 2β2 + γα 2γ2 + αβ Απόδειξη Στην περίπτωση αυτή η αντικατάσταση

γ = -α - β είναι ασύμφορη αφού οι πράξεις είναι πολλές.

Εργαζόμαστε λοιπόν ως εξής : α + β + γ = Ο ::::::> γ = -α - β ::::::> βγ = -αβ - β2 , Άρα: 2α2 + βγ = 2α2 - αβ - β2 = α2 - β2 + α2 - αβ = ( α + β ) ( α - β ) + α( α - β ) = ( α - β ) ( 2α + β ) = ( α - β ) ( α + α + β ) = ( α - β ) ( α - γ ) Παρατηρούμε ότι οι παρανομαστές των κλα-

σμάτων προκύπτουν δια κυκλικής εναλλαγής των γραμμάτων έτσι 2β2 + γα = ( β - γ ) ( β - α ) και 2γ2 + αβ = ( γ - α ) ( γ - β ) .

Άρα Ι Α + = ( α-β) ( α-γ) ( β -γ) ( α-β) ( β-γ) ( α-γ)

( β - γ ) - ( α - γ ) + ( α - β ) Ο ( α - β ) ( α - γ ) ( β - γ ) = ( α - β ) ( α - γ ) ( β - γ ) = Ο

18. Να αποδειχτούν οι ταυτότητες : i) α ( β - γ ) + β ( γ - α ) + γ ( α - β ) = Ο ii) α2 ( β - γ ) + β2 ( γ - α ) + γ2 ( α - β ) =

= - ( α - β ) ( β - γ ) ( γ - α ) iii) α3 ( β - γ ) + β3 (γ - α) + γ3 (α - β) =

= -(α - β)(β - γ)(γ - α) ( α + β + γ ) iv) α(β - γ)( τ - α )2 + β( γ - α) ( τ - β )2 +

+γ(α - β) ( τ - γ γ = Ο , αν τ = α + � + γ

Απόδειξη i) Α = α β - αγ + βγ - αβ + αγ - αβ = Ο = Β i i ) Α = α2β - αzγ + β2γ - αβ2 + γ2 (α - β) = αβ(α - β) - γ ( αz - βz ) + γz (α - β) = αβ( α - β) - γ( α - β)( α + β) + γ2 (α - β) = (α - β) [ αβ - γ( α + β) + γ2 ] = (α - β) [ αβ - αγ - βγ + γ2 ] =

ΕΥΚΛΕΙΔΗΣ Β' 65 τ. l/32

Page 35: Ευκλειδης Β 65

Μαθηματικά για την Α · Λυκείου

(α - β) [ α(β - γ) - γ(β - γ) ] = (α - β)(β -γ)(α - γ) = -(α - β)(β - γ)(γ - α) = Β i i i ) Α = α3β - α3γ + β3γ - αβ3 + γ3 ( α - β ) =

αβ ( α2 _ β2 ) _ γ ( αJ _ βJ ) + γJ ( α _ β ) = αβ{α-βΧα+β) -γ(α-β)( α2 +αβ+ β2 ) +y(α-β) = (α - β) ( αβ(α + β) - γ ( α2 + αβ + β2 ) + γ3 ] = (α - β) ( α2β + αβ2 - α2γ - αβγ - β2γ + γJ ] = (α - β) [ α2 (β - γ) + αβ(β - γ) - γ ( β2 - γ2 ) ] = (α - β)(β - γ) [ α2 + αβ - γ ( β + γ ) ] = (α - β)(β - γ) [ α2 + αβ - βγ - γ2 ] = (α - β )(β - γ) [ ( α - γ ) ( γ + α ) - β( γ - α) ] =

(α - β )(β - γ) [ -(γ - α) ( γ + α ) - β( γ - α) ] = -(α - β)(β - γ)(γ - α) ( α + β + γ ) = Β iv) Για την απόδειξη της πρότασης αυτής θα

χρησιμοποιήσουμε τις ταυτότητες (i), (ii), (iii) που προηγήθηκαν

Έχουμε: Α = α ( β - γ ) ( τ2 - 2τα + α2 ) +

+β( γ - α) ( τ2 - 2τβ + β2 ) + γ(α - β)( τ2 - 2τγ + γ2 ) = [ α ( β - γ ) + β ( γ - α ) + γ ( α - β ) ] τ2 -

-2τ [ α2 ( β - γ ) + β2 ( γ - α ) + γ2 ( α - β ) ] + +αJ ( β - γ ) + βJ ( γ - α ) + γJ ( α - β ) = = Ο · τ2 + 2τ ( α - β ) ( β - γ ) ( γ - α ) -

- ( α - β ) ( β - γ ) ( γ - α ) ( α + β + γ ) = Ο = Β

Ισότητες και ανισότητες , , ,

απο γεω μετρικη σκοπια Γιάννης Στρατήγης

Γνωρίζουμε γενικά από την καθημερινότητα, ότι αν συγκρίνουμε δυο ομοειδή μεγέθη θα είναι ίσα ή ά­νισα. Στα Μαθηματικά υπάρχουν ανισοϊσότητες που ξέρουμε να αποδεικνύουμε αλγεβρικά. Παρακάτω θα δώσουμε για μερικές από αυτές, γεωμετρικές αποδείξεις χρησιμοποιώντας εμβαδά ή μήκη πλευρών, όταν πρόκειται για θετικούς αριθμούς.

1 1 1 \ • • ' 2+β2>2 β · Λ ν ι σ ο ισοτητα : α _ α Έστω δυο τετράγωνα ΑΒΓ Δ, ΒΕΖΗ με πλευ­

ρές α, β αντίστοιχα ( α=::β>Ο). Στην ΑΒ σημειώνου­με σημείο Μ ώστε ΑΜ=β και φέρνουμε ΜΚlΑΒ.

Δ Κ Γ

α

Α β Μ α-β Β

β

β Ε

Α ν η προέκταση της ΖΗ τέμνει το ΜΚ στο Λ είναι προφανές ότι για τα εμβαδά των τετραγώνων ΑΒΓ Δ, ΒΕΖΗ και των ορθογωνίων ΑΜΚΔ, ΕΖΑΜ ισχύει: ΕΑΒrΔ+ΕΒΕzΗ=ΕΑΜκΔ+ΕεzΛΜ+ΕΚΛΗr2:ΕΑΜΚΔ+ ΕεzΛΜ, οπότε: α2+β22:αβ+[( α-β)+β)β . Άρα: α2+β22:αβ+αβ=2αβ

Η ισότητα αληθεύει μόνο όταν το γραμμοσκι­ασμένο τετράγωνο είναι ανύπαρκτο (πλευρά ίση με μηδέν), δηλαδή μόνο όταν α=β.

Αν δε α<β εναλλάσσουμε τη θέση των δυο αρ­χικών τετραγώνων.

2η ανισοϊσότητα : (α+β)22:4αβ Υποθέτουμε ότι α=::β>Ο και κατασκευάζουμε

το τετράγωνο ΑΒΓ Δ με πλευρά α+ β. Στις πλευρές ΑΒ, ΒΓ, ΓΔ, ΔΑ παίρνουμε τα σημεία Ε, Η, Μ, Κ αντίστοιχα ώστε ΑΕ=β=ΒΗ=ΓΜ=ΔΚ και φέρνου­με από αυτά καθέτους στις πλευρές του ΑΒΓΔ. Το τετράπλευρο ΖΘΛΙ που σχηματίζεται έχει πλευρά α-β, καθόσον ΖΘ=ΖΗ-ΘΗ=ΕΒ-ΜΓ=

=(ΑΒ-ΑΕ)-ΜΓ=( α+β)-β-β=α-β. Δ α Μ β Γ

β I I I

K r- - - - -IDΙ Λ α

- - - - - Η α

Ζ : α-β Θ I I

b Γl β

Α β Ε α Β Έχουμε τότε: ΕΑΒrΔ2:ΕΑεικ+ΕΚΛΜΔ+ΕΜrΗe+ΕΗzΕΒ, συνεπώς ( α+β)22:αβ+αβ+αβ+αβ, οπότε : ( α+β)22:4αβ και η ισότητα ισχύει μόνον όταν α= β.

Ι 3 '1 Ανισοϊσότητα : χ + - � 2 , χ > Ο

χ

Θεωρούμε ορθογώνιο τρίγωνο ΑΒΓ με μήκη καθέτων πλευρών ΑΓ=2 και ΑΒ = l x - � � - Τότε

ΕΥΚΛΕΙΔΗΣ Β' 65 τ.l/33

Page 36: Ευκλειδης Β 65

Μαθηματικά για την Α · Λυκείου

από το πυθαγόρειο θεώρημα το μήκος της υποτεί­νουσας θα είναι

(ΒΓ) = �(ΑΓ)2 + (ΑΒγ = 22 + lx -� � 2 = � �4+ - � )' � ( 2 1 1 ) 4 + χ + χ 2 - 2χ; =

= χ2 + J,_+ 2χ_!_ = ΓΙ?" = χ + _!_ , χ>Ο. χ - χ ν lχ τ ;J χ Β

Γ 1 Προφανώς ( ΓΒ) ;::: ( ΓΑ) , δηλαδή χ + - ;::: 2, χ

που η ισότητα αληθεύει μόνον όταν (ΑΒ)=Ο δηλα-δή χ = _!_ ισοδύναμα χ= 1 , επειδή χ>Ο. χ

Π αρατή ρηση Α. Από το σχήμα της 1 ης ανισοϊσότητας προ­

κύπτει η ταυτότητα: α2+β2=2αβ+(α-β)2, ενώ από της 2ης η : (α+β)2=4αβ+(α-β)2

Β. Από την 1 η ανισοϊσότητα με πρόσθεση στα . 2+β2 . μέλη της του παραγοντα α προκυπτει 2( α2+β2):-Ξ:α2+β2+ 2αβ οπότε 2( α2+β2)Ξ':( α+β)2 και σε συνδυασμό με τη 2η δίνει τη διπλή ανισοϊσότητα: 2( α2+β2)Ξ':( α+β)2Ξ':4αβ.

Ε ΦΑ Ρ Μ Ο Γ Εl: 1 . Αποδείξτε ότι: �--

i) l l x i - IY I I � �χ2 + Υ2 � l x l + I Y I , x, y ε 1R ·

ii) .J20022 + 1 < 2003. Λύση

ί) Για την l l x i - I Y I I � �χ 2 + y2 Αρκεί να δείξουμε ότι: l l x i - I Y I I 2 � �χ 2 + y2 2 , ή l x l 2 - 2 l x i · I Y I + I YI 2 � χ2 + y2 , ή -2 lxyΙ � o , που είναι αληθής. Ομοίως για την �χ2 + y2 � l x l + I YI αρκεί να δείξουμε ότι: �χ2 +Υ 2 � ( 1� +1� )2 , ή x2 +Y �I�2 +21+ 1� +1�2 ' ή 21� ;:::ο που ισχύει. Οι ισότητες επαληθεύο­νται μόνο όταν χ=Ο ή y=O. ί ί) Από το δεύτερο τμήμα της σχέσης του προηγούμενου ερωτήματος με χ=2002 και y= 1 έχουμε .J20022 + 1 2 < 1 2002 1 + 1 1 1 δηλαδή .J20022 + 1 2 < 2003 , αφού στην περίπτωσή

μας είναι χ, y:1:0. ι ι i+x+-� 2 . Για κάθε θετικό αριθμό χ ισχύει: � >2 i�

Λ ί>ση Έχουμε 3 1 1 χ + χ +- +­χ χ3 ( 1 )3 1 ( 1 ) 1 χ +� -3χ · � χ +� + χ +�

= 2 1 χ +­χ2 2 1 χ +­χ2

� (χ + �χχ ' + 2χ� + (�)' - 2ι χ +� � 2 2 1 χ χ + -χ2

Η ισότητα επαληθεύεται μόνο όταν x= l . 3. Αν οι πραγματικοί αριθμοί γ, δ είναι aντί­

στροφοι των θετικών αριθμών α, β αντί­στοιχα, να προσδιορίσετε το ελάχιστο των παραστάσεων: i) Α= α2+β2+γ2+δ2 ii) Β= αβ+αδ+βγ+γδ iii) Γ=(αβ+γδ)(αδ+βγ)

' ' 1 1 Λυση Ισχυει γ = - και δ = - ( 1 ) α β

ο Είναι Α�α' +β' +r' +δ' �α' +β' +ω' +(i )' � = ( α2 + �2 ) + (β2 + β

\ ) ;::: 2 + 2 = 4 .

{ Ι ) i i ) Έχουμε Β = αβ + αδ + βγ + γδ = αβ + αt + β± +± ·t = [ αβ+ �)+[* +�) ;::: 2+2 =4 .

Η ισότητα επιτυγχάνεται μόνον όταν α=β= 1 , οπότε το ελάχιστο της παράστασης Α είναι 4.

( Ι ) i i i) Είναι Γ = ( αβ + γ δ) ( αδ + βγ) =

= [ αβ + ± . t ) [α · t + β ·±) ;::: 2 · 2 = 4

Προφανώς οι ισότητες και στα Β, Γ αληθεύουν μόνον όταν α=β= 1 , οπότε έχουμε ελάχιστο 4. 4. Αν α, β, γ είναι θετικοί αριθμοί δείξατε ότι:

i) α + β + γ � 3�αβγ

ii) (1 +�)(ι + *)[ι +�) � 64, με α+β+γ=ι •

ΕΥΚΛΕΙΔΗΣ Β ' 65 τ. l /34

Page 37: Ευκλειδης Β 65

Μαθηματικά για την Α ' Λυκείου

Λύση i ) Αν χ,ψ,ω>Ο το δεύτερο μέλος της ταυτότητας

Cauchy - Euler χ3 + ψ3 + ω3 - 3χψω = = �(χ + ψ + ω) [(χ - ψ)2 + (ψ - ω)2 + (ω - χ )2 ] είναι μη αρνητικό οπότε χ3+ψ3+ω32':3χψω ( 1 ) . Η ισότητα μόνον όταν χ=ψ=ω. Αν αντικαταστήσουμε τις τιμές των χ, ψ, ω με

�. {/β, <f(, αντιστοίχως έχουμε: �3 + {./β3 + <Ff3 2 3� · w . <ΙΎ δηλαδή

α + β + γ 2 3{Γαβ1 (2) ίί) Ισχύει

( 1 + � ) ( 1 + i )( 1 + �) = ( 1 + i + � + :β ) ( 1 + �) = = 1 + _!_ + _!_ + _.!_ + (-1 + _1 + _1 ) + _1_ = γ β α βγ αγ αβ αβγ

= 1 + _.!_+ _!_ + _!_ + α + β + γ + -1- = α β γ αβγ αβγ 1 1 1 2 ' = 1 + - + - + - +- (3) , διοτι α+β+γ= 1 . α β γ αβγ

Από τη σχέση (2) επειδή α+β+γ= 1 , προκύπτει: 1 2 3{Γαβγ" οπότε -1- 2 33 = 27 (4) αβγ

1 1 1 �(4) Εξάλλου έχουμε - + - + - 2 33 - 2 3 · 3 = 9 α β γ αβγ Οπότε η (3) δίνει

( 1 + � ) ( 1 + i )( 1 + �) 2 1 + 9 + 2 . 27 = 64

Η ' ' ' ' β 1 ισοτητα ισχυει μονον οταν α = = γ = 3 . 5. Αν για τους μη αρνητικούς αριθμούς α, β εί­

ναι α+β=l τότε: i) α, β ε [Ο, ι ] ii) Ο�4αβ�1

iii) 1 � -α- + _β_ � 4 αz + β βz + α

Λύση i ) Από την υπόθεση έχουμε α= 1-β και β= 1-α,

αλλά α2':0 και β2':0. Επομένως 1-β2':0 και 1-α2':0 δηλαδή 12':β και 12':α. Έτσι Ο�α� 1 και Ο�β�1 .

i i ) Ισχύει: α+β= 1 ::::::>( α+β)2= 1 2= 1 , οπότε αρκεί να αποδείξουμε ότι: Ο�αβ::;1 =( α+β)2 που αλη­θεύει από τη 2η ανισοϊσότητα, καθόσον α, β2':0. Η ισότητα προς τα αριστερά ισχύει μόνον αν (α, β)=(Ο, 1 ) ή ( Ι , Ο) και προς τα δεξιά μόνο αν (α, β)=( 1 /2, 1 /2).

2 Υ π . 2 ( 1 )2 3 i i i ) Έχουμε α + β = α - α + 1 = α -2 + 4 > 0

2 2 ( 1 )2 3 Και β + α = β - β + 1 = β - 2 + 4 > Ο , δη-λαδή α2 + β > Ο και β2 + α > Ο .

Αν Α η παράσταση � + � θα έχουμε : α + β β + α α (β2 + α) +β (α2 + β) (α2 + β2 ) + αβ (β + α) Α - -- (α2 + β) (β2 + α) (α3 + β3 ) + α2β2 + αβ -

= [(α + β )2 - 2αβ ] + αβ (α + β) α+�= Ι

(α + β) {α2 - αβ + β2 ) + α2β2 + αβ -

12 - 2αβ + αβ · 1 _

1 - αβ _ (α2 - αβ + β2 ) + α2β2 + αβ - [(α + β)2 - 2αβ J -

= 1 - αβ = 1 - α β = 1 ( Ι ) 1 - 2αβ + α2β2 ( 1 - αβ )2 1 - αβ

Αλλά από την ii) αβ :::; _!_ < 1 ::::::> 1 - αβ > Ο (2) 4 Α ' λ ' δ θ ' 1 1 4 ' ρκει οιπον να ει χ ει: :::; -- :::; - , η 1 - αβ 3

3 1 2 1 - αβ 2 - , ή 4 Ο 2 -αβ 2 _ _!_ , ή τέλος 4 Ο :::; 4αβ :::; 1 , που ισχύει από το ii) ερώτημα Π ΡΟΤ Ε Ι ΝΟ ΜΕΝΑ ΘΕΜΑΤΑ

1 . Χαρακτηρίσατε με Σωστό (Σ) ή Λάθος (Λ) τις ακόλουθες προτάσεις :

i) Για κάθε α, βε (Ο, 1 ), ισχύει Ο < � < 1 β ίί) Για κάθε α, βεΙR, ισχύει

( Ι α Ι + Ι β Ι )2::;(α+β)2+(α-β)2 , . . . ) Γ , θ β Ο , 2 1 1 ιn ια κα ε α, > ισχυει -- :::; - + -α + β α β

Απ. ί) Σ ί ί ) Σ i i i) Λ. 2 . Αποδείξατε ότι: i) αβ+βγ+γα::; α2+β2+γ2

(Αλγεβρικά και με γεωμετρική ερμηνεία) ii) αβ+βγ+γα::; ( α +�+ γ Ι :=ς α2 + β2 + γ2 , iii) ( α+β-2γ/+(β+γ-2α)2+(γ+α-2β)22':

2': ( α-β)2+(β-γ )2+( γ-α )2 3. Να συγκρίνετε τους αριθμούς Α, Β στις παρα­

κάτω περιπτώσεις : i) Α=χ2, Β=χ, αν χ> 1 ii) Α= I x2- 1 l , Β=χ2+ Ι iii) A = �2 - .J3 + �2 + .J3, Β = � 2

2 iν) Α = �008 , Β = 1 Απ. ί ) > i i)::; i ii)<, iv)�

( χ - 1) + α2

ΕΥΚΛΕΙΔΗΣ Β ' 65 τ.l/35

Page 38: Ευκλειδης Β 65

Κυριάκου Κ. Καμπούκου - Γιώργου Σ. Τασσόπουλου

Για να διευκολύνουμε τους μαθητές της Α 'Λυκείου στο να αντιμετωπίζουν με ενιαίο τρόπο μη τετριμμένα προβλή­ματα υπολογισμού ευθυγράμμων τμημάτων, τόξων ή γωνιών δίνουμε τρείς βασικές προτάσεις και τις υποπεριπτώσεις τους, στις οποίες ανάγονται τα περισσότερα από τα προβλήματα αυτά ή αντιμετωπίζονται με αυτές ευκολότερα.

1 . Επί ημιευθείας ΣΧ θεωρούμε τα διαφορετι­κά σημεία Α, Β και σημείο Μ εσωτερικό του ΑΒ τέτοιο ώστε ΑΜ=λΑΒ, όπου φυσικά Ο<λ<1. Να δείξετε ότι:

ΣΜ=λΣΒ+(1-λ)ΣΑ (1)

Θεωρούμε αρχικά τη διάταξη Σ, Α, Μ, Β . Σ Α Μ Β •· - - 1 • - 1 · - · ·

χ

Εκφράζουμε το ΣΜ ως συνάρτηση των ΣΒ, ΣΑ όπως ακριβώς ζητείται στη σχέση ( 1 ) ,

οπότε έχουμε: ΣΜ=ΣΒ-ΜΒ } ΣΜ=ΣΑ+ΜΑ

Από τη μορφή της αποδεικτέας φαίνεται ότι το ΣΒ πρέπει να πολλαπλασιαστεί επί λ και το ΣΑ επί ( 1 -λ) . Έτσι έχουμε:

λΣΜ = λΣΒ - λΜΒ } ( 1 - λ)ΣΜ = ( 1 - λ)ΣΑ + ( 1 - λ)ΜΑ Με πρόσθεση κατά μέλη παίρνουμε: ΣΜ = λΣΒ + ( 1 - λ) ΣΑ + ( 1 - λ) ΜΑ - λΜΒ = λΣΒ + ( 1 - λ) ΣΑ + ΜΑ - λ (ΜΑ + ΜΒ ) =

= λΣΒ + ( 1 - λ)ΣΑ + ΜΑ - λΑΒ = λΣΒ + ( 1 - λ)ΣΑ , αφού ΜΑ=λΑΒ

Αρκεί ΣΜ=λΣΒ+ΣΑ-λΣΑ, ή ΣΜ-ΣΑ=λ(ΣΒ­ΣΑ), ή τέλος ΑΜ=λΑΒ, που ισχύει. Όμοια εργα­ζόμαστε και για τη διάταξη Σ,Β,Μ,Α.

Α ν λ = 2_ , δηλαδή Μ μέσο του ΑΒ τότε 2

η ( 1 ) γίνεται:

ΣΜ = 2_ΣΒ + ( 1 - 2_)ΣΑ = ΣΑ + ΣΒ . 2 2 2

Αν Σ ' εσωτερικό σημείο του ΑΒ τότε ομοίως αποδεικνύεται ότι: Σ 'Μ = lλΣΆ - ( 1 - λ )Σ'Β I και

1 IΣΆ - Σ 'Β I για λ = - γίνεται Σ 'Μ = .

2 2

2. Θεωρούμε τα συνευθειακά σημεία Α, Β, Γ διαφορετικά ανά δυο και τα σημεία Μ, Ν των ΑΒ, ΑΓ αντιστοίχως τέτοια ώστε:

ΑΜ=λΑΒ, ΑΝ=λΑΓ Να δείξετε ότι: ΜΝ=λΒΓ, για οποιαδήποτε διάταξη των Α, Β, Γ, όπου Ο<λ<1 .

Θεωρούμε ημιευθεία ΣΧ που περιέχει τα ση­μεία Α, Β, Γ. Τότε σύμφωνα με την προηγούμενη πρόταση έχουμε ΣΜ=λΣΒ+( Ι -λ)ΣΑ και ΣΝ=λΣΓ+( l-λ)ΣΑ, οπότε ΜΝ= I ΣΝ-ΣΜ I = I λΣΓ -λΣΒ I =λ I ΣΓ -ΣΒ I =λΒΓ

Σ Β Α Ν Γ • • • • • •

Μ χ

Αν λ = 2_ , δηλαδή Μ, Ν 2

μέσα των ΑΒ, ΑΓ αντιστοίχως τότε ΜΝ = 2_ ΒΓ. 2

Θεωρώ-ντας γνωστά από το Γυμνάσιο τα περί ομοιότητας τριγώνων αντι-λαμβανόμαστε ότι Β η πρόταση ισχύει

Α

Μ ___ Ν

Γ

και όταν το Α είναι εκτός της ευθείας ΒΓ. Πράγ-ματι τότε έχουμε : ΑΜ ΑΝ Δ Δ ΜΝ - =- =λ=:>ΑΜΝ-ΑΒΓ=>- =λ=>ΜΝ=λΒΓ ΑΒ ΑΓ ΒΓ

3. Θεωρούμε τα διαδοχικά συνευθειακά ση­μεία Α, Β, Γ, Δ και τα σημεία Μ, Ν των ΑΒ, Γ Δ αντιστοίχως τέτοια, ώστε: ΑΜ=λΑΒ και ΔΝ=λΔΓ. Να δείξετε ότι ΜΝ=λΒΓ+(1-λ)ΑΔ, όπου Ο<λ<1

Έστω ημιευθεία ΣΧ που περιέχει τα σημεία Α, Β, Γ, Δ. Σύμφωνα με την 1 η πρότα­ση έχουμε ΣΜ=λΣΒ+( 1-λ)ΣΑ και ΣΝ=λΣΓ+( l-λ)ΣΔ>ΣΜ, αφού ΣΓ>ΣΒ και ΣΔ>ΣΑ, οπότε ΜΝ=ΣΝ-ΣΜ=λ(ΣΓ -ΣΒ)+ +( 1-λ )(Σ Δ-ΣΑ )=λΒΓ +( 1-λ )ΑΔ

Σ •

Α ' '

Μ •

Β ' '

Γ ' '

Ν •

Δ . -

χ

Η πρόταση ισχύει για οποιαδήποτε διάταξη των Α, Β, Γ, Δ. με τη γενική μορφή ΜΝ = iλΒΓ ± ( 1 - λ)ΑΔ 1 .

ΕΥΚΛΕΙΔΗΣ Β ' 65 τ.l/36

Page 39: Ευκλειδης Β 65

Μαθηματικά για την Α ' Λυκείου

Παρατηρούμε ότι αν Σ=Α=Β=Μ* τότε έχουμε: Σ, Α, Β , Μ Γ ·- - - - - - - - ··

Ν ·•·--

Δ •

χ

ΣΝ = ΜΝ = λΒΓ + { 1 - λ)ΑΔ = λΣΓ + {1 - λ) ΣΔ , αφού ΔΝ = λΔΓ και ΑΜ = λΑΒ {Ο = λ · Ο)

Προκύπτει λοιπόν η πρόταση 1 . Εξάλλου όταν Β = Γ τότε: Α Β , Γ · - . . .

Μ Ν

Δ •

χ

ΑΜ = ΔΝ = λ � ΒΜ =

ΒΝ = 1 - λ = � ΑΒ ΔΓ ΒΑ ΒΔ ρ � ΜΝ = λΒΓ + ( 1 - λ)ΑΔ = { 1 - λ)ΑΔ = ρΑΔ . Προκύπτει λοιπόν η πρόταση 2 . Τελικά βασική πρόταση μπορεί να θεωρηθεί η 3 και απ ' αυτή απορρέουν οι 1 και 2 .

Αν λ = _!_ τότε ΜΝ = ΒΓ + ΑΔ .

2 2 Η σχέση εξακολουθεί να

ισχύει ακόμη και όταν ΒΓ//ΑΔ, δηλαδή σε κάθε τραπέζιο ΑΔΓΒ με βάσεις ΑΔ, ΒΓ και Μ, Ν ση­μεία των μη παράλληλων πλευρών του ΑΒ, Γ Δ α­ντιστοίχως τέτοια, ώστε ΑΜ=λΑΒ, ΔΝ=λΔΓ, οπό­τε ΜΝ//ΑΔ, ΒΓ.

Α Δ Δ Πράγματι από την ομοιότητα των ΔΚΝ,

Δ' ΚΝ ΔΝ ΔΒΓ εχουμε: - = - = λ � ΚΝ = λΒΓ . ΒΓ ΔΓ

Εξάλλου : ΑΜ=λΑΒ�ΑΒ-ΒΜ=λΑΒ� ΒΜ ΜΚ ΒΜ ( 1-λ)ΑΒ=ΒΜ�-=1-λ�-=-=1-λ� ΑΒ Μ ΒΑ

Δ Δ � ΜΚ = { 1 - λ )ΑΔ , αφού Β Μ Κ - Β ΑΔ . Άρα: ΜΝ = ΚΝ + ΚΜ = λΒΓ + { l - λ)ΑΔ .

Οι προτάσεις 1 , 2 , 3 ισχύουν επίσης ανάλογα για τόξα ή γωνίες. Συγκεκριμένα:

Αν Α, Β σημεία ενός τόξου ΣΧ και ση-� � μείο Μ εντός του ΑΒ τέτοιο ώστε: ΑΜ = λΑΒ ,

τότε ΣΜ: = λrn + ( 1 - λ)ΣΑ. , όπου Ο<λ< 1 . Αν ΟΑ, ΟΒ δυο ημιευθείες που βρίσκο­

νται στο ίδιο μέρος μιας ημιευθείας ΟΣ δηλαδή

• Τα υπόλοιπα σημεία παραμένουν διαφορετικά ανά δύο.

στο ίδιο ημιεπίπεδο με αρχική ευθεία τη ΣΧ που περιέχει την ΟΣ, και η- Α � μιευθεία ΟΜ εντός της • . Β

ΑΟΒ τέτοια ώστε

τότε Λ Λ ΑΟΜ = λΑΟΒ ,

ΣΟΜ = λΣΟΒ + ( 1 - λ )ΣόΑ όπου Ο<λ< 1 .

Αν Σ ' εσωτερικό σημείο του ΑΒ ή αν η ημιευθεία ΟΣ ' είναι εσωτερική της ΑΟΒ τότε θα έχουμε:

Σ •

ο , \

\ \ \ \ \

\ \

\

χ

fM = Ι λΣ-Α - {1 - λ ) Σ 'Β Ι \ Β Σ και

Σ 'ΟΜ = ΙλΣ 'ΟΑ - ( 1 - λ ) Σ 'όΒ j Β •

' \ Α, Μ

Μ •

Σ • e Γ

, Αν Α, Β, Γ τρία διαφορετικά ανά δυο

� σημεία ενός τόξου ΣΧ

�χ

και τα σημεία Μ, Ν των ΑΒ, ΑΓ αντιστοίχως τέ--.. � .-. τοια, ώστε: ΑΜ=λΑΒ, ΑΝ=λΑΓ, τότε ΜΝ = λΒΓ ,

όπου Ο<λ< 1 . Θεωρούμε τις

ημιευθείες ΟΑ, ΟΒ, ΟΓ διαφορετικές ανά δυο προς το ίδιο μέρος της ημιευθείας ΟΣ και τις ημιευθείες ΟΜ, ΟΝ εντός των

Λ Λ ΑΟΒ, ΑΟΓ αντι-στοίχως τέτοιες ώστε

ΑΟΜ = λΑΟΒ , Σ

Λ Λ

Β

ΑΟΝ = λΑΟΓ .

ο) ι ' - ι · ' I \ I \ I '

,

' χ

I \ I \ I \ I \ I \ Γ I Ν'

Μ • Α

Τότε ισχύει: ΜΟΝ = λΒΟΓ , όπου Ο<λ< 1 . � , Επί τόξου ΣΧ θεωρούμε τα διαδοχικά

σημεία Α, Β, Γ, Δ και τα σημεία Μ, Ν των ΑΒ, Γ Δ αντιστοίχως τέτοια, ώστε

� ------ .-. ΑΜ = λΑΒ, ΔΝ = λΔΓ .

Τότε ισχύει ΜΝ =Μ + ( 1-λ)Μ, όπου Ο<λ<1 . Για άλλη διάταξη των Α,Β,Γ,Δ θα

ΕΥΚΛΕΙΔΗΣ Β' 65 τ. l/37

Page 40: Ευκλειδης Β 65

Μαθηματικά για την Α ' Λυκείου

έχουμε: Κ1Ν = lλΒr- { 1 - λ)ΑΔI . ---�Μ--.._ Β

Γ

χ

3γ) Προς το ίδιο μέρος ημιευθείας ΟΣ θεω­ρούμε τις διαδοχικές ημιευθείες ΟΑ, ΟΒ, ΟΓ, ΟΔ

Λ Λ και τις ημιευθείες ΟΜ, ΟΝ εντός των ΑΟΒ, ΓΟΔ Λ Λ αντιστοίχως τέτοιες, ώστε ΑΟΜ = λΑΟΒ ,

ΔΟΝ = λΔΟΓ Τότε ισχύει: ΜΟΝ = λΒΟΓ + { 1 - λ)ΑΟΔ , όπου Ο<λ< 1 . Ση μείωση : Για άλλη διάταξη των ΟΑ, ΟΒ,

ΟΓ, ΟΔ θα έχουμε : ΜΟΝ = ΙλΒΟΓ - { 1 - λ)ΑΟΔ, . Οι αποδείξεις είναι ουσιαστικά αντιγραφή των

προηγούμενων. Ειδικές περιπτώσεις για λ= 1 /2

Σ

Α

I χ

Ακολουθ�ί μια εφαρμογη που

I αναφέρεται σε ευθύγραμμα τμή-

Ν

Γ

Δ ματα, τόξα και γωνίες. Εμείς θα ασχοληθούμε μό­νο με γωνίες και θα αφήσουμε για το μαθητή τις δι­ατυπώσεις και τις λύσεις των περι­

πτώσεων ευθύγραμμων τμημάτων και τόξων. Εφαρ μογή

Σε ένα ημιεπίπεδο θεωρούμε τις ημιευθείες ο

Β

ΟΑ, ΟΒ, ΟΓ με οποιαδήποτε

διάταξη τέτοιες ώστε ΑΟΓ = 60" και τις ημιευ­θείες ΟΜ, ΟΝ εντός των

Λ Λ

ΑΟΒ , ΒΟΓ

αντιστοίχως τέτοιες ώστε Λ

2 Λ

ΒΟΜ = -ΒΟΑ , 3

Λ 1 Α Α

ΓΟΝ =-ΒΟΓ . Να υπολογίσετε τη γωνία ΜΟΝ . 3

Λύση 2 Λ Λ 2 Λ Έχουμε ΒΟΜ = -ΒΟΑ και ΒΟΝ = -ΒΟΓ , 3 3

οπότε σύμφωνα με την πρόταση 2γ με λ=2/3

Λ 2 Λ 2 παίρνουμε ΜΟΝ = -ΑΟΓ = -60" = 40° . 3 3 Ας γίνει και απόδειξη απευθείας, για να φανεί

η χρησιμότητα της πρότασης 2γ. (Να διακρίνετε τρείς περιπτώσεις : ΟΓ εντός της ΑΟΒ ή ΟΒ εντός της ΑΟΓ ή ΟΑ εντός της ΒΟΓ ) .

Π αρατή ρηση

Αυτό που θέλουμε να αντιληφθεί ο μαθητής με αυτά τα παραδείγματα είναι ότι, αν γνωρίζει μια πρόταση που αναφέρεται σε υπολογισμό ευθύ­γραμμου τμήματος, μπορεί εντελώς ανάλογα να διατυπώσει και να αποδείξει αντίστοιχη πρόταση για τόξα και γωνίες αλλά και αντίστροφα. Επίσης να κατανοήσει τη μεγάλη χρησιμότητα της πρότα­σης 1 καθώς και των 1 τ, 1γ, στο να δίνει απλές και ξεκάθαρες λύσεις. Θεωρείστε για παράδειγμα τα συνευθειακά σημεία Α, Β, Γ, Δ με την παρακάτω διάταξη και τα σημεία Μ, Ν των ΑΒ, Γ Δ τέτοια, ώστε ΑΜ = �ΑΒ και ΓΝ = .!_ΓΔ . 3 3

Σ Α Γ Μ Β Ν Δ .. --·· --·-·----·-1·11---- ----<8----�---· χ

Α ν ΑΔ=α, ΒΓ=β, προσπαθήστε να υπολογίσε­τε χωρίς τη χρήση της πρότασης 1 το μήκος του ΜΝ, για να διαπιστώσετε τη δυσκολία που παρου­σιάζει, καθόσον δεν είναι συγκεκριμένη η θέση των Μ,Ν ως προς τα Β, Γ.

Εμείς αν λάβουμε υπόψη μας ότι: ΑΜ = �ΑΒ , ΔΝ = �ΓΔ , τότε με βάση την πρό-3 3 ταση 3 (σημείωση) βρίσκουμε αμέσως:

ΜΝ = ι� ΒΓ -.!.. ΑΔΙ = ι�β - .!.αl = Ι α - 2βl 3 3 3 3 3

αφού λ = j . Αναλυτικά δηλαδή , θεωρούμε ημιευ­θεία ΣΧ που περιέχει τα σημεία Α,Β,Γ,Δ και εκ­φράζουμε το ΜΝ με ευθ . τμήματα που έχουν άκρο το Σ, όπως ακρίβως κάναμε για την απόδειξη της πρότασης 3 . Έτσι έχουμε :

ΜΝ = IΣΝ - ΣΜI = I%ΣΓ + �ΣΔ - (%ΣΒ + �ΣΑ Jl =

= Ι� {ΣΔ - ΣΑ) - j{ΣΒ - ΣΓ)Ι = Ι�ΑΔ --j-BΓI =

=l.!.α - �βl

= Ια - 2β

ι. 3 3 3

Παρατηρείστε τέλος ότι μπορούμε σε κάθε τέ­τοιο πρόβλημα να θεωρούμε όλα τα σημεία επί η­μιευθείας ΣΧ, οπότε η έκφραση των τμημάτων που μας ενδιαφέρουν με τμήματα που έχουν άκρο στο Σ, όπως προηγουμένως, Αλγεβρικοποιεί και καθι-

ΕΥΚΛΕΙΔΗΣ Β' 65 τ. l/38

Page 41: Ευκλειδης Β 65

Μαθηματικά για την Α ' Λυκείου

στά ευκολότερα τα Γεωμετρικά προβλήματα αυ­τού του είδους. Τελειώνουμε με μια εφαρμογή που σχετίζεται και με ισότητες τριγώνων.

Θεωρούμε τρίγωνο ΑΒΓ και ση μείο Δ επί της ευθείας ΒΓ διαφορετικό από τα Β, Γ. Προεκτείνουμε τις διάμεσους ΑΜ, ΑΝ των τριγώνων ΑΒΓ, ΑΒΔ κατά ΜΚ=ΑΜ και ΝΛ=ΑΝ αντιστοίχως.

α) Να δείξετε ότι ΚΓ=ΛΔ β) Αν θεωρήσουμε γνωστή την πρότα­

ση 2 και τη γενίκευσή της, τότε να δείξετε ότι: ΚΛ=ΓΔ

Λ �) �Jη Δ Δ

α) Έχουμε Κ Μ Γ = Α Μ Β διότι:

ΑΜ=ΜΚ, ΜΒ=ΜΓ και Μ1 = Μ2 Δ Δ Άρα ΚΓ=ΑΒ ( 1 ) Ομοίως ΛΝΔ=ΑΝΒ διότι:

ΑΝ=ΝΛ, ΝΒ=ΝΔ και ΝΊ = Ν2 Άρα ΛΔ=ΑΒ (2) Από ( Ι ) , (2) ΚΓ=ΛΔ

Α

β) Αφού ΒΜ =�ΒΓ και ΒΝ = �ΒΔ , σύμ-

, 2 θ ' 1

φωνα με την προταση α εχουμε ΜΝ = -Γ Δ 2

(3) για οποιαδήποτε διάταξη των Β, Γ, Δ .

Ε , , ι ι πισης, αφου ΑΜ = -ΑΚ, ΑΝ = - ΑΛ, 2 2

σύμφωνα με τη γενίκευση της πρότασης 2 θα

έχουμε: ΜΝ = �ΚΛ (4)

Από τις (3), (4) παίρνουμε ΚΛ=ΓΔ

Ση μr.ίωση Το δεύτερο ερώτημα κανονικά έχει θέση

μετά την παραλληλία, για να μπορεί να απο­δειχτεί η γενίκευση της πρότασης 2 με λ= 1 12 , που θεωρήσαμε γνωστή . (Διαφορετικά παρου­σιάζεται ανακολουθία) . Τότε βέβαια θα έχου­με και απλούστερη απόδειξη , αφού :

n n n ΚΓ= ΑΒ, ΛΔ =ΑΒ � ΚΓ=ΛΔ � ΚΛΔΓ πα-

11 ραλληλόγραμο � ΚΛ = Γ Δ .

Εδώ όμως θέλουμε απλώς οι μαθητές να συνειδητοποιήσουν τη χρησιμότητα και τη χρηστικότητα της πρότασης 2 και της γενί­κευσής της. Α ν εξειδικεύσετε την παραπάνω πρόταση θεωρώντας, το Α επί της ευθείας ΒΓ, τότε δεν θα χρειαστεί η γενίκευση της πρότα­σης 2 .

Τον περασμένο Μάιο, έφυγε από κοντά μας, ο εξαίρετος συνάδελφος και συ­νεργάτης της Ελληνικής Μαθηματικής Εταιρείας Βαγγέλης Γιαννακόπου­λος.

Ο Βαγγέλης Γιαννακόπουλος γεννήθηκε το 1 938 , πήρε το πτυχίο του Μαθηματικού του 1 964 από το Πανεπιστήμιο Αθηνών και ήταν μέλος της Εταιρείας μας από το 1 97 5 . Υπηρέτησε με εξαιρετικό ζήλο, με μεγάλη αγά­

πη και συντροφικότητα την Εταιρεία μας από διάφορες θέσεις. Από τη θέση του μέλους του Δ.Σ. από το 1 977- 1 98 1 , τη θέση του Εφόρου Βιβλιοθήκης από

το 1 98 1 - 1 983 και επί σειρά ετών ως ενεργό μέλος πολλών ομάδων εργασίας και επιτροπών των εκδόσεων των περιοδικών της Ε.Μ.Ε.

Ο Βαγγέλης δεν ήταν απλά συνάδελφος. Ήταν ένας αγαπη μένος φίλος, ανήκε σε αυτούς

τους συναδέλφους που αγάπησαν την Ελληνική Μαθηματική Εταιρεία και εργάστηκαν σκληρά

για το καλό και την πρόοδό της. Ήταν πάντα παρών στις δύσκολες στιγμές, στις σημαντικές αλλαγές και σε κάθε προσπάθεια

για να πάει μπροστά η Εταιρεία μας. Ήταν πάντοτε αγωνιστικός, πάντοτε διεκδικητικός, πάντοτε ανιδιοτελής, πάντοτε πρόθυμος

για προσφορά, αλλά και άριστος Μαθηματικός. ,

Όλοι εμείς στη Μαθηματική Εταιρεία είμαστε περήφανοι που υπήρξε συνάδελφός μας και θα τον θυμόμαστε για πάντα.

ΕΥΚΛΕΙΔΗΣ Β' 65 τ. Ι/39

Page 42: Ευκλειδης Β 65

Μαθηματικά για την Α ' Λυκείου

Απόδειξη με την εις άτοπον απαγωγή Γενικά Συνήθως οι αποδείξεις των θεωρημάτων ή των

πορισμάτων γίνονται με τρόπο άμεσο με την έν­νοια ότι η αλήθεια αναδεικνύεται απ' ευθείας από τις υποθέσεις που δίνονται σε συνδυασμό με προ­ηγούμενες γνώσεις.

Θα αναφερθούμε εδώ στην έμμεση απόδειξη η την απόδειξη με άτοπο. Η διαδικασία αυτή συνί­σταται στην παραδοχή ότι η πρόταση την οποία θέλουμε να αποδείξουμε είναι ψευδής και έτσι με μια σειρά συλλογισμών καταλήγουμε σε μια αντί­φαση , σε κάτι που δεν ισχύει, σε άτοπο δηλαδή , όπου θα προκύπτει αντίθεση με τα δεδομένα ή με κάποιο θεώρημα ή πόρισμα.

Ειδικότερα η συνεπαγωγή p � q, μπορεί να αποδειχθεί αν από την σύζευξη() της λογικής προ­τάσης p με την άρνηση της q δηλαδή με την q προκύψει αντίφαση .

Μερικές φορές συμβαίνει όταν αρνούμαστε την ισχύ του συμπεράσματος μιας Μαθηματικής πρότα­σης να βρισκόμαστε μπροστά σε περισσότερες της μιας περιπτώσεις: για παράδειγμα αν θελήσουμε να αποδείξουμε με άτοπο ότι «Ο αριθμός α είναι ίσος με τον αριθμό β», τότε πρέπει να δοκιμάσουμε ότι θα οδηγηθούμε σε άτοπο τόσο αν υποθέσουμε ότι «α>β», όσο και όταν υποθέσουμε ότι «α<β».

Από ιστορικής άποψης δεν είμαστε σίγουροι σε ποιόν πρέπει κατ 'αρχήν να αποδόσουμε την σύλληψη αυτής της αποδεικτικής μεθόδου. Κάποι­οι την αποδίδουν στην Ελεατική Σχολή (που ιδρύ­θηκε στην Ελέα, πόλη της Νότιας Ιταλίας τον 5° π.Χ. αιώνα) και ειδικότερα στον Παρμενίδη και στον μαθητή του τον Ζήνωνα(495-435π.Χ.) . Άλ­λοι όμως υποστηρίζουν ότι πρώτος ο Δεινόστρατος (4ος π.Χ. αιώνας, που δίδαξε σΊ-ην Ακαδημία του Πλάτωνος) εισήγαγε ως αποδεικτική μέθοδο την απαγωγή σε άτοπο.

Η μέθοδος αυτή πάντως συχνά κατακρίθηκε ως μη ικανοποιητική καθότι έμμεση ,αφου αποφεύ­γει το εμπόδιο της άμεσης απόδειξης μιας πρότα­σης p και μεταθέτει το βάρος της στην εύρεση α­ντίφασης που προκύπτει από την άρνηση της p δη­λαδή από την p .

Ας σκεφτούμε για παράδειγμα ένα δικαστή

* Σύζευξη προτάσεων ονομάζεται η λογική πράξη με την οποία όταν συνδέσουμε απλές ή σύνθετες προτά­σεις με τον σύνδεσμο "και" προκύπτει μια νέα πρόταση που είναι αληθής μόνο όταν και οι δύο προτάσεις, είναι αληθείς .

Κουνάδης Φώτης

που πρέπει να ανακαλύψει τον ένοχο ενός εγκλή­ματος μεταξύ πέντε υπόπτων και είναι σίγουρος ότι ο ένοχος είναι κάποιος από αυτούς. Α ν τέσσε­ρις από τους υπόπτους έχουν ισχυρό άλλοθι, τότε ο δικαστής θα μπορούσε να καταδικάσει τον πέμπτο, όχι επειδή έχει στοιχεία εναντίον του αλλά επειδή σκέφτεται ότι: «αν ο ένοχος δεν είναι αυτός, τότε είναι κάποιος από τους άλλους τέσσερις, κάτι που όμως είναι άτοπο».

Ανεξάρτητα πάντως από τις όποιες επιφυλά­ξεις υπάρχουν δεν μπορούμε να παραβλέψουμε την αξία της μεθόδου που υιοθετήθηκε από πολ­λούς μελετητές με προεξέχοντες τον Αριστοτέλη και τον Ευκλείδη καθώς εκτός των άλλων έχει το πλεονέκτημα να aπλουστεύει σημαντικά τις απο­δείξεις κάποιων θεωρημάτων, εφαρμόζεται σε ό­λους τους κλάδους των Μαθηματικών και χρησι­μοποιείται και στα στοιχειώδη και στα ανώτερα Μαθηματικά.

Εφαρμογi:ς της μιr. ΙJόδου : Στα Στοιχεία του Ευκλείδη σχετικά με την

Θεωρία των Αριθμών, στο βιβλίο VII συναντάμε την πρόταση 22 :

Έστω α,β οι ελάχιστοι θετικοί ακέραιοι για

' ' α γ

δ θ ' τους οποιους ισχυει - = - με γ, ετικους α-β δ

κέραιους. Να δείξετε ότι οι α, β είναι πρώτοι μεταξύ τους.

Απόδειξη Υποθέτουμε ότι δεν είναι πρώτοι μεταξύ τους,

τότε θα είναι σύνθετοι και θα υπάρχει κάποιος θε­τικός ακέραιος λ που τους διαιρεί.

Αφού λ/α τότε α είναι πολλαπλάσιο του λ, δη­λαδή α=ε·λ με ε θετικός ακέραιος, ε<α

Όμοια λ/β, επομένως β πολλαπλάσιο του λ και γράφεται β=ζ·λ με ζ θετικός ακέραιος, ζ<β.

Οπότε έχουμε α γ ε ·

Χ γ ε γ β = δ =>

ζ -Χ = δ => ζ = δ με ε<α και ζ <β

λ τοπο αφού οι α και β ήταν από υπόθεση οι μι­κρότεροι θετικοί ακέραιοι με ίδιο λόγο με τους α­ριθμούς γ και δ. Στο βιβλίο ΙΧ των Στοιχείων του Ευκλείδη βρίσκουμε την πρόταση 20 με την οποία αποδεικνύεται ότι οι πρώτοι αριθμοί είναι άπειροι

Να αποδείξετε ότι οι πρώτοι αριθμοί είναι άπειροι.

Απόδειξη Έστω ότι οι πρώτοι αριθμοί είναι πεπερασμέ­

νου πλήθους οι: α ι ,αz, . . . ,α, .

ΕΥΚΛΕΙΔΗΣ Β ' 65 τ. l /40

Page 43: Ευκλειδης Β 65

Μαθηματικά για την Α ' Λυκείου

Σχηματίζουμε τον αριθμό Α=α1 ·αz · . . . ·αν+ Ι . Αν αυτός είναι πρώτος θα έχουμε βρει ακόμη

έναν πρώτο διαφορετικό από τους προηγούμενους καθώς είναι μεγαλύτερός τους, πράγμα άτοπο.

Α ν ο Α δεν είναι πρώτος τότε θα διαιρείται με κάποιον από τους α1 ,αz , . . . ,αν .

Έστω λ ο αριθμός από τους α 1 ,αz, . . . ,αν που δι-αιρεί τον Α. Οπότε: Α= α1 ·αz · . . . ·λ· . . . αν+ 1

Τότε άφού /JA και f.Jα1 α2 . . . λ . . . αν θα έχουμε IJ1 πράγμα άτοπο, αφού λ πρώτος. Αποκλείεται λοιπόν οι πρώτοι αριθμοί να είναι οι α 1 ,αz , . . . ,αν δηλαδή πεπερασμένου πλήθους.

"- ι . . • : . • • Πρόκειται για την καταπληκτική σε απλότητα απόδειξη που υπάρχει στα Στοιχεία του Ευκλείδη , προσαρμοσμένη όμως εντελώς στη μορφή Απαγωγής σε άτοπο.

Παραθέτουμε κάποια επιπλέον παραδείγματα της εφαρμογής της μεθόδου 1) Αν ο αριθμός α2 είναι άρτιος τότε και ο α εί­

ναι άρτιος.

Έστω ότι ο αριθμός α δεν είναι άρτιος ενώ ο α2 είναι άρτιος σύμφωνα με την υπόθεση .

Αφού ο α δεν είναι άρτιος θα είναι περιττός, δηλ. α=2κ+ 1 , κ ακέραιος.

Τότε α2=(2κ+ 1 )2=4κ2+4κ+ 1=2(2κ2+2κ)+ 1= =2ρ+ 1 όπου ρ=2κ2+2κ ακέραιος. Άρα α2 περιττός ενώ α2 άρτιος πράγμα άτοπο.

Επομένως από τη σύζευξη της υπόθεσης ότι ο α2 είναι άρτιος και της άρνησης του συμπεράσμα­τος ότι ο α επίσης είναι άρτιος προέκυψε η σύνθε-, 2 , , , τη προταση : ο α ειναι αρτιος και ο α ειναι περιτ-τός που μας οδήγησε σε αντίφαση με το ότι ο α2 είναι άρτιος. Καταλήξαμε δηλαδή σε Jie J . , '· •

Άρα ο α είναι άρτιος και η πρόταση αποδεί­χθηκε.

2) Ο αριθμός .J2 είναι άρρητος.

Αρνούμαστε τον ισχυρισμό, υποθέτοντας ότι ο αριθμός .fi είναι ρητός. Τότε υπάρχουν φυσικοί

αριθμοί α και β * Ο ώστε .fi = α ( 1 ) και Χωρίς β

βλάβη της γενικότητας δεχόμαστε ότι το κλάσμα αυτό είναι ανάγωγο δηλ. Μ.Κ.Δ.( α, β)= 1 .

'Εχουμε λοιπόν την ισότητα: .fi = α ή ισοδύ-

β α2

ναμα: -2 = 2 ( 1 ) . β 2

Τότε ( 1) => 2 = α2 => α2 = 2β2 => α2άρτιος=> β

α άρτιος => α = 2κ => 4κ2 = 2β2 => β2 = 2κ2 =>

=> β2 άρτιος => β άρτιος => β = 2λ . Άρα 2/α και 2/β οπότε Μ.Κ.Δ. (α, β ) 2:: 2 ενώ

δεχτήκαμε ότι: Μ.Κ.Δ. (α, β)= Ι πράγμα άτοπο .

3) Η εξίσωση 6χ =7+ 9y ( 1 ), δεν έχει ακέραιες λύσεις .

Υποθέτουμε ότι (ρ, κ) είναι ένα ζεύγος ακε­ραίων τιμών που επαληθεύουν την εξίσωση ( Ι ) . Τότε 6ρ = 7 + 9κ =:> 6ρ - 9κ = 7 => 3(2ρ - 3κ) = 7 => => 3 / 7 ενώ 3 % 7 πράγμα

4) Θεωρούμε το πολυώνυμο P(x)=αvxv +αv-ιχ

v-ι+ .. . +α2χ

2+αιχ+ 1 με I α; I :52, ί=1 ,2, . . . ν . Να δείξετε ότι δεν υπάρχει ρίζα ρ του πολυ-

1 ωνύμου τέτοια ώστε I ρ I � - ·

3

1 Έστω ρ ρίζα του πολυωνύμου με I ρ I � 3 ·

Τότε παίρνουμε διαδοχικά Ρ(ρ)=Ο, ανρν+αν- 1 ρν-

1+ . . . +αzρ2+α1 ρ+ Ι =0, ν+ ν- 1+ + 2+ - Ι ανρ αν- 1 ρ . . . αzρ α 1 ρ- - ,

ι= Ι ανρν+αν-Ι ρν-

Ι+ . . . +αzρ2+α l ρ Ι :::; ::5 I αν 1 · 1 ρ ! ν+ I αν- Ι I · I ρ I ν- Ι + . . . + I αz l · l ρ I + I α1 1 · 1 ρ

::5 I αν 1 · 1 ± I ν+ I αν- Ι 1 · 1 ± I ν- 1 + . . . + I αz l · l .!_ 1 2+ I α1 1 · 1 .!_ I ::5 3 3 < 2 · ( .!_ )ν + 2 · ( .!_ )ν- 1+ . . . +2 ·( .!_ )2+2 · .!_ ' - 3 3 3 3 1:52 . (_.!__ + _Ι_ + . . . + _.!__ + .!) 3v 3v-l 32 3

το άθροισμα όμως στη παρένθεση είναι άθροισμα ν πρώτων όρων γεωμετρικής προόδου με πρώτο . 1

λ ' λ . 1 . . ορο - και ογο επισης - , επομενως περνουμεε 3 3

! · (__!__ - ι) Ι <2 · 3 3v - Ι '

- - 1 3

! . (_!_ - ι) Ι <2 · 3 3v - -2

3 1 Ι < --+ Ι - 3v ' ο 1 . < -- που ειναι - 3v

Άρα δεν ισχύει ότι ρ είναι ρίζα του πολυωνύ-1 μου και I ρ I ::53 .

5) Α ν μια ευθεία τέμνει τη μια από δύο ράλληλες τότε θα τέμνει και την άλλη.

πα-

ΕΥΚΛΕΙΔΗΣ Β ' 65 τ. l/41

Page 44: Ευκλειδης Β 65

Μαθηματικά για την Α ' Λυκείου

Θεωρούμε τις παράλληλες ευθείες (ε) και (η) και την ευθεία (ζ), που τέμνει την (ε) στο σημείο Ρ (σχήμα 1 ) .

Αν υποθέσουμε ότι η ευθεία (ζ) δεν τέμνει την (η) τότε θα είναι παράλληλη προς αυτήν ή θα ταυ­τίζεται με αυτήν: α) Αν (ζ)//(η) τότε από το Ρ υ­πάρχουν δύο παράλληλες προς την (η), που είναι

καθώς αντιβαίνει στο γνωστό αξίωμα της παραλληλίας: Από ένα σημείο εκτός ευθείας άγεται μόνο μια παράλληλη προς αυτήν την ευθεία.

β) Αν (ζ)=(η) τότε αφού η (ε) τέμνει την (ζ) θα τέμνει και την (η) ενώ έχουμε (ε)//(η) πράγμα άτοπο.

ζ

ε Ρ Σχήμα 1 .

η

Δοθέντος ότι έχει προηγηθεί η πρόταση κατά τη οποία σε κάθε τρίγωνο ΑΒΓ ισχύει β � γ ::::} Β � Γ αποδεικνύεται με απαγωγή σε άτοπο, η παρακάτω πρόταση : 8) Σε κάθε τρίγωνο η μεγαλύτερη πλευρά βρί­

σκεται απέναντι από τη μεγαλύτερη γωνία.(Από τα Στοιχεία του Ευκλείδη , πρότα­ση 1 9 βιβλίο 1) .

Λ Λ Έστω τρίγωνο ΑΒΓ με τη γωνία Β > Γ . Α

Β Γ Σχήμα 4 Θα δείξουμε ότι ΑΓ>ΑΒ. Αν αυτό δεν συμβαίνει τότε υπάρχουν δύο πε­

ριπτώσεις : ΑΓ=ΑΒ ή ΑΓ<ΑΒ.

α) Έστω ότι ΑΓ=ΑΒ, οπότε το ΑΒΓ είναι ισο­σκελές τρίγωνο με βάση ΒΓ και θα είναι Β =Γ , που είναι

β) Έστω ότι ΑΓ<ΑΒ, οπότε επειδή σε κάθε τρίγωνο απέναντι από τη μεγαλύτερη πλευρά βρί-

σκεται η μεγαλύτερη γωνία, θα είναι Β < f που επίσης είναι · · . Άρα ΑΓ>ΑΒ. 9) Έστω Α, Β υποσύνολα του ίδιου βασικού

συνόλου Ω και Α ', Β ' τα συμπληρωματικά τους σύνολα αντίστοιχα. Να δείξετε ότι αν A c B, τότε Β ' c A '.

Έστω ότι Α � Β και Β ' ι;ι Α' , δηλαδή έστω χ Ε Β ' και χ � Α ' , τότε x E A c B, άρα χ Ε Β, δηλα­δή χ � Β ' , που είναι Και δύο παραδείγματα από τον 1 0) Να δείξετε ότι η εξίσωση

χ5+χ3+χ=Α 2( α-χ)+Β2(β-χ)+ Γ2(γ-χ), Α,Β,Γ,α,β,γ πραγματικοί αριθμοί έχει το πο­λύ μία πραγματική ρίζα

Θεωρούμε τη συνάρτηση f( χ)= xs +χ3 +χ-Α 2( α-χ)-Β2(β-χ)-Γ2( γ-χ) Έστω ότι η εξίσωση f(x)=O είχε δύο λύσεις κ,λ

με κ<λ και f(κ)=Ο, f(λ)=Ο. Τότε για την συνάρτηση f(x) ισχύουν οι προϋ­

ποθέσεις του θεωρήματος Rolle καθώς: f(x) συνε­χής στο [κ,λ] , f (x) παραγωγίσιμη στο (κ,λ), με f(χ)=5χ4+3χ2+ 1+Α2+Β2+Γ2, f(κ)=f(λ) (=0).

Άρα από την υπόθεση του θεωρήματος του Rolle υπάρχει τουλάχιστον ένα ξ Ε (κ, λ) ώστε f(ξ)=Ο Άρα: 5ξ4+3ξ2+ 1 + Α2+Β2+Γ2=0. " ' ' όμως αφού 5ξ4+3ξ2+ 1 + Α2+Β2+Γ2>0

Να αποδείξετε ότι, αν για μία συνάρτηση f που είναι παραγωγίσιμη στο R ισχύει :

J2 (x) + 3/(χ) = ex + χ9 + 3χ + 2007 (1) για κάθε χ Ε JR, τότε η f δεν έχει ακρότατα.

Υποθέτουμε ότι η συνάρτηση J, έχει ακρότατα και ένα από αυτά το παρουσιάζει στη θέση α.

Τότε από το θεώρημα του Fermat ισχύει

ότι: f'(a) = O. Παραγωγίζουμε τη σχέση ( 1 ) οπότε: ( 1) ::::}(j2(x)+3/(x))' = (ι! +� +3χ+2007)'::::} 2f(x)•f'(x) + 3f'(x) = ex + 9χ8 + 3 , για κάθε

χ ε IR . Αν χ=α τότε : .f'(a)=O

2/(α) · f'(a) + 3f'(a) = ea + 9α8 + 3 ::::} Ο= ea + 9a8 + 3 , που είναι γιατί το 2° μέλος της ισότητας αυτής είναι θετικός αριθμός.

Επομένως η συνάρτηση f δεν έχει ακρότατα στο R.

ΕΥΚΛΕΙΔΗΣ Β ' 65 τ. l /42

Page 45: Ευκλειδης Β 65

�ωωίUίΡ0'ί)Όm& [JJfJ([J rJ[j]ί!J llf rJc!J�[[J fJ([)f]) �f])ωe5tJωf]J

Χριστιάς Σπύρος Με αυτή την δημοσίευση δίνονται κάποιες εφαρμογές που αφορούν στους τριγωνομετρικούς αριθμούς

αθροίσματος και διαφοράς γωνιών καθώς και στους τριγωνομετρικούς αριθμούς διπλασίου τόξου. Είναι σημαντικό να τονίσουμε ότι πολλοί από τους τύπους που χρησιμοποιούνται δεν χρειάζονται απο­

μνημόνευση διότι εύκολα συνάγονται από τύπους που έχουν δοθεί ως θεωρία του σχολικού βιβλίου. Τέλος θεωρείται γνωστή η επίλυση των στοιχειωδών εξισώσεων: συνχ=Ιl, ημχ=Ιl, συνχ=Ο, ημχ=Ο

και των βασικών εξισώσεων συνχ=συνθ, ημχ=ημθ, εφχ=εφθ, σφχ=σφθ στις οποίες ανάγονται και οι συν­θετότερες εξισώσεις που ακολουθούν.

Εάν α+β= 7π

και συνασυνβ:;ο!:Ο τότε να δει-4

χθεί ότι : (1-εφα)(l-εφβ)=2.

Α ' β 7π θ ' 7π β ' φου α+ = - α εχουμε : α= -- οποτε 4 4

7π εφ- - εφβ εφα=εφ(

74π -β)= 4

7 π = 1 + εφ-εφβ 4

= - 1 - εφ β = _

1 + εφ β . ( 1 ) 1 - εφβ 1 - εφβ

Καθόσον συνασυνβ*Ο και: Ίπ π π εφ - =εφ(2π-- )=εφ(-- )=-1 . 4 4 4

Άρα από την ( 1 ) παίρνουμε:

( 1-εφα)( 1-εφβ)=( 1 + 1 + εφβ ) ( 1-εφβ)= 1 - εφβ

( 1 - εφ β+ 1 + εφβ)(1 - εφ β) = (

2 )(1 - εφ β) = 2. 1 - εφβ 1 - εφβ

Δίνεται ότι οι εφα, εφβ είναι ρίζες της δευ­τεροβάθμιας εξίσωσης: χ2-χ-1=0. Να υπολογι­σθεί η τιμή της παράστασης :

Α=6ημ2( α+β)-ημ( α+β)συν( α+β)-συν2( α+ β).

Αφού οι εφα, εφβ είναι ρίζες της δευτεροβάθ­μιας χ2-χ-1=0 τότε λόγω των τύπων Vieta θα έ­χουμε: εφα+εφβ= 1 και εφαεφβ=-1 . Άρα:

η μα +

ημβ = 1 και η μα ·

ημβ = - 1 οπότε

συνα συνβ συνα συνβ

ημασυνβ+ημβσυνα=συνασυνβ και ημαημβ=­-συνασυνβ ( 1 ) . Άρα ημ(α+β)=συνασυνβ ενώ συν(α+β)=συνασυνβ-ημαημβ και λόγω της ( 1 )

παίρνουμε: συν( α+β)=2συνασυνβ. Τότε :

Α =6συν2ασυν2β-2συν2ασυν2β-4συν2ασυν2β=Ο.

i) Να δειχθεί ότι: ημ(χ+y)ημ(χ-y)=ημ2χ-ημzy. ii) Εάν Α, Β, Γ είναι γωνίες τριγώνου τέτοιες

ώστε ημ2 Α=ημ2Β+ημ2Γ να δειχθεί ότι το τρίγω­νο είναι ορθογώνιο.

Έχουμε: ημ(χ+y)ημ(χ-y)= =(ημχσυνy+ημyσυνχ)(ημχσυνy-ημyσυνχ)= =ημ2χσυν2y-ημ2yσυν2χ=ημ2χ( 1-ημ2y)-ημ2y( 1-ημ2χ) =ημ2χ-ημ2χημ2y-ημ2y+ ημ2χημ2y=ημ2χ-ημ2y.

Έχουμε : ημ2 Α=ημ2Β+ημ2Γ=> ( i )

ημ2 Α-ημ2Β=ημ2Γ => ημ(Α+Β)ημ(Α-Β)=ημ2Γ (1 ) . Όμως Α+Β+Γ=π =>Α+Β=π-Γ=>

ημ(Α+Β)=ημ(π-Γ)=ημΓ. Άρα ( 1 )=>ημΓημ(Α-Β)=ημ2Γ =>

ημΓ(ημ(Α-Β)-ημΓ)=Ο => ημΓ(ημ(Α-Β)-ημ(Α+Β))=Ο=>ημΓ(-2ημΒσυνΑ)=0=> ημΒημΓσυνΑ=Ο=>συνΑ=Ο=>Α=90°.

Αφού Β, Γ γωνίες τριγώνου=>ημΒημΓ:;ο!:Ο. ημ2 Α=ημ2Β+ημ2Γ=>

α2 βz γ2 -- = -- + -- => αz = βz + γz => Α = 90ο 4R2 4R2 4R2 Δίνεται ότι: συνα+όυνβ=συνγ (1 ) και η­

μα+ημβ=ημγ (2).

Να δειχθεί ότι α=β+2κπ ± 2π

, κ ε Ζ. 3

ΕΥΚΛΕΙΔΗΣ Β ' 65 τ.l/43

Page 46: Ευκλειδης Β 65

Μαθηματικά για την Β · Λυκείου

( 1 )=>συν2α+συν2β+ 2συνασυνβ=συν2γ και (2)=>ημ2α+ημ2β+ 2ημαημβ=ημ2γ. Προσθέτοντας κατά μέλη θα έχουμε :

( συν2α+ημ2α)+ 2( συνασυνβ+ημαημβ)+( συν2β+ 2 ) 2 2 1 +η μ α =συν γ+ημ γ => συν( α-β)= - - => 2

2π 2π συν(α-β)=συν3=>α-β=2κπ ± ) ' κ Ε Ζ.

2π :::::>α=β+2κπ ± - , κ Ε Ζ. 3

Εάν ημαημβημγ :;t: Ο (1), ημ2(α - γ) :;t: Ο (2)

σφα ημ2β και + -- = 1 (3) τότε να δειχθεί ό-

σφ(α - γ) ημ2α

τι: σφ2β=σφασφγ. ',. Ί · ; �.·:'Π

Από τις ( 1 ), (2) εξασφαλίζεται η ύπαρξη των σφα, σφβ, σφ(α-γ) και το ότι σφ(α-γ):;t:Ο, οπότε

Ι (3 ) => σφα + Ι + σφ2β = Ι => σφασφγ + Ι Ι

σφγ - σφα Ι + σφ2α σφασφγ - σφ 2α Ι + σφ 2α ----'---'--'-----'--- + = Ι => σφασφγ + Ι Ι + σφ 2β

( σφασφγ-σφ2α)( Ι +σφ2β)+( 1 +σφ2α)( σφασφγ+ Ι )= =(σφασφγ+ Ι )( Ι +σφ2β) => σφασφγ( l +σφ2α)=

=σφ2β( 1 +σφ2α) => σφασφγ=σφ2β.

Εάν συνα+συνβ+συνγ=Ο με συνα, συνβ, συνγ :;t: Ο να υπολογισθεί η τιμή της παράστασης:

Α συν3α + συν3β + συν3γ

συνασυνβσυνγ

Εύκολα αποδεικνύεται ότι: συν(3χ)=4συν3χ-3συνχ .

Έτσι χρησιμοποιώντας τον τύπο αυτό στην παράσταση Α θα έχουμε Α= 4(συν 3α + συν 3β + συν 3γ) - 3(συνα + συνβ + συνγ)

συνασυνβσυνγ _ 4(συν 3 α + συν 3 β + συν3 γ) , - , αφου συνα+συνβ+ συνασυνβσυνγ +συνγ=Ο. (Από την ταυτότητα του Euler έχουμε : α+β+γ=Ο => α3+β3+γ3=3αβγ).

Ά Α 4 · 3συνασυνβσυνγ ρα = = Ι 2 . συνασυνβσυνγ Εφnφ,�ωγi� Ν α υπολογισθούν οι παραστάσεις :

π 3π 5π 7π Α= (1 + συν-)(1 + συν-)(1 + συν-)(1 + συν-) 8 8 8 8

π 2π 4π Β= συν - συν -συν -

7 7 7

ΛίJση . π 7π 3π 5π Ε χουμε: - +-=π καθώς και - + - = π 8 8 8 8 ' 7π π 5π 3π Ετσι συν- = -συν- και συν- =-συν-- . Άρα :

8 8 8 8

π π 3π 3π Α= (1 + συν-)(1 - συν-)(1 + συν -)(1 - συν-) = 8 8 8 8 π

π 3 1 + συν- 3π Ι +συν-= (1 -συJ -)(1 -συJ �) = (1 4 )(1 8 8 2

__ ...!..._4 ) = 2

π 3π π π Ι - συν - Ι - συν- Ι - συν- Ι + συν -= ( 4 )( 4 ) = ( 4 )( 4 ) = 2 2 2 2 Ι -σW � Ι-�

4 4 = / =� , αφού συν� =σνν( π-�)=---συν�

Γ 'ζ ' ημ2α ' νωρι ουμε οτι mMX= 2ημα, οταν ημα :;t: Ο .

2π 4π 8π -

ημ7 ημ7 η μ-Άρα: 7 Β--- · π 2π 4π 2ημ- 2ημ- 2ημ-7 7 7 8π π π ημ- ημ(π + -) η μ- 1 = -7- 7 _ _ 7_ = = = - -π π π 8 8ημ- 8ημ- 8ημ-7 7 7

2 2 3+ 4συν4χ Να δειχθεί ότι: εφ χ+σφ χ= 2 · ό-

1 -mΝ4χ ταν ημ2χ :;t: Ο .

t ) (i!\Ί] Παρατηρούμε ότι: ημ2χ :;t: Ο:::::> ημχσυχ :;t: 0=>

Υπάρχει το Α ' μέλος. Επίσης ημ2x :;t: O:::::>σW2x :;t: I:::::>2σw22x :;t: 2=>

2συν2 2 χ - Ι :;t: Ι => συν4χ :;t: 1 => Υπάρχει το Β ' 'λ , Α ' 2 2 Ι -συ�χ Ι +συ� με ος οποτε =εφ χ+σφ χ + = Ι +συ�χ Ι -συ�χ = (Ι - συν2χ) 2 + (l + συν2χ) 2 2(1 + συν 2 2χ)

(1 - συν2χ)(Ι + συν2χ) Ι - συν 2 2χ (l + Ι + συν4χ )

= 2 · 2 = 2 · 3 + συν4χ = Β ' Ι - Ι + συν4χ 1 - συν4χ

2

=

ΕΥΚΛΕΙΔΗΣ Β' 65 τ.Ι/44

Page 47: Ευκλειδης Β 65

Μαθηματικά για την Β' Λυκείου

Εφιφμογή 9' 1 Να λυθεί η εξίσωση : (1-2ημ2χ)(συνχ+ημχ)=συνχ-ημχ. (1 ) Λ ()!(!Τ� ( 1 )<:::>συνχ+ημχ-2συνχημ2χ-2ημχημ2χ= =συνχ-ημχ <:::> 2(ημχ-2ημχσυν2χ-ημχημ2χ)=Ο <:::> ημχ( 1-2συν2χ-ημ2χ)=Ο <:::> ημχ( -συν2χ-ημ2χ)=Ο<:::>ημχ=Ο (2) ή ημ2χ=-συν2χ (3) . Έχουμε : (2) <:::> χ=κπ, κ Ε Ζ . Αν συν2χ=Ο τότε ημ2χ=± 1 , οπότε η (3) δεν

ισχύει. Α ν συν2χ;t:Ο τότε (3 )<:::>εφ2χ=-1 <:::> π κπ π z ' ' <:::>2χ=κπ-- <:::>χ=- -- Κ Ε οι τιμες αυτες 4 2 8 ,

είναι δεκτές δηλαδή ικανοποιούν τη σχέση συν2χ;t:Ο (γιατί;) .

Δίνονται οι ισότητες ημα-συνα=κ (1) και ημ2α=λ-κ2 (2) με κ,λ Ε JR . Να δειχθεί ότι λ=l

και lκ i � J2 .

Λ {Jση ( 1 ):::::>ημ2α-2ημασυνα+συν2α=κ2:::::>

2 ( 2) 2 2 1-ημ2α=κ ::::::> 1-λ+κ =κ :::::>λ= l . Εξάλου ( 1 ) :::::>ημ2α=1-κ2 ::::::>

1-κ2�-1:::::>2�κ2:::::> .J2 � Ι κΙ .

Για τα τόξα α,β Ε (0, π

] δίνεται ότι : 2

.Jι - συν2α + �1 - συν2β = .Jι + συν2α . (1).

Ν α δειχθεί ότι: i) ημα+ημβ=συνα. ii) 2ημ2α-συν2β=l .

Ο ( 1 ) :::::> �2ημ 2α + �2ημ 2β = �2συν 2α ::::::>

.J2 (ημα+ημβ)= .J2 συνα ::::::> ημα+ημβ= =συνα ( αφού α, β Ε [0, � ]:::::>ημα,ημβ,συνα �0). 2

( i ) ϊ ϊ ) 2ημ2α-συν2β=4ημασυνα-συν2β =

=4ημα(ημα+ημβ)---( 1-2ημ2β)= =4ημ2α+4ημαημβ-1 +2ημ2β= =2ημ2α+ 2ημ2α+4ημαημβ+ 2ημ2β-1 = =2ημ2α+ 2( ημ2α+ 2ημαημβ+ημ2β)-1 = =2ημ2α+ 2( ημα+ημβ/ -1 =2ημ2α+ 2συν2α-1 = =2(ημ2α+συν2α)- 1 =2-1 = 1 .

Εφιφ μαyϊ;� ! 2 ' 1 Ά ν σε τρίγωνο ΑΒΓ ισχύει: 3+συν2Α +συν2Β+συν2Γ=4συνΒσυν Γ (1 ),

τότε να υπολογισθούν οι γωνίες του τριγώνου.

/ � �-J ση Χρησιμοποιώντας τύπους αποτετραγωνισμού

έχουμε : ( 1 ):::::>3+2συν2Α-1+2συν2Β-1+2συν2Γ - 1= 4συνΒσυνΓ :::::>συν2 Α+συvΒ+συν2Γ -2συνΒσυνΓ=Ο:::::> συν2 Α+( συν Β-συνΓ)2=0 ::::>συν Α =Ο και συνΒ=συνΓ:::::> Α=90° και Β=Γ=45°, αφού Α, Β, Γ είναι γωνίες τριγώνου .

1 - ημ2χ Δίνεται η συνάρτηση f(χ)= --�-

ημχ - συνχ

i) Ν α βρεθεί το πεδίο ορισμού της. ii) Να δειχθεί ότι f(χ)=ημχ-συνχ. iii) Να λυθεί η εξίσωση f(2x)=l .

n ) Για να ορίζεται η f(x) πρέπει και αρκεί ημχ­συνχ ;t: Ο . Θεωρούμε την αντίστοιχη εξίσωση ημχ­συνχ=Ο δηλαδή την ημχ=συνχ ( 1 ) .

Αν συνχ=Ο τότε ημχ=± 1 οπότε δεν ισχύει η ( 1 ). Α ν συνχ;t:Ο τότε

π ( 1 ) <:::> εφχ=1 <:::> χ = κπ + - , κ Ε Ζ. 4 Δεκτές τιμές (γιατί;) . Άρα για να ορίζεται η συνάρτηση f πρέπει και

αρκεί χ Ε JR με χ ;t: κπ + � , κ Ε Ζ. 4 i ' J> Έχουμε τότε : f(x) 1 - ημ2χ 2 ? ημ χ - 2ημχσυνχ + συν - χ

ημχ - συνχ ημχ - συνχ (ημχ - συνχ) 2

....:.....:..: ___ __:__ = ημχ - συνχ. ημχ - συνχ (2 } f(2x)= 1 <:::> ημ2χ-συν2χ= 1 <:::>

2ημχσυνχ-( 1 +συν2χ)=Ο<:::>2ημχσυνχ-2συν2χ=Ο<:::> 2συνχ(ημχ-συνχ)=Ο<:::> συνχ=Ο<:::> χ=κπ+� , κ Ε Ζ , (αφού ημχ-συνχ ;t: Ο ) . 2

Έστω η συνάρτηση :

f(x)= �1 + συν(2χ) - �1 - συν(2χ) .

i) Να βρεθεί το πεδίο ορισμού της.

ii) Να δειχθεί ότι f(x)= J2 ( iσυνχ i - Ιημχ Ι).

iii) Να λυθεί η εξίσωση f(x)= J2 .

Ο Έχουμε : - 1::s;συν(2χ) ::s; 1 ::::::> 1 + συν(2χ) � Ο, 1 -συν(2χ) � Ο για κάθε χ Ε JR . Άρα η f ( χ ) έχει πεδίο ορισμού το JR

Λόγω των τύπων αποτετραγωνισμού έ-χουμε :

f(x)= �1 + συν(2χ) - �1 - συν(2χ) =

ΕΥΚΛΕΙΔΗΣ Β ' 65 τ.l/45

Page 48: Ευκλειδης Β 65

Μαθηματικά για την Β ' Λυκείου

= � 2συν 2 χ - � 2ημ 2 χ = .J2Ισυνχl - .J2Ιημχl = = .J2 ( iσυνχi - Ιημχ Ι) .

( i i ) f(x)= .J2 => .J2 ( Ισυvχ i - Ιημχl ) = .J2 =>

Ισυνχi - Ιημχ l = 1 => Ισυνχ l = Ι ημχ l + 1 => iσυνχ l � 1 => iσυνχ l = 1 => ημχ=Ο=> χ=κπ, κ Ε Ζ

( αφού ισχύει και: iσυνχ l :;; 1 ) . Προφανώς για χ=κπ, κ Ε Ζ επαληθεύεται η

f ( χ ) = .J2 οπότε τελικά f ( Χ ) = .J2 <::::> Χ = ΚΠ, Κ Ε Ζ .

Εάν ο αριθμός ρ=συν(χ+y) είναι ρίζα της δευτεροβάθμιας εξίσωσης κ2+(2συνχ)κ+1=0 (1)

τότε να δειχθεί ότι συνχ+συνy=Ο.

Αφού η εξίσωση ( 1 ) έχει πραγματική ρίζα την ρ=συν(χ+y) θα είναι Δ � Ο .

Όμως Δ � 0=:>4συν2χ--4 � Ο=> συν2χ � 1 => συν2χ = 1 =>συνχ= 1 ή συνχ=-- 1 ( αφού ισχύει και: συν2χ :;; 1 ) . Αν συνχ= 1 τότε ( 1 ) => r+2κ+ l =O => (κ+ 1 )2=0 => κ=--1=> συν(χ+y)=-- 1=> συνχσυνy-ημχημy=--1=:> 1 συνy-Οημy =-- 1=> συνy=-- 1 (αφού συνχ= 1=>ημχ=Ο). Άρα: συνχ+συνy= 1 +( - 1 )=Ο Αν συνχ=-- 1 τότε συνy= 1 (ομοίως), οπότε συνχ+συνy=-- 1 + 1 =0

Καρδαμίτσης Σπύρος

Στον Κρότωνα της Κάτω Ιταλίας τον 6° π.χ. αιώνα ο Πυθαγόρας από την Σάμο ιδρύει την σχολή του. Στη βάση της φιλοσοφίας της σχολής των Πυθαγορείων υπάρχουν δύο αρχές το άπειρον και το πέρας, απ ' αυτές τις αρχές προ­κύπτει το «BV» και από το εν προκύπτουν οι ακέραιοι αριθμοί με τους οποίους εκφράζεται ο κόσμος. Οι Πυθαγόρειοι με βάση το πυθαγόρειο θεώρημα ορίζουν τους πλευρικούς και διαμετρικούς αριθμούς και οδηγούνται στην ανακάλυ­ψη των ασύμμετρων αριθμών.

1 . Α ν ΑΔ το ύψος οποιουδήποτε ορθογωνίου τριγώνου ΑΒΓ που αντιστοιχεί στην υποτεί­νουσα, τότε ΑΒ2 = Β Γ-ΔΓ

2 . Αν το τρίγωνο ΑΒΓ είναι ορθογώνιο και ισο­σκελές με ίσες πλευρές ΑΒ = ΑΓ = 2, τότε η υποτείνουσα του ΒΓ έχει μέτρο 2 J2

3 . Αν β=4 και γ=3 ο ι κάθετες πλευρές ορθογωνί­ου τριγώνου τότε το ύψος που αντιστοιχεί στην υποτείνουσα του έχει μέτρο ίσο με 5

4 . Υπάρχει ισοσκελές τρίγωνο ΑΒΓ με κορυφή Α τέτοιο ώστε: α2 = 2β2( 1 + συνΑ)

5 . Ένα σημείο Ρ είναι εσωτερικό σημείο του κύ-κλου (O,R) αν και μόνο αν Δ�ο.R > <Ο

6 . Σε κάθε ορθογώνιο τρίγωνο ΑΒΓ (Α=90°) με Λ

γωνία Γ =60°, ο λόγος των καθέτων πλευρών του l είναι ίσος με .J3 β

7 . Σε κάθε τρίγωνο ΑΒΓ ορθογώνιο στην κορυφή Α, ισχύει β2<α2+γ2

8 . Υπάρχει τετράγωνο με πλευρά α και διαγώνιο δ τέτοιο ώστε ο λόγος �να είναι ρητός αριθμός. α

Λ - Σ - Λ - Σ - Σ - Σ - Σ - Λ

1 . Το ύψος ισοπλεύρου τριγώνου είναι J3 , τότε η πλευρά του είναι ίση με : Α. J2 Β. 3 Γ. 4 Λ. 2 Ε. .J5

2 . Σε ισοσκελές τρίγωνο ΑΒΓ (ΑΒ = ΑΓ) ισχύει η σχέση α2 = 3 β2, τότε η γωνία Α του τριγώ­νου είναι ίση με: Α. 1 20° Β. 1 3 5° Γ. 1 50° Δ. 60° Ε. 45°

3 . Α ν ο λόγος των προβολών των καθέτων πλευ­ρών ορθογωνίου τριγώνου στην υποτείνουσα του είναι 4, τότε ο λόγος των καθέτων πλευ-ρών του είναι ίσος Α. 4 Β. 3 Γ. 2 Δ. J3 Ε. 1 /2

4. Έστω Μ είναι το μέσο της ακτίνας ΟΑ = R κύκλου (0, R). Αν η ακτίνα του κύκλου είναι R = 2 τότε η δύναμη του σημείου Μ ως προς τον κύκλο (0, R) είναι ίση με: Α. -3 Β. -2 Γ. -1 Δ. Ε. 2

5 . Σε τρίγωνο ΑΒΓ ισχύει � = i α, τότε μεταξύ 2 των πλευρών του τριγώνου α, β, γ ισχύει η σχέση : Α. β2 + γ2 = α2 Β. β2 + γ2 = 2α2 Γ. β2 + γ2 = 3α2 Δ. β2 + γ2 = 4α2 Ε. β2 + γ2 = 5α2 Δ - Α - Γ - Α - Ε

ΕΥΚΛΕΙΔΗΣ Β ' 65 τ. l/46

Page 49: Ευκλειδης Β 65

Μαθηματικά για την Β ' Λυκείου

Δίνεται ορθογώνιο τρίγωνο ΑΒΓ με υποτεί- έχουμε: Λ

νουσα ΒΓ = α και γωνία Γ =15°. Να υπολογίσε­τε τις κάθετες πλευρές β και γ του τριγώνου ως συνάρτηση της υποτείνουσας α.

Γ

β

Φέρνουμε την διάμεσο του τριγώνου ΑΜ που αντι­στοιχεί στην υποτείνουσά του.

Είναι ΑΜ=�=ΜΓ συ-2 ' νεπώς το τρίγωνο ΑΜΓ εί­ναι ισοσκελές, και επομέ-

Λ νως είναι Α1 = 1 5° και Λ

Το τρίγωνο ΑΜΕ είναι ορθογώνιο στο Μ και ε­φαρμόζοντας το πυθαγόρειο θεώρημα έχουμε :

ΕΑ2=ΜΑ2+ ΜΕ2, αλλά ΜΑ= ΒΓ , συνεπώς η πυ-2 θαγόρεια σχέση γράφεται:

ΕΑ2 = (Β;) 2 +ΜΕ2 = Β�2 + ΜΕ2 άρα

2ΕΑ2 = ΒΓ2 +2ΜΕ2 2 (2) Από τις σχέσεις ( 1 ) και (2) έχουμε : ΕΒ2+ΕΓ2=2ΕΑ2 .

: y � .- ' : ι ,

Η διάκεντρος των κύκλων (Κ, 4α) και (Λ, α) έχει μήκος 6α.

Α Β ΜΙ = 1 80° - 30° = 1 50°. Ν α υπολογίσετε το μήκος δ της κοινής εξω­τερικής εφαπτομένης τους και την οξεία γωνία που σχηματίζει αυτής με τη διάκεντρο.

Α

Ακόμη είναι συν1 50°=συν(1 80°-30")=-συν30°=-� , επομένως εφαρμόζοντας τον νόμο των συνημιτόνων στο τρί­γωνο ΑΜΓ έχουμε:

β2 = ( � )2 + ( � )2 - 2 . � . � συν 1 50° = = α2 +� - α2 · (-J3 J = α2 (2 + J3)

4 4 2 2 4

. . β α.J2;J3 αρα ειναι = 2 Στο ορθογώνιο τρίγωνο ΑΒΓ εφαρμόζοντας το

πυθαγόρειο θεώρημα έχουμε:

Β

γ2 = α2 - β2 = α2 - α2 (2 + J3) α2 (2 - J3) 4 4

Ε .

.

α�

πομενως ειναι γ = 2

Ε

Δίνεται ορθο­γώνιο αλλά μη ι­σοσκελές τρίγωνο

ΑΒΓ ( Α =90°) και η διάμεσός ΑΜ αυτού. Από το ση­μείο Μ φέρνουμε την κάθετη στην

διάμεσο ΑΜ που τέμνει την ευθεία ΑΓ στο ση­μείο Ε. Να αποδείξετε ότι: ΕΒ2 + ΕΓ2 = 2ΕΑ2•

\ Στο τρίγωνο ΒΕΓ η ΜΕ είναι διάμεσός του και

εφαρμόζοντας το πρώτο θεώρημα των διαμέσων

Α ν δι το μήκος της κοινής εσωτερικής εφα-

πτομένης τους τότε να αποδείξετε ότι � <c\ <δ. 2

Α.

:� Κ j 6α ' Λ . Μ =

Έστω ΑΒ η κοινή εξωτερική εφαπτομένη των δύο κύκλων. Φέρνουμε την ΛΓ .lAK, τότε το τετράπλευρο ΑΒΛΓ είναι ορθογώνιο και είναι ΒΛ = ΑΓ = α, οπότε ΓΚ = 4α - α = 3α. Στο ορθογώνιο τρίγωνο ΚΛΓ είναι Γ Λ 2 = ΚΛ 2 - ΚΓ2 = (6α)2 - (3α/ =36α2 -9α2 = 27α2 επο-μένως είναι δ= Γ Λ ==α J27 =3 .J3 α. Εξάλου ΚΓ = ΚΛ ::::} κλr = 30° ::::} κΜΑ == 30° . 2 . Έστω ΑΒ η κοινή εσωτερική εφαπτομένη των δύο κύκλων. Φέρνουμε την ΛΓ .lAK, τότε το τετράπλευρο ΑΒΛΓ είναι ορθογώνιο και είναι ΒΛ = ΑΓ = α, οπότε ΓΚ = 4α + α = 5α. Στο ορθογώνιο τρίγωνο ΚΓ Λ είναι

Γ Λ2 = ΚΛ2 - ΚΓ2 = (6α)2 - (5α)2 =36α2 -25α2 = 1 1 α2 επομένως είναι δ ι= ΓΛ=α.Jίl .

ΕΥΚΛΕΙΔΗΣ Β ' 65 τ. l/47

Page 50: Ευκλειδης Β 65

Μαθηματικά για την Β ' Λυκείου

Για να είναι � <δ1 <δ αρκεί αfi7 <α.Jlί<αJ27 , 2 2 δηλαδή 27<44< 1 08 που ισχύει. Σε οξυγώνιο τρίγωνο ΑΒΓ φέρνουμε την

διάμεσό του ΑΜ και το ύψος του ΒΕ. Να απο­δείξετε ότι: ΜΑ2 = ΜΒ2 + ΑΓ·ΑΕ.

Α

Β Μ Γ Στο τρίγωνο ΑΒΓ εφαρμόζουμε το πρώτο

θεώρημα των διαμέσων: ΒΓ 2 ΑΒ2 + ΑΓ2 =2ΜΑ2 + -- (1 ) 2

Επειδή Α <90° εφαρμόζοντας το θεώρημα της οξείας γωνίας έχουμε :

ΒΓ2 = ΑΒ2 + ΑΓ2 - 2ΑΓ-ΑΕ (2) Συνδυάζοντας τις σχέσεις ( 1 ) και (2) έχουμε:

ΒΓ2 ΒΓ2 = 2ΜΑ2 + --- 2ΑΓ-ΑΕ (3) 2

ΒΓ2 Αλλά (3)=> 2ΜΑ2 = -- + 2ΑΓ-ΑΕ => 2

ΜΑ2� Β: ' +ΑΓ·ΑΕ=>ΜΑ'� (Β;) ' +ΑΓ·ΑΕ=> ΜΑ2 = ΜΒ2 + ΑΓ-ΑΕ

Δίνεται ορθογώνιο τρίγωνο ΑΒΓ ( Α =90°) Λ

με γωνία Β = 30°. Προεκτείνουμε την πλευρά ΑΓ κατά τμήμα ΓΔ = 2ΑΓ. Να αποδείξετε ότι:

i) γ = β .J3 ii) ΒΔ = 2ΑΒ

iii) η ΒΓ διχοτομεί την ΑΒΔ

Β

Υ

Δ

• α Έχουμε: Β = 30° => β = -=> α = 2β. 2 Στο ορθογώνιο τρίγωνο ΑΒΓ εφαρμόζοντας το πυθαγόρειο θεώρημα έχουμε:

α2 = β2 + γ2 => (2β)2 = β2 + γ2 => γ2 = 3β2

άρα γ = β.fj Αφού ΑΓ = β, θα είναι Γ Δ=2ΑΓ=2β οπότε ΑΔ =β+2β = 3β. Στο ορθογώνιο τρίγωνο ΑΒΔ εφαρμόζοντας το πυθαγόρειο θεώρημα έχου­με :ΒΔ2=ΑΔ2+ΑΒ2=γ2+(3β)2=3β2+9β2= 1 2β2 επομένως ΒΔ = 2β .J3 =2γ=2ΑΒ.

: ; ' . Αφού ΓΒ=Γ Δ=2β και ΒfΆ = 60° θα είναι Δ = ΓΒΔ = 30° = ΓΒΑ . Άρα ΒΓ διχοτόμος της ΑΒΔ .

Δίνεται ισόπλευρο τρίγωνο ΑΒΓ πλευράς α, στην προέκταση της πλευράς του ΒΓ παίρνουμε σημείο Δ τέτοιο ώστε ΓΔ=ΒΓ. Αν Μ είναι το μέσο της ΑΔ να υπολογίσετε τις πλευρές ΜΓ, ΜΒ του τριγώνου ΜΒΓ καθώς και τα ύψη αυ­τού, ως συνάρτηση του α.

Α

' ·�:' Β '

---t- - -- -- --� Μ ' α Δ

" " 'Εχουμε ΜΓ//ΑΒ και ΜΓ= ΑΒ = � . Εξάλλου 2 2 fι = Β = 60° , οπότε Mf'B = 1 20° και ΜΒ2 = ΓΜ2 + ΓΒ2 - 2ΓΜ · ΓΒσυν120ο = α2 2 α ( 1

J 7α2 . α.J7 =4+α - 2'2α -'2 =4 . Άρα ΜΒ = -2- .

Έστω ΜΜ' , ΒΒ ' , ΓΓ ' τα ύψη του τριγώνου ΜΒΓ. Αν ΑΑ' το ύψος του ισοπλεύρου τριγώ­νου ΑΒΓ τότε ΜΜΊΙΑΑ ' και ΜΜ' = ΑΑ' = αJ3 . 2 4 Εξάλλου f'2 = f'1 = 60° = Γ3 και ΒΓ=ΑΓ=>

ΒΒΤ = ΑΑΤ => ΒΒ ' = ΑΑ' = αJ3 . 2 Τέλος: ΓΒ2 = ΜΒ2 + ΜΓ2 - 2ΜΒ · ΜΓ' =>

α2 = 7α2 + i:__ - 2 α.J7 ΜΓ' => 4 4 2

= �=>Γr=rM-rM=: � =� =>Γ'Γ = �� .

Δίνεται ισόπλευρο τρίγωνο ΑΒΓ πλευράς α και ο εγγεγραμμένος του κύκλος (O,R). Αν Μ

ΕΥΚΛΕΙΔΗΣ Β ' 65 τ. l/48

Page 51: Ευκλειδης Β 65

Μαθηματικά για την Β ' Λυκείου

είναι τυχαίο σημείο του κύκλου να αποδείξετε 5

ότι: ΜΑ2 + ΜΒ2 + ΜΓ2 = - α2 4

Α

Β σn. Δ σn. Γ Ο εγγεγραμμένος κύκλος (O,R) εφάπτεται στο

μέσο Δ της πλευράς του τριγώνου ΒΓ. Επειδή το τρίγωνο είναι ισόπλευρο, το Ο είναι το βαρύκε-ντρο του τριγώνου, επομένως είναι ΟΔ = .!_ ΑΔ και 3 ΟΕ = ΟΔ = R, τότε θα είναι:

ΑΕ = ΕΟ = ΟΔ = R = .!_ΑΔ ( 1 ) 3 Στο ορθογώνιο τρίγωνο ΑΔΒ έχουμε:

ΑΔ2 = α2 - (i)2 = α2 - : = 3:2 Ά ΑΔ α.J3 , , ( 1 ) , ρα = -2- επομενως απο την εχουμε

1 1 α.J3 α.J3 2 α2 R = -ΑΔ = - ·--=-- και R = -3 3 2 6 1 2 Στο τρίγωνο ΜΒΓ η ΜΔ είναι διάμεσός του

2 επομένως ΜΒ2 + ΜΓ2 = 2ΜΔ2 + � (2) 2 Στο τρίγωνο ΜΑΟ η ΜΕ είναι διάμεσός του

ΑΟ2 επομένως ΜΑ2 + Μ02 = 2ΜΕ2 + --= 2 =2ΜΕ2 + (2R)2 = 2ΜΕ2 + 2R2 2

άρα MA2+R2=2ME2+2R2�MA2=2ME2+R2 (3) Προσθέτοντας τις σχέσεις (2) και (3) έχουμε :

2 ΜΑ2 + ΜΒ2 + ΜΓ2 =2(ΜΔ2 + ΜΕ2) +�+ R2 2

2 2 ΜΑ2 + ΜΒ2+ΜΓ2=2(ΜΔ2+ΜΕ2)+�+� (4) 2 1 2 Αλλά στο τρίγωνο ΕΜΔ η ΜΟ = R διάμεσός του :

=2R2 + (2R)2 = 4R2 = 4 � = � 2 1 2 3

Επομένως η σχέση ( 4) γίνεται: 2 2 2 α2 α2 α2 1 5α2 5 2 ΜΑ +ΜΒ +ΜΓ =2-+-+-=-- =- α . 3 2 1 2 1 2 4

Δίνεται ορθογώνιο και ισοσκελές τρίγωνο ΑΒΓ με υποτείνουσα ΒΓ και ύψος ΑΔ=α. Προε­κτείνουμε τη ΒΓ κατά τμήμα ΓΕ =β.

Η παράλληλη από το Α προς την ΒΓ και η παράλληλη από το Ε προς την ΑΔ τέμνονται στο σημείο Ζ. Α ν η ΖΕ τέμνει την προέκταση της ΑΓ στο σημείο Η, να αποδείξετε ότι:

i) ΑΗ = (α + β)· .fi ii) ΒΗ2=2α2+2(α+β)2

. \ · :ι .

Η

Έχουμε προφανώς ΔΒ=ΔΓ=α και Α1 = Α2 = 45° . Αφού ΓΗΕ = Α2 = 45° , τα τρί­γωνα ΗΕΓ, ΗΖΑ είναι ορθογώνια και ισοσκε­λή με ΕΗ=ΕΓ=β και ΗΖ=ΑΖ=ΔΕ=α+β Άρα: ΑΗ2=ΑΖ2+ΖΗ2=(α + β)2+(α +β)2=

=2( α + β)2 � ΑΗ = (α + β) · J2 Στο ορθογώνιο τρίγωνο ΑΔΒ έχουμε ΑΒ2 = α2+α2=2α2 και εφαρμόζοντας το πυθαγόρειο θεώρημα στο ορθογώνιο τρίγωνο ΑΒΗ έχου­με: ΒΗ2 = ΑΒ2 + ΑΗ2 = 2α2 + 2(α + β)2

Η παραπάνω άσκηση αποτελεί παραλλαγή της πρότασης 10 του δεύτερου βιβλίου των Στοιχείων του Ευκλείδη και αποδίδεται στους Πυθαγόρειους. Η πρόταση αυτή τους οδήγησε στον ορισμό των πλευρικών και διαμετρικών αριθμών και στην μετέ­πειτα ανακάλυψη της aσυμμετρίας.

? �: ! Στο ορθογώνιο τρίγωνο ΒΕΗ είναι

ΒΕ = 2α+β, επομένως αν εφαρμόσουμε το πυθαγό­ρειο θεώρημα έχουμε: ΒΗ2 = (2α + β/ + β2, τότε με βάση το ίί) ερώτημα έχουμε την γεωμετρική από­δειξη της αλγεβρικής ταυτότητας:

2 ) 7 ") (2α + β) + β- = 2α- + 2(α +βΓ.

Ε Υ Κ \Ε Β . ΗΣ Β ' 65 τ. l /49

Page 52: Ευκλειδης Β 65

Μαθηματικά για την Β ' Λυκείου

. -.-: ! .. � "\ i '

Γ

Του Γιώργου Σ. Τασσόπουλου

Ένας ωραίος τρόπος για να κατανοήσουν οι μαθητές της Θετικής και Τεχνολογικής κατεύθυνσης Β 'Λυκείου τις έννοιες της πρόσθεσης διανυσμάτων και του πολλαπλασιασμού πραγματικού αριθμού επί διάνυσμα, είναι το πρόβλημα του προσδιορισμού ενός σημείου που ικανοποιεί μια διανυσματική σχέση ε­ξαρτώμενη από αυτές τις πράξεις. Θα ξεκινήσουμε από εντελώς απλά παραδείγματα, τα οποία όμως αποτε­λούν τη βάση για τα δυσκολότερα που ακολουθούν.

1 . Αν δοθούν τα διαφορετικά σημεία Α , Β τότε να βρεθεί σημείο Μ τέτοιο, ώστε:

α) ΑΜ = _! ΑΒ (1 ). β) ΑΜ = -_! ΑΒ (2) 3 3

γ) ΑΜ = _!ΜΒ (3) 3

( 1 ) <=> ΑΜ /' /' ΑΒ και ( ΑΜ) = 3_( ΑΒ) <=> 3

Το Μ ανήκει στην ημιευθεία Αχ που περιέχει 2 το Β και (ΑΜ) = -(ΑΒ) <=> Μ = Τ (σχ. α) 3

· ·- · • Α · -- ·----τ Β Σχ. α

χ

- 2 - -- -( 2 ) <=> ΑΜ = --ΑΒ <=> ΑΜ /'ι.(' ΑΒ και 3

( ΑΜ) = 3_( ΑΒ) <=> Το Μ ανήκει στην ημιευ-3 θεία Αχ · που δεν περιέχει το Β και (ΑΜ) = % (ΑΒ) <=> Μ = Σ (σχ. β)

. . . . . -� . -- . -- . .... . . - · · --· .. ·- · ..• . ·• · . ·-χ ' Σ Α Β

Σχ. β Η δυσκολία εδώ σε σχέση με τις προηγούμενες περιπτώσεις είναι ότι έχουμε δυο άγνωστα δι­ανύσματα, ενώ προηγουμένως είχαμε μόνο ένα άγνωστο διάνυσμα ως συνάρτηση του γνω-στού ΑΒ . Εύκολα όμως αναγόμαστε στην προηγούμενη μορφή, χρησιμοποιώντας διανυ­σματικές ακτίνες με αρχή το Α ή το Β. Έχουμε

ο πραγματικός αριθμός λ, τότε υπάρχει μο­ναδικό σημείο Μ τέτοιο, ώστε για κάθε ση-

μείο Ο να ισχύει ΟΜ = λΟΒ + (l - λ)ΟΑ (1)

και να βρεθεί το Μ.

( 1 ) <=> ΟΜ = λΟΒ + ΟΑ - λΟΑ <=> ΟΜ -ΟΑ = λ ( ΟΒ - ΟΑ) <=> ΑΜ = λΑΒ <::> Μ = Τ , όπου (Ατ) = lλi · (ΑΒ) και Α Τ /' /' ΑΒ όταν λ>Ο ή Α Τ /'ι.(' ΑΒ όταν λ<Ο. Ειδικά για λ=Ο έχουμε: Μ = Α .

ι ι c:-τ � ;γ- υ ; · λ= l /2 , λ=2, λ=-- 1 - 1 -α) Για λ= l /2 έχουμε ΑΜ = -ΑΒ , δηλαδή το 2

Μ είναι το μέσο του ΑΒ Α Μ • · 11 -· - .. - -- - Η -

Β •

β) Για λ=2 έχουμε ΑΜ = 2ΑΒ , δηλαδή το Μ είναι το συμμετρικό του Α ως προς Β

Α Β Μ -- ·- - · - ··/f- - --. • . ·// - . •

γ) Για λ=-- 1 έχουμε ΑΜ = -ΑΒ , δηλαδή το Μ είναι το συμμετρικό του Β ως προς Α

Μ Α Β • -- .. - /f ---------- - ---Η----·-8--

3. Α ν δοθούν τα διαφορετικά σημεία Α, Β και οι πραγματικοί αριθμοί κ, λ, τότε για κάθε σημείο Ο υπάρχει μοναδικό σημείο Μ τέ-

τοιο, ώστε: ΟΜ = κ · ΟΑ + λΟΒ (1) και να βρεθεί το Μ.

λοιπόν: , Αν κ+λ;t:Ο τότε (3 ) <=> ΑΜ = -3_ (ΑΒ - ΑΜ ) <=> ΑΜ = 3_ΑΒ - 3_ΑΜ �1)<=>0Μ=(κ+λ)(�ΟΑ+__!:_ΟΒJ =(κ+λ) ·σr

3 3 3 κ+λ κ+λ 5 - 2 - -- 2 - - κ - λ - λ -ΑΜ = -ΑΒ <::> ΑΜ = -ΑΒ <::> Μ = Ρ (σχ. γ) όπου ΟΤ = --ΟΑ +--ΟΒ Αν ρ = -- τό-3 3 5 κ + λ κ + λ κ + λ

Α •- ρ -• - • Β χ τε (2) <=> 0Τ = (1 - ρ)ΟΑ + ρ · ΟΒ <::> ΑΤ = ρ ·ΑΒ Σχ. γ Προσδιορίζε�αι λοιπόν το σημείο Τ όπως προη-

2. Α ν δοθούν τα διαφορετικά σημεία Α, Β και

ΕΥΚΛΕΙΔΗΣ Β' 65 τ. l/50

Page 53: Ευκλειδης Β 65

Μαθηματικά για την Β ' Λυκείου

γουμένως, οπότε: ( 1 ) <=> ΟΜ = (κ + λ) · ΟΤ και το Μ ορίζεται (Πρόβλημα 1 α)

Αν κ+λ=Ο τότε ( 1 ) <::> ΟΜ = -λ · ΟΑ + λ · ΟΒ <::>

ΟΜ = λ (ΟΒ - ΟΑ ) <=> ΟΜ = λ · ΑΒ και το Μ ορίζεται εύκολα.

λ 3 Αν κ=2, λ=3 τότε ρ = κ + λ

= 5 , οπότε - -

ΟΜ = 20Α + 30Β <=> ΟΜ = 50Τ <=> Μ = Σ , - - - 3 -όπου ΟΣ = 50Τ και ΑΤ = -ΑΒ (σχ. α) .

5 Α

. . ---- · - . - - - · - - - -0 τ Σ

. Β

. Θ

. ο

•· · - - · - -· - - - - · --•· · · · - - ·------ -··• - · -

Σχ. α

Α Β Κ Σχ.β

Α 2 λ 5 . λ -5 5 . ν κ= , =- τοτε ρ = -- = - = - , οποτε κ + λ -3 3

- -ΟΜ = 20Α - 50 Β <=> ΟΜ = -30Κ <:::> Μ = Θ ,

- -- -- 5 -όπου ΟΘ = -30Κ και ΑΚ = -ΑΒ ( σχ. β) .

3 . Δ

Α ν δοθεί τρίγωνο Α Β Γ και ση-μείο Ο, τότε το διάνυσμα - -ΟΜ = 20Α + 30Β + ΟΓ μπορεί να προσδιορι-στεί με τρεις τρόπους ( 20Α + 30Β) + ΟΓ,

(20Α + ΟΓ) + 30Β , (30Β + ΟΓ) + 20Α . Φυ­

σικά λόγω της aντιμεταθετικής ιδιότητας του αθροίσματος τα αθροίσματα αυτά είναι ίσα. Αποτελεί όμως ενδιαφέρουσα και δύσκολη δι­αδικασία το να διαπιστωθεί αυτό κατασκευα­στικά. Γι' αυτό το αφήνουμε ως άσκηση στους μαθητές. l Ί ' 1 ! : ΟΜ = ΟΑ + ΟΒ + ΟΓ . Τότε ΟΑ + ΟΒ = 20Τ όπου Τ το μέσο του

ΑΒ, οπότε ΟΜ = 20Τ + ΟΓ = 30G όπου - 1 -TG = - ΤΓ δηλαδή G βαρύκεντρο του ΑΒΓ.

3 Α

ο

Γ Μ

Σημείωση : Υπολογίζοντας το άθροισμα αυτό με δύο ακόμη τρόπους (ΟΑ + ΟΓ) + ΟΒ, (ΟΒ + ΟΓ) + ΟΑ μπο­

ρούμε να δείξουμε διανυσματικά ότι οι τρείς διάμεσοι του τριγώνου συντρέχουν .

Δ 4. Δίνεται τρίγωνο Α Β Γ . Ν α βρεθεί σημείο Μ

τέτοιο ώστε /2ΜΑ+ 3ΜΒ / = 10 (1) και

ΜΑ+ ΜΒ + ΜΓ // ΒΓ (2)

Σύμφωνα με τα προηγούμενα λοιπόν θα έχου­με

( 1) <=> 15� = 10 <=>5 1� = 10 <:::> �� = 2 <::>Μ ε (C) όπου (C) ο κύκλος με ακτίνα ρ=2 και κέντρο το

. . . - 3 -σημειο Κ, τετοιο ωστε ΑΚ = -ΑΒ

5 Εξ άλλου : (2) <::>3ΜGΙΙΒΓ <::>ΜGΙΙΒΓ <::>Μ ε (ε)

Δ όπου (ε ) I I ΒΓ από το βαρύκεντρο G του Α Β Γ .

Α

(ε)

Β

Τελικά λοιπόν: ( 1 ), (2)<::>Με (C)n(ε) . Το πρό­βλημα λοιπόν μπορεί να έχει δυο, μια ή καμιά λύση ανάλογα με τη θέση του κύκλου (C) και της ευθείας (ε) .

5. Δίνεται παραλληλόγραμμο ΑΒΓΔ. Να βρε­θεί σημείο Μ τέτοιο ώστε

- - - -ΜΑ + ΜΒ + ΜΓ = ΜΔ (1 )

Πρόκειται για την Άσκηση 5 σελίδα 28 του Σχολικού Βιβλίου. Κατά τη Λύση το Σχολικό Βι­βλίο χρησιμοποιεί διανυσματικές ακτίνες με αρχή

ΕΥΚΛΕΙΔΗΣ Β ' 65 τ.l/51

Page 54: Ευκλειδης Β 65

Μαθηματικά για την Β ' Λυκείου

το Β, ως να γνωρίζει εκ των προτέρων ότι τελικά το Μ θα ταυτιστεί με το Β . Κάτι τέτοιο αποτελεί αυθαιρεσία.

Δ

Το μόνο σημείο το οποίο διαφέρει από τα άλ­λα στη σχέση ( 1 ) και το οποίο θα ήταν λογικό να χρησιμοποιηθεί ως αρχή των διανυσματικών ακτι­νών είναι το Δ. Φυσιολογικότερο βέβαια είναι να αντικαταστήσουμε το άθροισμα των διανυσματι-

- - - -

κών ακτινών ΜΑ, ΜΒ, ΜΓ με 3MG , όπου G το βαρύκεντρο του ΑΒΓ. Έτσι θα έχουμε ( 1 ) <:::::> 3MG = ΜΔ . Αναγόμαστε λοιπόν στο πρό­βλημα 1γ, με σταθερά σημεία τα Δ, G. Έτσι έχου­με : ( 1 ) <:::::> -3GM = GΔ -GM <:::::> -2GM = GΔ <:::::>

GM = _ _!_GΔ <:::::> GM /'./ GΔ και 2

( GM ) = _!_( GΔ ) <:::::> G = Β (γιατί;) 2

Στην ίδια σχέση καταλήγουμε αν χρησιμοποι­ήσουμε ως αρχή το Δ. Πράγματι τότε έχουμε :

( Ι ) <:::::> ΔΑ - ΔΜ + ΔΒ - ΔΜ + ΔΓ - ΔΜ =

-ΔΜ <:::::> ΔΑ + ΔΒ + ΔΓ = 2ΔΜ <:::::> <:::::> 3ΔG = 2ΔΜ <:::::> -3GΔ = 2 ( GM - GΔ ) <:::::>

- I -GM = --GΔ <:::::> Μ = Β . 2

Είναι φυσικό πλέον να έχουμε απλούστατη λύση αν πάρουμε ως αρχή το Β . Έχουμε τότε :

( 1 ) <:::::> ΒΑ - ΒΜ - ΒΜ + ΒΓ - ΒΜ = ΒΔ - ΒΜ

<:::::> ΒΑ + ΒΓ - ΒΔ = 2ΒΜ <:::::> - - - - -

ΒΔ - ΒΔ = 2ΒΜ <:::::> ΒΜ = Ο <:::::> Μ = Β . Αφήνουμε στους μαθητές ως ανοιχτό πρόβλη­

μα να προσδιορίσουν το σημείο Μ χρησιμοποιώ­ντας ως αρχή το Α ή το Γ.

Στην πρώτη περίπτωση με αρχή το Α θα οδη-- - - -

γηθούν στη σχέση ΑΒ + ΑΓ - ΑΔ = 2ΑΜ και εί-ναι ενδιαφέρον να αποδείξουν ότι Μ=Β κατα­σκευάζοντας το πρώτο μέλος με τρεις τρόπους:

(ΑΒ + ΑΓ ) - ΑΔ, (ΑΒ - ΑΔ ) + ΑΓ ,

( ΑΓ - ΑΔ ) + ΑΒ . Α ν άλογα να εργαστούν με αρχή

το Γ ή οποιοδήποτε σημείο Θ.

6) Δίνεται τραπέζιο ΑΒΓ Δ με ΔΓ = 2ΑΒ . Να βρεθεί σημείο Μ τέτοιο ώστε

- - - -

ΜΑ + ΜΓ + ΜΔ = 3ΒΑ (1)

Δ Λ · τρQΊπ :ί�ς Αν G το βαρύκεντρο του Α Δ Γ τό-

τε όπως και προηγουμένως:

<:::::> ABMG παραλληλόγραμο <:::::> ΒΜ = AG <:::::> - 2 - - 2 -ΒΜ = -ΑΝ <:::::> ΒΜ = -ΒΓ <:::::> Μ = Τ ,

3 3 - - 2 όπου ΒΤ /' /' ΒΓ και (ΒΤ ) = - (ΒΓ ) , αφού για

3 το μέσο Ν του ΓΔ έχουμε :

- Ι - - - -ΝΓ =- ΔΓ = ΑΒ � ΑΒΓΝ παραλληλόγραμο � ΑΝ = ΒΓ

2

Δ,-· · ·--· -+1---- �-- -- ---+ι ·--- Γ . \ G j' //

\ ι,ι------,---1 τ

\\\\. / ' / \ /f

\L� - -A

/ Η-------1 Β

/

β ' :φόrως Αντιλαμβανόμαστε πλέον ότι αν χρησιμοποιήσουμε διανυσματικές ακτίνες με αρχή το Β η λύση θα είναι απλούστερη .

Πράγματι ( 1 ) <:::::> ΒΑ - ΒΜ + ΒΓ - ΒΜ + ΒΔ - ΒΜ = 3ΒΑ <:::::>

ΒΓ + ΒΔ - 2ΒΑ = 3ΒΜ <:::::> 2ΒΝ - 2ΒΑ = 3ΒΜ <:::::> 2ΑΝ = 3ΒΜ <:::::> ΒΜ = �ΑΝ <:::::> ΒΜ = � ΒΓ <:::::> Μ = Τ

3 3

Είναι ενδιαφέρον όμως να αποδειχθεί από τους μαθητές ότι Μ=Τ και στις περιπτώσεις που ως αρ­χή θα ληφθεί κάποιο από τα σημεία Α, Γ, Δ.

Για παράδειγμα με αρχή το Α έχουμε : ( Ι ) <:::::> ΑΓ + ΑΔ + 3ΑΒ = 3ΑΜ και για εξάσκηση ας αποδείξουν το ζητούμενο, δηλαδή Μ=Τ κατα­σκευάζοντας το πρώτο μέλος με τρεις τρόπους ( ΑΓ + ΑΔ ) + 3ΑΒ, ( ΑΓ + 3ΑΒ) + ΑΔ,

(ΑΔ + 3ΑΒ ) + ΑΓ,

Το ίδιο να δείξουν όταν ληφθεί ως αρχή το Γ ή το Δ ή οποιοδήποτε σημείο Θ.

ΕΥΚΛΕΙΔΗΣ Β ' 65 τ. l /52

Page 55: Ευκλειδης Β 65

ΙΙ•Ιιιιι•ι••• ,.. φ r Ί�IΙΙ Ί•ιι Αrι•ά•ιι

Κατεύθυνση Μιγαδικοί Αριθμοί

Οι μιγαδικοί αριθμοί παρότι δημιουργήθηκαν με αφορμή την επίλυση της εξίσωσης χ2+ Ι =0, η οποία δεν έχει πραγματικές ρίζες, χρησιμοποιού­νται ευρέως για την επίλυση πραγματικών προ­βλημάτων σε πολλούς τομείς, όπως η Αεροδυνα­μική, η Υδροδυναμική, η Μηχανική Συζευγμένων πεδίων κ.α.

Ακολουθούν σχόλια, παρατηρήσεις και ασκή­σεις, που αποτελούν ένα εργαλείο για την αντιμετώ­πιση στοιχειωδών προβλημάτων, μιγαδικών αριθμών και βοηθούν στην πιο πλήρη κατανόησή τους.

Σzόλια - παρατη ρήσεις 1 . Επειδή ισχύουν (l+i)2=2i και (l-i)2=-2i,

χρησιμοποιούμε τις παραπάνω ισότητες για να υπολογίσουμε δυνάμεις του 1±ί. Για παράδειγμα, να υπολογίσετε το (1 +i)

100

λίι σ η Είναι ( 1 +ί) ι οο=[( l +i)2]so=(2i)so=2so . iso=2so . iso=_2so, αφού iso=i4' I 2+2= i2

2. Από μια ισότητα δύο μιγαδικών υψωμένων σε μεγάλες δυνάμεις, προκύπτουν ίσα μέτρα αυτών, αλλά το αντίστροφο δεν ισχύει. Για παράδειγμα, αν (1-z)

1000= z1 000, z ε C (1) να αποδείξετε ότι Re(z)=112 Λ [Jση Έχουμε : ( 1 )�1 1-zl ι ooo=Ιzl ι ooo� �1 1-zi=Ιzl�l 1-zi2=Ιzl2

� ( 1 - z ) ( 1 - ;) = z · ; � 1 - ; - z + z; = z; �

� 2 Re ( z ) = 1 � Re(z)= l l2 .

3. Έστω z1 ,z2 ε C και P1(z1), P2(z2) οι εικόνες τους. Αν z= λιzι+ λ2z2 (λι , λ2 ε R) και Ρ η ει­κόνα του z, τότε ισχύει η ισοδυναμία:

- - -

z=λιzι+ λzzz<=> ΟΡ = λ1 ΟΡι + λ2 OPz (όπου

Ο η αρχή των αξόνων) διότι κάθε μιγαδικός

Θανάσης Τσιούμας - Βαγγέλης Τσιούμας

μπορεί να παρασταθεί και με τη διανυσμα­τική του ακτίνα. Για παράδειγμα, αν z1 ,z2 ε C και Μ1(zι), M2(z2) οι εικόνες τους, τότε ισχύει:

- -

Ζ ι Ζ2 = Ζι Ζ 2 <=> Μ1 , Ο, Μ2 συνευθειακά σημεία (όπου Ο η αρχή των αξόνων). Λύση

Αν z2 * Ο τότε zι � =�ι Ζ2 <=>� =t <=>� =[� ) z - -

<=> -1 = λ ε .IR <:::::> z1 = λz2 <=> ΟΜ1 = λΟΜ2 z2

<::::> ΟΜ1 Il OM2 <=> οπότε τα σημεία Μ1 , Ο, Μ2 είναι συνευθειακά. Α ν z2=0 τότε Μ2=0 και η ισοδυναμία είναι προφανής. β ' τρόπος Αν z1 =α1 +β 1 - Τ , z2 =α2 +β2 - Τ όπου

α1 , β1 , α2 , β2 ε JR , τότε ΟΜ1 = (α1 , β 1 ) , ΟΜ2 = ( αυβ2 ) οπότε:

ΟΜ1 11 ΟΜ2 <:::::> det ( ΟΜ1 , ΟΜ2 ) = Ο <=> Ι αΙ β Ι Ι . <=> = Ο <:::::> α1 β2 = α2β2 επισης α2 β2

Ζ1Ζ2 = z1 z2 <=> α1β2 = α2β 1 (μετά από πράξεις) .

4. Α ν έχουμε ως υπόθεση μια από τις ισότητες z2-z+1=0 ( z ε C ) ή z2+z+1=0 τότε αν τις πολλαπλασιάσουμε με z+1 ή z-1 αντίστοιχα προκύπτει ότι z3=--1 και z3=1, οπότε μπο­ρούμε να aπλοποιήσουμε (ή να υπολογίσου­με) παραστάσεις με μεγάλες δυνάμεις του z. Για παράδειγμα, αν z2 -z+ 1 =Ο να αποδείξετε

3 8 1 ότι: α) z =--1 β) z + -8 = -1 .

z

ΕΥΚΛΕΙΔΗΣ Β ' 65 τ. l/53

Page 56: Ευκλειδης Β 65

Μαθηματικά για την Γ Λυκείου

α) Έχουμε: z2-z+ 1 =0 � (z+ 1 )(z2-z+ 1 )=0� z3 + 1 =O�z3=-1 β) Είναι z;t:O αφού αν z=O θα είχαμε 1 =Ο άτο­πο. Άρα z8= z6 .z2=(z3)2z2=(- 1 )2z2=z2 οπότε

s 1 2 1 z4 + 1 z3 • z + 1 z +s = z + -2 = --2 - == z = z z z z = -z + 1 = -z + 1 = _ 1 . z2 z - 1

Αφού από z2 -z+ 1 =Ο προκύπτει z2 = z-1 . 5. Είναι γνωστό ότι αν α, βΕR τότε από την

ισότητα α2+β2=0 προκύπτει ότι: α=β=Ο. Όμως αν z1,z2 Ε C και z12+z2 2=0 δεν σημαίνει απαραίτητα ότι z1=z2=0, αφού για παρά­δειγμα, z1=1:#0 και z2=i:f0 ενώ 1 2+ί2=0. Η παράσταση z12+z2 2 μπορεί επίσης να ανα­λυθεί σε γινόμενο ως εξής:

z12+ z/= z/ - (iz2)2=(zt- iz2)(zt+iz2) Ε , , , 2+ 2 ο πομενως απο την ισοτητα Zt Ζ2 = προ-κύπτει z1=iz2 ή z1=-iz2 καθώς και Ιzt i=Ιz2 1

Έστω Zt2+ z/=0 ( z 1 , z2 Ε C ), να αποδείξετε ότι α) z1 10+ z/0=0 β) z12ν+ z/ν=Ο όπου ν περιττός φυσικός γ) l z 1 - z2 1 = ν'ilzι l = ν'21z2 1 ·

Σύμφωνα με το παραπάνω σχόλιο είναι z ι=iz2 ή z ι=-iz2 Aν zι=iz2 τότε έχουμε: α) z ι 1 0+ z2 1 0=(iz2) 1 0+ z2 1 0=i 1 0z2 1 0+z2 1 0=-z2 1 0+z2 1 0=0 αφού ί 1 0= i2=-1 β) Ζ ι 2ν+z2 2v=(iz2)2v+z2 2v=i2vz/v+z/v=

=(-1 )vz/v+z/v=O διότι (-1 )ν=-1 όταν ν περιτ­τός φυσικός.

γ) l z1 - z2 1 = 1 iz2 - z2 1 = 1 ( i - 1 ) z2 1 = 1- 1 + i l l z2 1 = = h Ιz2 l = J2 lzι l ·

Ομοίως αν z1=- iz2.

6. Βασικοί γεωμετρικοί τόποι Έστω z Ε C . Τότε: Η εξίσωση : Ιz -z0Ι=ρ (1), ρ>Ο παριστάνει κύκλο (c) με κέντρο το K(z0) και ακτίνα ρ. Η εξίσωση : Ιz-zt i=Ιz-z2 1 παριστάνει τη με­σοκάθετη του τμήματος με άκρα τα σημεία A(z1) και B(z2). Η εξίσωση : Ιz-zt i+Ιz-z2 1=2α όπου α>Ο, Zt , z2 δοσμένοι αντίθετοι μιγαδικοί και Ιzt-Ζ2 1<2α, παριστάνει έλλειψη με εστίες E(zt) και E '(z2) και μεγάλο άξονα 2α.

( 1 ) γράφεται (ΜΕ)+(ΜΕ ')=2 ·5>2 ·3 (ορισμός της έλλειψης) . Η εξίσωσή της θα είναι:

χ2 y2 -+- = 1 αφού β2=α"'2 - γ2= 1 6 1 6 25 Εντελώς ανάλογα: Η εξίσωση : Ι lz-zt l-lz-z2 1 1=2α με α>Ο και Ιzt-Ζ2 1>2α παριστάνει υπερβολή με απόστα­ση κορυφών 2α και εστίες E(zt), E '(z2) (τα z 1 , z2 είναι γνωστοί αντίθετοι μιγαδικοί α­ριθμοί) .

7. Η αναζήτηση γεωμετρικών τόπων, χρειάζε­ται προσοχή, διότι θα πρέπει να ισχύουν ό­λες οι ισοδυναμίες, ιδιαίτερα σε θέματα, που αφορούν μέτρα μιγαδικών, αφού δεν ισχύει: z1=z2 (:)lzt i=Ιz2 1 αλλά μόνο :

z1=z2 �1zt i=Ιz2 Ι Για παράδειγμα, αν ( 1 +iz ')2= 1 ( 1 ), z ' Ε C τότε ( 1 )�1 1 + iz Ί2= 1�I-i2+ iz Ί= 1�1i(z '- i) l= 1 � Ιz '-i l= 1 που σημαίνει ότι η εικόνα του z ' ανή­κει στον κύκλο Ιz-i l= 1 που έχει με κέντρο Κ( Ο, 1 ) και ακτίνα 1 . Το όμως , διότι αν π.χ. z0= 1 +ί (που η εικόνα του ανήκει στον παραπά­νω κύκλο) τότε : ( 1 +ίz0)2=[ 1 +ί( 1 +ί)]2=( 1 +ί- 1 )2=-1:;i: 1 δηλαδή ο αριθμός z0 δεν επαληθεύει την ( 1 ) Επομένως δεν μπορούμε να ισχυριστούμε ότι ο γ.τ. των εικόνων των μιγαδικών που ικανοποι­ούν την ( 1 +iz )2= 1 είναι ο κύκλος, αλλά ότι οι εικόνες των z ανήκουν στον κύκλο. Εύκολα μάλιστα, μετά από πράξεις διαπιστώνουμε ότι μόνο δύο αριθμοί ικανοποιούν την ( 1 +iz )2= 1 , οι αριθμοί z 1=0+0i και z2=0+2i των οποίων οι εικόνες προφανώς ανήκουν στον κύκλο Ιz-i 12= 1 .

θ , 1 +αί τrn Ομοίως οι εικόνες των αρι μων z =--. , X E JN.. α+ι ' ' λ 2 2 1 Δ αλ ' ανηκουν στον κυκ ο χ + y = . εν κ υ-

πτουν όμως όλο τον κύκλο αφού το σημείο του , 1 + αί , Ο+ί δεν παίρνει τη μορφη --. για καποια α + ι

τιμή του α Ε ffi. διαφορετικά θα είχαμε - 1 = 1 πράγμα άτοπο.

8. Μέγιστα και ελάχιστα μέτρων μιγαδικών α­ριθμών. Έστω Ιz-il=l, (z Ε c) να βρεθεί η μέγιστη (max) και η ελάχιστη (min) τιμή του Ιz+4+ 2i l

Για παράδειγμα, η εξίσωση Ιz-3 i i+ Ιz+3 i l= 1 0 Οι εικόνες των z κινούνται στον κύκλο κέ-( 1 ) παριστάνει έλλειψη με μεγάλο άξονα 2α= 1 0 ντρου Κ(Ο, 1 ) και ακτίνας 1 , ενώ το Ιz+4+2i l γρά­και εστίες τα σημεία Ε(Ο,3) και Ε '(Ο,-3), διότι η φεται Ιz-(-4-2i) l και εκφράζει την απόσταση των

ΕΥΚΛΕΙΔΗΣ Β ' 65 τ.l/54

Page 57: Ευκλειδης Β 65

Μαθηματικά για την Γ Λυκείου

εικόνων του z από το Α(-4,-2) που είναι η εικόνα του z 1=-4-2i

· · · · - - · · · · · · · · · · · · · · · · · · · · · · · · · · · · · · · · · · · · · - · · · ·

A(-4-2i) Από το σχήμα προκύπτει

Υ ι

minlz+4+ 2ί i=(ΑΒ)=(ΚΑ)-ρ=5-1 =4 maxjz+4+ 2i i=(ΑΓ)=(ΚΑ)+ρ=5+ 1 =6

χ

Σε πολλά βιβλία δίνεται η παρακάτω λανθασμένη λύση με βάση την τριγωνική ανι-σότητα: l lz1 1 - lz2 1 1 � lzι ± z2 1 � lzι l + lz2 1 -

Γ ράψουν λοιπόν: z + 4 + zi = ( z - ί ) + ( 4 + 3ί ) , ο-

πότε l lz - i l - l4 + 3i l l � Ιz + 4 + 2i l � ιz - i l + l4 + 3i l οπότε 4 � lz + 4 + 2i l � 6 => min lz + 4 + 2i l = 4

και max lz + 4 + 2i l = 6 . Όμως η διαδικασία αυτή εξασφαλίζει απλώς

ότι το 4 είναι κάτω φράγμα και το 6 είναι άνω φράγμα του lz + 4 + 2i l και όχι ελάχιστη και μέγι­στη τιμή του αντιστοίχως. Για να συμβαίνει αυτό χρειάζεται να δείξουμε ότι υπάρχουν τιμές z ' και z" του z τέτοιες ώστε lz ' + 4 + 2i l = 4 και

lz" + 4 + 2i l = 6 πράγμα όχι και τόσο εύκολο να διαπιστωθεί Αλγεβρικά.

Στα επόμενα σχήματα μπορούμε να βρούμε (γεωμετρικά) το μέγιστο ή το ελάχιστο του Ιz l όταν το M(z) κινείται σε ευθεία (σχ. 1 ) καθώς και του Ιz-wΙ όταν το M(z) κινείται σε ευθεία και το P(w) σε κύκλο ( σχ.2) ή όταν κινούνται και τα δύο σε κύκλους ( σχ. 3) .

Υ

' ._M(z)

. , , . ,, , c, __.----.. ,'-. ,

/ κ

\

Ρ( ω)

(ΟΚ)= minlz l

minlz-wi=(AB)

(μέγιστη απόσταση δεν υπάρχει, γιατί;)

Cι � - - -M(z)

\ -· · - - - ,\Ρ(ω) Κ \ Ι Λ · . Ε ϊ - - - - - - ο----""Ά,---....,da:·� - - - - - z

' I minlz-wi=(AB) maxΙz-wΙ=(EZ)

1 . Δίνεται η συνάρτηση f(z) =(z+i)(�+i) , zεC α) Αν zι=l+i να βρεθούν ο ι μιγαδικοί

z2=f(zι) και z3=f(f(zι)) β) Αν Μι, Μ2, Μ3 οι εικόνες των zι, z2, z3

αντίστοιχα να αποδείξετε ότι τα σημεία Μι, Μ2, Μ3 είναι κορυφές ορθογωνίου τριγώνου.

γ) Να βρείτε το σύνολο των εικόνων των z ώστε i) f(z) εJR ii) f(z) φανταστικό.

Έχουμε f ( z) = zz + zi + zi - 1 = x2+y2 -1 + 2χί z = x + yi ) ( 1 )

α) f(z ι )=f( l +i)= 1 2+ 1 2-1 +2 · 1 - i= l +2i f(f(z 1 ))= f( 1 +2i)= 1 2+22-1 +2i=4+2i

(όπου

β) Είναι Μ 1 ( 1 , 1 ), Μ2( 1 ,2) και Μ3(4,2) άρα Μ1Μ2 = (0, 1 ) , Μ2Μ3 = (3, 0) και Μ1Μ3 = (3, 1 ) .

Παρατηρούμε ότι Μ1Μ2 · Μ2Μ3 = 0 · 3 + 1 · 0 = 0 άρα Μ1Μ2 .1 Μ2Μ3 δηλαδή το τρίγωνο ΜιΜ2Μ3 είναι ορθογώνιο στο Μ2 .

( Ι ) γ) i)f(z) εJR �Im(f(z))=O � 2χ=Ο<::>χ=Ο

Άρα ο γ. τόπος των εικόνων των z είναι ο άξο­νας y 'y.

ίί) f(z) φανταστικός �χ2+/-1=0<::>χ2+y2= 1�ογ. τόπος των εικόνων των z είναι ο κύκλος κέ­ντρου 0(0,0) και ακτίνας 1

2. Αν z3=1 και z � JR , να αποδείξετε ότι:

α) z - ! φανταστικός β) jz+l l=l z γ) (iz)2oιo+(l +z2)2oo9 -(iz)2oos=O

α) 'Εχουμε z3= 1 => Ιz l3= 1 => Ιz i= 1=> Ιz i 2= 1 - 1 1 - 1 -=> z · z = => - = z => z - - = z - z =>

z z

z - _!_ = 2βί => z - _!_ z z

είναι φανταστικός (όπου z = α + β ί , α, β ε R ) . β) Επίσης z= 1 =>z3-1=0=>(z-1 )( z2+z+ 1 )=0 ( 1 )

όμως z � JR => z:;t:1 οπότε ( 1 ) => z2+z+ 1=0 => z+ 1 =-z2=>lz+ 1 Ι=Ι-z2 Ι= Ι-z Ι2=Ιz Ι2= 1 ,λόγωτου (α) .

γ) Έχουμε ( iz )20 ι ο+( 1 +z2)2oo9--{ iz )2oos=

ΕΥΚΛΕΙΔΗΣ Β ' 65 τ.l/55

Page 58: Ευκλειδης Β 65

Μαθηματικά για την Γ Λυκείου

=( i4)so2 · iz( z3)67o+[ ( -z )3]669 ·zz -( i4)soz · (z3)669 ·z= =-1-z2 -z=-( 1 +z+z2)=0, λόγω του (β) .

3. Δίνονται οι μιγαδικοί z1 , z2, z3 τέτοιοι ώστε να ισχύουν: z/ = 1 + .J3i, z/ = 26 + νl53i, z/ = 3 + .J7i α) Να βρείτε τα μέτρα τους β) Να αποδείξετε ότι z1+z2+ Z3;CO γ) Να αποδείξετε ότι

2z 1 z 2 + 9z 1 z3 + 2z2z3 = 6 Ζ ι + Zz + z3

α) Είναι Ιz , 1 2 = 1 1 + �il = 2 άρα Ι z, I = J2, l z2 1 3 = 1 26 + ν'53il = 27 άρα lz2 1 = 3 και

l z3 1 4 = 13 + J?il = 4 άρα l z3 1 = J2. β) Υποθέτουμε ότι z 1+z2+z3=0 ( 1 ) .

Τότε ( l )=>zz=-z ι-z3 => Ιzz Ι= Ιz ι+z3 Ι::; Ιz ι Ι+Ιz3 Ι=> =>3�fi+fi => 3 � 2J2 � 9 � 8 άτοπο. Επομένως z ι+zz+ z3;CO .

2 z1 = - , ομοίως _ 9 _ 2 Ε ' z2 =- και z3 = - . ιναι z2 z3

z ,

I I 1- _ _ 1 2 9 2 z1 + z2 + z3 = z1 + z2 + z3 = -+ -+ - = z1 z2 z3 _ 2z2z3 + 9z1 z3 + 2z1 z2 _ l 2z2z3 + 9z1 z3 + 2z 1z2 1 _ - z, z2z3

- l z , l l z2 1 1 z3 1 -

_ l 2z2z3 + 9z1 z3 + 2z1 z2 l _ l 2z2z3 + 9z1 z3 + 2z1 z2 1 - J2 - 3 · J2 - 6 Άρα 2z2z3 + 9z1 z3 + 2z1 z2 = 6 . z 1 + z2 + z3

4. 'Εστω z ε C ώστε (2z - 1 )5 = (z - 2)5 (1)

α) Να βρείτε τη γραμμή στην οποία ανή­κουν οι εικόνες των z

β) Να αποδείξετε ότι οι εικόνες των ω όπου 2ω=3z+ 2i ανήκουν σε κύκλο c.

γ) Αν Ρι (ω1), (ω2) δυο σημεία του κύκλου c, να αποδείξετε ότι Ιωι-ω2Ι::;3

α) Παίρνουμε μέτρα στην ισότητα ( 1 ) και έχουμε ( 1) � l2z - 11 5 = lz-�5 =>

j 2z - 1j5 = lz - 21 5 � l 2z - 1 l = l z- 21 � � l 2z - 1 1 2 = l z - 2 1 2 � ( 2z - 1 ) ( 2z - 1 ) =

= (z - 2) (z - 2) � 3zz = 3 � lzl 2 = 1 => lzl = 1 . Άρα οι εικόνες των z ανήκουν στον μοναδι­καίο κύκλο.

β) Ομοίως 2ω=3z+2i=> 2ω-2ί =3z =>2Ιω-ί Ι=3 Ιz Ι =>lω-i l=3/2 => οι εικόνες των ω ανήκουν σε κύκλο c με κέντρο Κ(Ο, 1 ) και ακτίνα ρ=3/2

γ) Ισχύει ότι η χορδή Ρ 1Ρ2 είναι μικρότερη ή ίση της διαμέτρου του κύκλου c, οπότε: Ιω, - ω2 1 = (Ρ1 Ρ2 ) � 2ρ = 3

( ' Με εφαρμογή της τριγωνικής ανι-σότητας έχουμε:

Ιω, - ω2 Ι = I (ω, - ί ) - (ω2 - i ) I � Ιω, - i l + lω2 - i l = 3 3 = - + - = 3 . 2 2

5. α)Θεωρούμε τους μιγαδικούς z1, z2, z3 για τους οποίους ισχύουν:

zι+z2+z3=0 (1) και z/+z/+z/=0 (2) Να αποδείξετε ότι: ί) Ιzι Ι=Ιz2 Ι=Ιz3 Ι ίί) Ιzι-z2 Ι=Ιz2-z3 Ι=Ιzι-z3 Ι = .J3 Ιzι l β) Αν για τους μιγαδικούς z, ν, ω ισχύει: z2+ν2+ω2= zν+zω+νω Ν α αποδείξετε ότι τα σημεία Ρ1, Ρ2 και Ρ3 που είναι οι εικόνες των z, ν, ω (αντίστοιχα), είναι κορυφές ισοπλεύ­ρου τριγώνου.

i) Έχουμε ( 1 ) =>zz =-zι-z3 οπότε η (2) γίνεται Ζ ι 2+( -Ζ ι-Ζ3)2+z/=0:::::> =>zι2+z ι2+ 2z ιz3+Z3 2+z3 2=0=>zι2+z3 2= Ζ ιΖ3 (2)

2_ 3_ => Zz -zιz3 =>zz - ΖιΖzΖ3 Ομοίως z ι 3=z33=z ιzzz3 Επομένως Ιz ι l 3= lzz i 3=Ιz3 1 3 οπότε Ιz ι Ι= Ιzz Ι= Ιz3 Ι

ii) Είναι Ιz ι-zz l2=(z ι-Zz) (z, - z2 ) = = lz , 1 2 + lz2 1 2 - z,z2 - z2z, =

( Ι ) 2 2 _ _ = lz, I + l z2 1 + ( z2 + z3 ) z2 + ( z, + z3 ) Ζι = = 4 lz , l 2 + z3 {z , + z2 ) = 4 lzι l 2 - l z3 1 2 = 3 lz , l 2

(αφού l z , l = Ιz2 Ι = l z3 1 ) . Άρα l z , - z2 1 = � lz, l , ομοίως

lz2 - z3 1 = Ιz ι-z3 Ι=� Izι l

Αφού lz, I = l z2 1 = l z3 1 = ρ , οι εικόνες Α, Β, Γ των z1 , z2 , z3 ανήκουν στον κύκλο (0, ρ), οπότε :

z1 + z2 = -z3 �ΟΑ+ΟΒ = -ΌΓ� -2 -2 -2 �ΟΑ +ΟΒ + 20Α · ΟΒ =ΟΓ �

ΕΥΚΛΕΙΔΗΣ Β ' 65 τ. l/56

Page 59: Ευκλειδης Β 65

Μαθηματικά για την Γ Λυκείου

Λ Λ 1 � ρ2 + ρ2 + 2ρ2συνΑΟΒ = p2 � συνΑΟΒ = -- � 2

� ΑΟΒ = 1 20° . 'Ομοια ΒΟΓ = ΓΟΑ = 1 20° , οπότε lz, - z2 1 = lz2 - z3 1 = lz3 -z, l = � = ρJ3 = lz, ΙJ3 .

β) Από z2+ν2+ω2= zν+zω+νω προκύπτει ότι: 2z2+2ν2+2ω2-2zν-2zω-2νω=Ο οπότε,

(z-ν)2+(ν-ω )2+(z-ω )2=0 με (z-ν)+(ν-ω)+(z-ω)=Ο, δηλαδή αν θέσουμε z 1=z-ν, z2= ν-ω και z3=z-ω ισχύουν οι ( 1 ), (2) οπότε σύμφωνα με το ί) Ιz-ν Ι=Ιν-ωΙ=Ιω-z Ι , που σημαίνει ότι το τρίγωνο Ρ 1 , Ρ2, Ρ3 είναι ισό­πλευρο .

Να κυκλώσετε το Σωστό (Σ) ή το Λάθος (Λ) για κάθε μια από τις παρακάτω προτάσεις :

1 . Για κάθε α , β Ε C αν z=α+βί τότε Re(z)=α και Im(z)=β Σ Λ

2. Για κάθε z1 , z2 E C ισχύει Re(z1z2 ) = Re (z,z2 ) Σ Λ

3. Για κάθε x, y Ε C αν z=x+yi τότε z = χ - yi Σ Λ

4. Για κάθε z Ε C ο iz είναι φανταστικός αν και μόνο αν z = z Σ Λ

5. Για κάθε z1 , z2 E C ισχύει z;� +Z,Zz =2Re(Z,Zz ) Σ Λ

6. Ο αριθμός (2+3ί) 1 0+(3+2ί) 1 0 είναι πραγματικός Σ Λ

7. Για κάθε z Ε C ισχύει ( z - z)20 1 0 � Ο Σ Λ

8. Η εξίσωση z2+βz+ 1 =0, z Ε C αποκλείεται να έχει ρίζα την z 1= 1+i για κάποια τιμή του β στο JR Σ Λ

9. Για κάθε z Ε C , αν z2 = z2 τότε ο z είναι φα-νταστικός Σ Λ

10. Για κάθε α, β Ε C αν z=α+βί τότε

Σ Λ

1 1 . Αν Α(ί) και B( l ) τότε (ΑΒ)=2 Σ Λ 12. Για κάθε z1 , z2 Ε C ισχύει

Ιz ι-i i+ Ιz2+i i=O<::> z ι=i και z2=-1 Σ Λ

13. Για κάθε z1 , z2 E C με Ιz ι Ι= 1 , lz2 l = 3 ισχύει minlz ι-z2 1=2 Σ Λ

14. Ισχύει lz l2 = ( -z)

2, για κάθε z Ε C Σ Λ

15. Ισχύει l z , z2 1 = 1-z, z2 1 για κάθε z1 , z2 Ε C Σ Λ

16. Για κάθε z Ε C * αν Ιz-i i= Ιz+i l τότε z φανταστικός Σ Λ

17. Για κάθε z Ε C αν Ιz-5i i= Ιz-i l τότε η ελάχιστη τιμή του Ιz l είναι 4 Σ Λ

18. Η εξίσωση l2z+i 1=1 3+4i l παριστάνει κύκλο

Σ Λ 19. Η μέγιστη απόσταση δύο σημείων Μι (z 1 ) και

M2(z2) της καμπύλης Ιz-5 Ι+Ιz +5 1= 12 είναι 1 2 Σ Λ

20. Η εξίσωση Ιz-iHz +i l=2 παριστάνει ένα κλάδο υπερβολής Σ Λ

1 . Να βρείτε τον γεωμετρικό τόπο των εικόνων του μιγαδικού z για τον οποίο ισχύει:

α) z + z = (z - z)2

β) ω = -1 -2 E JR 1 + z

γ) Ιz-i i+ Ιz +i 1=4 2 . Να βρεθεί ο μικρότερος φυσικός αριθμός ν� 1

ώστε να ισχύουν: α) ( l +i)v=( l-i)v β) ί i2} . . . . ίν= 1 (χωριστά σε κάθε περίπτωση)

3 . Θεωρούμε τη συνάρτηση f(z)=iz+ 1 όπου zEC α) Να υπολογίσετε τον z1 Ε C έτσι, ώστε f(z ι )=z ι β) Να βρείτε το σύνολο των εικόνων M(z) ώ­στε lf(z) I= Ιz l

4 . Αν για τους μιγαδικούς z 1 , z2 (z 1 , z2:;t0) ισχύει

ιz ,+ ίz, ι= ιz , ι+ ιz, ι να αποδείξετε ότι ( :: )' < ο . 5 . Δίνεται η εξίσωση z2 -z+ 1 =Ο ( z Ε C ) και z 1 , z2

οι ρίζες της

) Ν λο , θ , z 1 + z2 - ί α α υπο γισετε τον αρι μο ω = . . ιz1 z2 - 1

β) Αν Α, Β, Ρ οι εικόνες των z 1 , z2 και ω αντί­στοιχα, να αποδείξετε ότι το τρίγωνο ΡΑΒ εί­ναι ισοσκελές

γ) Να αποδείξετε ότι ( z:0 + z�0 ) (� + �) :::::; 2 . z1 z2

6 . Για τον μιγαδικό z ισχύει 2 lz i= Ιz-3 1 ( 1 ) α) Να βρείτε τον γεωμετρικό τόπο των εικό­νων Μ (z) .

β) Α z - 3 δ 'ξ ' ' ν ω = -- , να απο ει ετε οτι οι εικονες z + 1

του ω κινούνται σε κύκλο.

( ο )2009 2 + 3i ' 7 . Αν 1 + ιz = � �. , z Ε C . Να αποδει-

ν 5 + 2ν 2ι ξετε ότι: α) οι εικόνες Μ (z) ανήκουν σε κύκλο c β) ο z δεν είναι πραγματικός γ) η ελάχιστη τιμή του Ιν-ωl όπου Ρ(ν) ανήκει στον κύκλο c και Ιω-2 Ι=Ιω-41 , ω ε C είναι 2 .

8 . Αν z E C * με z + _!. = -1 ( 1 ) . Να αποδείξετε z

ότι: α)z:;t 1 β) z3= 1 γ) Ιz + 1 1= Ιz l= 1 καθώς και το αντίστροφο, δηλαδή αν Ιz + 1 Ι= Ιz Ι= Ι τότε ι­σχύει η ( 1 )

ΕΥΚΛΕΙΔΗΣ Β ' 65 τ. l/57

Page 60: Ευκλειδης Β 65

Μαθηματικά για την Γ Λυκείου

so Ι ι οο Ι δ) z + 5Ο = z + 100 . z z 9. Έστω z1 , z2 μιγαδικοί α) Αν l z ι + z2 1 = 1 z1 - z2 1 να αποδείξετε ότι

z 1z2 + z2z1 = 0 . β) Α ν ( z1 + z2 )

20 = ( z 1 - z2 γο , να αποδείξετε ότι z1 z2 φανταστικος.

1 0 . Θεωρούμε τους μιγαδικούς z, ω με ω=2zi. Αν Ρ, Μ είναι οι εικόνες των z και ω αντίστοιχα, να αποδείξετε ότι: α) Οι εικόνες P(z) κινούνται σε κύκλο, όταν

οι εικόνες Μ( ω) κινούνται στον κύκλο (0,4) .

β) Το τρίγωνο ΟΡΜ είναι ορθογώνιο .

Ι .Λ, 2 .Σ, 3 .Λ, 4.Σ, 5 .Σ, 6.Λ, 7 .Λ, 8 .Σ, 9 .Λ, Ι Ο .Λ, Ι Ι .Λ, Ι 2 .Σ, 1 3 .Σ, Ι 4.Λ, Ι 5 .Σ, Ι 6 .Λ, Ι 7 .Λ, Ι 8 .Σ, Ι 9 .Σ, 20.Λ

1. Να βρείτε το πεδίο ορισμού των συναρτή­σεων:

i) r (x) - x + S -

χ3 - 3χ2 + 2χ

( ) χ2 - 2χ - 3 ii) f χ =

χ2 + χ

iii) r (x) = � + h χ - 2 9 - χ2

ΪV) f (x) = logχ+l ( Χ2 - 4)

v) r (x) - ημχ -2ημχ - συν2χ - 2

i) Για να ορίζεται η f(x) πρέπει και αρκεί χ3-3χ2+2χ*Ο ( Ι )

Έχουμε: ( l )<=:>x.(x2-3x+2)*0<=>x� {O, I ,2 } . Ε­πομένως το πεδίο ορισμού της f είναι

Ar = ( -οο, Ο) υ ( Ο, Ι ) υ ( Ι , 2 ) υ ( 2, +οο) ii) Για να ορίζεται η f(x) πρέπει και αρκεί

χ 2 - 2χ - 3 x2 + x :;t: O ( Ι ) και 2 � Ο (2) χ + χ

Έχουμε: ( Ι ) <=> χ :;t: Ο και χ :;t: -Ι . Τότε (2 ) <=> (χ2 - 2χ - 3) (χ 2 + χ ) � Ο <=:>

,. ".- ι •c: ' ]j ·-�; ; : •: ' "' . ·:\ ) (η .. 1 . α) Παραβολή y2=-I /2x β) Υπερβολή y2-x2= I

με εξαίρεση τα (Ο, Ι ) και (0,- Ι ) γ) Έλλειψη 2 2

c : � + L = I . 3 4

2. α) ν=4, β) ν=7 3. α) z1 = � + � i , β) η ευθεία y=� . 4. Είναι Ιz ι+ izz i=Ιz ι l+ l izz l . Αν Μι (z ι ) και Mz(izz)

τότε ΟΜ ι /' /' ΟΜ2 .

5. α) ω=-Ι β) +σ = z11 0 Ζ ι

6. α) Ιz+ 1 1=2 β) Ιω-Ι Ι=2 7. α) Ι I +izl= l 8. γ) από Ιz+ Ι Ι= Ι προκύπτει z + z = - Ι δ) είναι

zso=(z3) I 6 .z2=z2 9. α) ω = . . . = ω β) τριγωνική ανισότητα. 10. α) lz l = 2

ί ! ·i I ,, _,__ 1 :.._

β) Εφαρμόζεται το πυθαγόρειο θεώρημα.

Λεωνίδας Μαυρογιαννάκης

<=> χ ( χ + Ι )2 ( χ - 3) � Ο <=> <=> χ ε ( -οο, - Ι ) υ (- Ι, Ο) υ [ 3, +οο)

και τελικά το πεδίο ορισμού της f είναι Ar = ( -οο,- Ι ) υ ( - 1, 0) υ [3, +οο) .

iii) Για να ορίζεται η f(x) πρέπει και αρκεί χ-2�0 και .Jx - 2 :;t: O και 9 - χ2 � Ο και .J9 - x2 :;t: O ή ισοδύναμα έχουμε :

:=�:>00 } � :;<

2χ < 3} � 2 < χ < 3

δηλαδή το πεδίο ορισμού της f είναι:ΑF(2,3) iν) Για να ορίζεται η f(x) πρέπει και αρκεί

χ2 -4>0 και χ+ Ι >Ο και χ + Ι* Ι . Έχουμε επομένως: χ 2 - 4 > ο} χ < -2 ή χ > 2} χ + Ι > Ο <=> χ > - Ι <=:> χ > 2 , x + l :;t: l x :;t: O οπότε το πεδίο ορισμού της f είναι:ΑF(2,+οο)

ν) Για να ορίζεται η f(x) πρέπει και αρκεί 2ημχ--συν2χ-2*0

Λύνουμε την αντίστοιχη εξίσωση 2ημχ--συν2χ-2=0<=:> 2ημχ-( Ι-ημ2χ)-2=0<=:>

ημ2χ+2ημχ-3=0 <=> 2 <=> ημχ = y } y + 2y - 3 = 0

ΕΥΚΛΕΙΔΗΣ Β ' 65 τ. l/58

Page 61: Ευκλειδης Β 65

Μαθηματικά για την Γ Λυκείου

ημχ = Υ } <=> , <=> ημχ = Ι . y = -3 η y = l Επομένως η λύση της εξίσωσης είναι

χ = 2κπ + 2: , κ ε Ζ και το πεδίο ορισμού της f εί-2 ναι: Ar = {χ ε ΙR / χ :;t: 2κπ + � ' κ ε z} . 2 Δ , , r ( )

χ - 2 . ινονται οι συναρτησεις: χ = -2 - και χ - χ 2

g ( χ) = χ

2 + χ . Ν α βρεθούν οι συναρτήσεις:

χ - 4 α) f ( x) · g ( x) β) r ( x)

g(x)

Το πεδίο ορισμού της f είναι: AFIR- {0, 1 } και

της g είναι : Ag= JR- {2, -2 } α) Το πεδίο ορισμού της h(x)=f(x) .g(x) είναι:

Ah = Ar n Ag =JR-{-2, Ο, Ι , 2 } και

( χ -2 x ( x + l) x + l h

χ ) = χ ( χ - 1 ) . ( χ - 2) ( χ + 2) ( x + l) ( x + 2)

β) Το πεδίο ορισμού της φ( χ)= :�: � είναι: Αφ= {χ εΑh I g(x):;t:O}=JR-{-2, -1 , Ο, 1 , 2 } και

φ χ ) = χ - 2 . ( χ - 2) ( χ + 2) = ( χ - 2)2 ( χ + 2) ( χ ( χ - 1 ) x (x + l ) x2 ( x - l ) ( x + l ) "

3. Να βρεθούν τα παρακάτω όρια: ί) lim χ3 - l

χ -+ \ χ3 - 4χ + 3 χ3 + 2χ2 - 5χ - 6

ii) lim --"72---•--+-3 χ - 9

ίίί) lim �- 3 χ--+2 χ - χ - 2

. )

1 . .Jx + 3 - 2 ιν Iffi --==-•--+t .J2 - χ - χ

) I . 2ημχ - J2 v ιm ---,.-----"'------"7" <--+� 2 ( η μ 2χ - συν2χ) , Ί. �,) · ·. ) ·.: ' Ι

. χ3 - Ι . ( x - l ) ( x 2 + x + l) i) lιm = lιm =

χ-->Ι χ3 - 4χ + 3 χ-->Ι ( x - l ) ( x 2 + χ - 3)

= lim χ 2 + χ + 1 = -3 Χ-->1 Χ2 + Χ - 3

. χ3 + 2χ2 -5χ- 6 . ( χ + 3) (χ2 - χ - 2) ii) lιm = lιm = χ-+-3 χ2 - 9 χ-+-3 (χ + 3) (χ -3)

. χ2 - χ - 2 ( -3)2 + 3 - 2 10 5 lιm =-=--χ-+-3 χ - 3 -3 -3 -{j 3 . �χ2 + 5 -3 . (�χ2 + 5 -3) (�χ2 + 5 + 3)

iii) lιm 2 = lιm....:...._ __ -.:...,.;'-----� χ->2 χ - χ - 2 χ->2 ( χ2 - χ - 2)(�χ2 + 5 + 3)

lim (χ-2)(χ+2) lim χ+2 = χ--->2 ( χ-2)( χ+Ι){ �χ2 +5 +3) χ--->2 ( χ+Ι){ �χ2 +5 +3) 2 + 2 4 2

(2 + 1) { 3 + 3 ) =18 - 9 . . {�χ+3-2) . {�χ+3-2){�χ+3+2) (�2-χ+χ) ιv) lim =lim =

χ-->1 �2-χ-χ χ-->1 (�2-χ-χ) {�2-χ +χ) {�χ+3+2)

lim (χ+3-4)(�+χ) lim (χ-Ι) {�+χ) χ--.ι ( 2-χ-χ2) ( �χ+3+2) χ--.ι -{ χ+2)(χ-Ι) { �χ+3+2)

=-lim{Γx+3-2){�+x) lim (χ-Ι) {�+χ) χ--.ι ( 2-χ-χ2) ( �χ+3+2) χ--.ι -{χ+2)(χ-Ι) { �χ+3+2)

=-lim �+χ 1+1 =__! x-->l (�x+3+2) (x+2) 4· 3 6

v) lim 2ημχ -J2 = lim 2ημχ - J2 = χ->� 2 ( ημ2χ - συν2χ ) χ->� 4ημ2χ - 2

= lim 2ημχ -J2 = lim 1 = Η� ( 2ημχ - .Ji)( 2ημχ + .Ji) Η� 2ημχ + J2 Ι Ι Ι J2

2ημ� + J2 = J2 + .Ji

= 2.Ji = 4

4

,C ' x:;t: !2i-15x+28

4 4. Δίνεται η <ΠJνάρτηση f(x)- �· ,_:!6

Να εξετάσετε εάν είναι συνεχής στο χ0=4.

Για να είναι η f συνεχής στο χ0=4 πρέπει και αρκεί lim f ( χ ) = f ( 4) . χ--+4

Ε, 1 . f ( ) 1 . 2χ2 - Ι 5χ + 28 ιναι ιm χ = ιm 2 = χ--+4 χ--+4 χ - 1 6 2 ( χ - 4 ) ( χ - 2)

= l im 2 = l im l x - 7 = .!_ = f ( 4 )

χ --> 4 ( χ - 4 ) ( χ + 4 ) χ --> 4 χ + 4 8 Επομένως είναι συνεχής στο χ0=4.

ΕΥΚΛΕΙΔΗΣ Β ' 65 τ. l/59

Page 62: Ευκλειδης Β 65

Μαθηματικά για την Γ Λυκείου

5. Δίνεται η συνάρτηση {κχ2 + λχ - 8 2 f { x) = χ - 2 , x :;t:

6, Χ = 2 Να βρείτε τις τιμές των κ και λ έτσι ώστε

η f να είναι συνεχής στο χ0=2.

Έστω ότι είναι η f συνεχής στο χ0=2 . Τότε lim f ( χ ) = f ( 2 ) ( 1 ) Έχουμε f(2)=6 και για χ * 2 Χ-->2 έχουμε κχ2 + λχ - 8 = ( χ - 2 ) f ( χ ) , επομένως lim(κx2 + λχ - 8) = lim( χ - 2) 1imf( χ) => χ---+2 χ�2 χ�2 4κ + 2λ- 8 = 0 · 6 => 2(2κ + λ-4) = 0 => λ = 4- 2κ (2) . Αντικαθιστώντας το λ, στην f(x) για χ * 2 έχουμε : f ( x ) = κχ2 + (4 - 2κ) χ - 8 = κχ2 + 4χ - 2κχ - 8 =

χ - 2 χ - 2 κχ ( χ - 2 ) + 4 ( χ - 2 ) = = κχ + 4 (3) , οπότε χ - 2

lim f ( x ) = lim ( κx + 4) = 2κ + 4 και χ-->2 χ-->2 ( 1 ) => 2κ + 4 = 6 => κ = 1 .

Επομένως (2 ) => λ = 2 . Για κ= 1 και λ=2 εύκολα διαπιστώνουμε ότι ι­

σχύει η ( 1 ) . Άρα: η f(x) είναι συνεχής στο Χ 0 = 2 όταν και

μόνον: κ = 1 και λ = 2 . 6. Να βρείτε την παράγωνο των συναρτήσεων:

ί) f { x) = e' · συνχ ίί) f { x) = ι:2χ

ίίί) f { χ) = � χ3 + χ2 όταν χ Ε { -1, Ο) u {Ο + οο) ίν) r ( χ) = ( χ2 + ιη χΥ ν) r ( x) = eσ"'2'

νί) f { x) = ln (3x2 + 2) νίί) f { x) = 31

2 χ - χ

ί) f ι ( χ ) = ( ex · συνχ ) ι = ( ex ) ι συνχ + ex ( συνχ ) ι = = ex . συνχ - ex . ημχ = ex ( συνχ - ημχ ) . ίί) f ι ( χ ) = ( 1::)

Ι 2 - χ - ln x · 2χ χ χ4

ίίί) f'( χ) = ( �χ3 +χ2 )

( ln Χ ) ι Χ 2 - lhx ( Χ 2 ) ι ( χ2 )2

χ - 2χ · In x 1 - 2 ln x χ4

1 ' ( χ3 +χ2 ) 2�χ3 +χ2 χ3 3χ2 +2χ 2�χ3 +χ2

ίν) fι ( χ) =[( χ2 +lnx)3 ] =3( χ2 +lnx)2 ( χ2 +lnx)' = = 3 ( x2 + ln x )2 (2x + � ) .

ν) fι (x) =[eσw'x] =eσνν'χ (συΥχ) ' =eσνν'χ 2συνχ(συνχ) ' = eσuv' x 2συνχ ( -ημχ ) = -2ημχ . συνχ . eσυv' χ = = -ημ ( 2χ ) eσuv' x

νί) f ι(x) =[In(3x2 +2)]' =-21

- (3χ2 +2) ' =+ 3χ +2 3χ +2

νίί) f'( χ) =(�)' -1 2 ( xJ -χ2 )' = χ -χ (χ) -χ2 )

3χ2 -2χ 2-3χ χ4 ( χ- Ι)2 χ3 ( χ - Ι)2 .

viii) fι( χ) = [ ημ2 ( χ2 -π)] = 2ημ( χ2 -π)[ ημ( χ2 -π)] = = 2ημ( χ2 -π)συν( χ2 -π) 2χ =4χημ( χ2 -π)συν( χ2 -π) = = 2χημ(2χ2 -2π) = 2χ · ημ2χ2 •

7. Ένα βότσαλο που πέφτει σε μια λίμνη προ­καλεί κύμα σχήματος κύκλου, που η ακτίνα του αυξάνει με ταχύτητα 0,5 m/s. Την χρο­νική στιγμή t0 που ο κύκλος έχει ακτίνα 2 m να βρείτε τον ρυθμό μεταβολής του εμβαδού του κύκλου.

Έστω r(t) η ακτίνα του κύκλου, οπότε η ταχύ­τητα είναι r ' (t)=0,5rn/s. Η επιφάνεια του κύκλου έχει εμβαδόν E(t)=π?(t) . Ο ρυθμός μεταβολής ε­πομένως είναι E '(t)=[πr2(t)] '=2 πr(t)r ' (t) . Άρα την χρονική στιγμή t0 ο ρυθμός μεταβολής του εμβα­δού του κύκλου θα είναι:

mz Ε ι ( to ) = 2πr ( to ) r ' ( to ) = 2π2m0, 5m / s = 2π­s

8. Δίνεται η συνάρτηση f(x) =(2α-1)r -6x+3 . Να βρείτε την τιμή του α Ε JR έτσι ώστε η εφαπτομένη της Cr στη θέση χ0=-1 να σχη­ματίζει με τον οριζόντιο άξονα χ 'χ γωνία

π ω = - .

4

Έχουμε f ι ( χ ) = 2 ( 2α - Ι ) χ - 6 επομένως f ι ( - Ι ) = 2 ( 2α - 1 ) ( - 1 ) - 6 = -4α - 4 (2)

Επομένως πρέπει και αρκεί: f ι ( -Ι) = εφ� ( 1 ) 4 ( 2 ) π Άρα ( Ι )<::::>- 4α - 4 = εφ- <::::> -4α - 4 = 1 <::::> 4

ΕΥΚΛΕΙΔΗΣ Β ' 65 τ. l /60

Page 63: Ευκλειδης Β 65

Μαθηματικά για την Γ Λυκείου

5 <::::> -4α = 5 <::::> α = -- . 4 2

9. Δίνονται οι συναρτήσεις f ( χ) = -- και χ - 1

g ( x) = -2x2 - 6χ + 4 . i) Να βρείτε την εξίσωση της εφαπτομένης

της γραφικής παραστασης της f στο ση­μείο με τετμημένη χ0 = 2.

ii) Ν α δείξετε ότι η ευθεία αυτή εφάπτεται και στην γραφική παράσταση της g και να βρείτε το σημείο επαφής.

\ ;li (ϊ ! ] Το πεδίο ορισμού της f είναι Ar = IR - { 1 } και

της g είναι Ασ = IR . . �

i) Είναι f ' ( χ ) = (-2-J ' = 2 , , για κάθε χ - 1 ( χ - 1) -

χ =1- 1 και ο συντελεστής διεύθυνσης της εφαπτομέ-νης στο σημείο Μ(2, 2) είναι: λ = f ' ( 2) = -� = -2 .

1 Η εφαπτομένη θα έχει εξίσωση : (ε ) : y = λχ + β <=> (ε ) : y = -2χ + β , οπότε

Μ Ε ( ε ) <::::> 2 = -2 · 2 + β <=> β = 2 + 4 <=> β = 6 . Επομένως η ζητούμενη ευθεία έχει εξίσωση

y = -2x + 6 (ε) ii) Η ευθεία (ε) θα δείξουμε ότι εφάπτεται στην Cg. Οι συντεταγμένες των κοινών σημείων των (ε)

y=-2x +6 }( 1) και Cg επαληθεύουν το σύστημα: , ( ) y=-2x' -6χ +4 2

χ ' -8 -6

6 χ

y ' ( Ι )

Έχουμε: ( 2 )<=>- 2χ2 - 6χ + 4 = -2χ + 6 <::::> <=> -2χ2 - 6χ + 2χ + 4 - 6 = 0 <::> -2χ2 - 4χ - 2 = 0 <=> <::::> -2 ( χ 2 + 2χ + I ) = Ο <::::> -2 ( χ + I )2 = Ο <::::> χ = - I . Άρα ( 1 ) <::::> y = 8 οπότε μοναδικό κοινό σημείο των (ε) και Cg είναι το Ν(- 1 , 8) . Εξάλλου

g ' ( x ) = -4x - 6 και g ' (- 1 ) = -2 επομένως η εφα­πτομένη (δ) την Cg στο Ν είναι (δ ) : y = -2χ + κ με 8 = -2 (- 1 ) + κ δηλαδή κ=6. Άρα (δ) : y = -2χ + 6 οπότε (δ ) = (ε ) . 1 Ο. Δίνεται η συνάρτηση f ( χ) = χ3 - 3χ - 2. Ν α

βρείτε την εξίσωση της εφαπτομένης της γραφικής παράστασης της f που περνά από το σημείο Α(-1 , 5).

\ 1'.1 '(} ] ' Το πεδίο ορισμού της f είναι όλο το IR και

f ' ( x ) = 3x 2 - 3 . Έστω y = λχ + β η εξίσωση της εφαπτομένης της Cr που περνά από το Α και M (x0 , f ( xo ) ) το σημείο επαφής. Τότε λ = f ' ( Χ0 ) = 3χ� - 3 και η εξίσωση γίνεται: (ε ) : y = ( 3χ� - 3) χ + β εξάλλου : Α Ε ( ε ) <=> 5 = ( 3χ� - 3) ( -1) + β <::> 5 = -3χ� + 3 + β <::> β = 3χ� + 2 οπότε : (ε) : y = (3χ� -3) χ + 3χ� + 2 (ε)

Επίσης Μ Ε ( ε ) <=> χ� - 3χο - 2 = = ( 3χ� -3) χο + 3Χ� + 2 <::::> 2Χ� + 3χ� + 4 = Ο <::::>

<=> (χο + 2) ( 2χ� - Χ0 + 2) = 0 <::::> Χ0 = -2 Άρα το σημείο επαφής είναι Μ(-2, -4) και η

ζητούμενη εξίσωση : y=9x+ 1 4 .

1 1 . Δίνεται η συνάρτηση f ( χ) = ιη ( 1 - χ: J 1 + χ α. Να βρείτε το πεδίο ορισμού της β. Να βρεθεί η παράγωγος f'(x) γ. Να μελετηθεί ως προς την μονοτονία και

τα ακρότατα . . '\

Για να ορίζεται η f(x) πρέπει και αρκεί 1 - χ2 1 + χ2 =1- Ο και --2 > Ο ( 1 ) . 1 + χ

Αφού 1 + χ2 > Ο Vx Ε IR έχουμε: ( 1 ) <=> 1 - χ2 > 0 <::> - 1 < χ < 1 .

Άρα το πεδίο ορισμού της f είναι το A r = (-1 , 1 ) . Για χ Ε ( -1 , 1 ) έχουμε

f ' (ψH :::: JJ � :::: ( : ::: ) � 1 + χ 2 -2x ( l + x 2 ) - ( l - x 2 ) 2x = 1 - χ 2

. ( 1 + χ 2 )2

=

ΕΥΚΛΕΙΔΗΣ Β ' 65 τ.l/61

Page 64: Ευκλειδης Β 65

Μαθηματικά για την Γ Λυκείου

-2χ _ 2χ3 _ 2χ + 2χ3 -4χ 13. Η ποσότητα σε εκατομμύρια κυβικά μέτρα

( 1 _ χ2 ) ( 1 + χ2 )2 =

1 _ χ4 · 1 + χ2 · νερού, που περιέχονται σε ένα φράγμα από το οποίο υδρεύεται μια πόλη, κατά την διάρκεια ενός έτους, δίνεται από την σχέση :

γ. -4χ 4χ

f ' ( x ) > 0 <=> --4 > 0 <=> --4 < 0 <=> 1 - χ 1 - χ <=> 4χ ( 1 - χ 2 ) < Ο <=> - 1 < χ < Ο και αντίστοιχα f ' ( χ ) < ο <=> ο < χ < 1 .

χ - 1 ο

f'(x) + ο

f(x) l Μεγ.

Άρα η f(x) είναι γνησίως αύξουσα στο ( - 1 , Ο] και γνησίως φθίνουσα στο [0, 1 ) . Παρουσιάζει μέ-γιστο στο χο=Ο που είναι το: f (Ο) = ln 1 = Ο .

12 . Δίνεται η συνάρτηση ι

f (χ) = 2χ2 - 3 Ιη χ + - , χ > Ο χ

α) Να μελετηθεί ως προς την μονοτονία και τα ακρότατα.

zx>-r!-3 β) Να δείξετε ότι r �e χ για κάθε χ>Ο.

α) Για χ>Ο έχουμε:

f ' ( x ) = 4x -� - -1

= 4χ3 - 3χ - 1 =

χ χ 2 χ2 ( χ - 1 ) (4χ2 + 4χ + 1 ) ( χ - 1 ) (2χ + 1 )2 ....:....__�---::-------'- =

χ 2 χ2 Η μονοτονία φαίνεται από τον πίνακα μεταβο­

λών της f: . χ ο +οο

f'(x) ο +

f(x) � Ελαχ. � Στο διάστημα (0, 1 ] είναι γνησίως φθίνουσα

και στο [ 1 , +οο) είναι γνησίως αύξουσα. Παρουσιάζει ελάχιστο στο Χ0 = 1 που είναι

f ( 1 ) = 3 . β ) Από (α) έχουμε για κάθε χ>Ο,

1 f ( x ) � 3 � 2x2 - 3 ln x + - � 3 � χ

1 2 χ2 +_!_-3 � 3 ln x ::; 2x2 + - - 3 � ln x3 ::; In e χ � χ

2 χ 2 +_!_-3 � χ3 ::; e χ

( ) 8 , , , Ρ t = 2 , t μηνες απο την αρχη

t - 6t + 13 του έτους. α) Να βρείτε την ποσότητα του νερού στο

τέλος του Ιανουαρίου. β) Να βρείτε πότε το φράγμα περιέχει την

μέγιστη ποσότητα και πόση είναι αυτή. γ) Σε περιόδους ανομβρίας η υδροδότηση

της πόλης ενισχύεται από μια λίμνη. Δε­δομένου ότι η άντληση από το φράγμα προς την πόλη δεν μπορεί να γίνει όταν η στάθμη κατέβει τόσο ώστε η ποσότη­τα του νερού στο φράγμα να γίνει μι­κρότερη από 400.000 κυβικά μέτρα, να εξετάσετε αν η πόλη θα χρειαστεί την συγκεκριμένη χρονιά νερό από τη λίμνη. Ποιος είναι ο πρώτος μήνας που η πόλη θα χρειαστεί νερό από τη λίμνη.

Είναι Ο ::; t ::; 1 2 α) Για t= l έχουμε Ρ ( 1 ) = 2

8 = 1εκ. m3 1 - 6 · 1 + 1 3

8( t2 -6t + 13) ' 16 ( t -3) β) Ρ ' ( t) = 2 2 , Ο ::; t ::; 1 2 .

( t2 - 6t + 1 3) ( t2 - 6t + 13) 1 6 ( t - 3) P ' (t) > O <=>= 2 > 0<=> t - 3 < 0 <=> t < 3 .

( t2 - 6t + 13) Για t=3 η συνάρτηση P(t) παίρνει την μέγιστη τιμή.

P' (t)

P(t)

ο 3 1 2 + ο

Μεγ.

Επομένως το φράγμα περιέχει την μέγιστη πο­σότητα νερού στο τέλος Μαρτίου και είναι Ρ (3 ) = 2εκ · m3 . γ) Αφού η P(t) είναι γνησίως αύξουσα στο [0, 3 ]

και γνησίως φθίνουσα στο [3 , +οο) με Ρ(3)>0,4 θα υπάρχει το πολύ μια τιμή του t στο (3 , +οο) τέτοια ώστε ρ(t)=0,4. Πράγματι:

8 2 Ρ ( t ) = Ο, 4 <=> 2 = - <=> t - 6t + 1 3 5 <=> 2t2 - 1 2t + 26 = 40 <=> t 2 - 6t - 7 = ο <=>

t = - 1 ή t = 7 <=> t = 7 αφού Ο ::; t ::; 1 2 .

Αφού η g(t)= 1n t είναι γνησίως αύξουσα στο Άρα στο τέλος Ιουλίου θα χρειαστεί η πόλη (Ο, + οο) . Η ισότητα ισχύει μόνο για χ = 1 . νερό από τη λίμνη .

ΕΥΚΛΕΙΔΗΣ Β ' 65 τ.l/62

Page 65: Ευκλειδης Β 65

Μαθηματικά για την Γ Λυκείου

14. Η πιο κατάλληλη εποχή για φύτεμα ενός αγροτικού προϊόντος είναι ο Ιανουάριος και ο Φεβρουάριος, η δε συγκομιδή γίνεται στο τέλος της Άνοιξης και αρχές του Καλοκαι­ριού. Η ποσότητα σε χιλιάδες κιλά που μπο­ρεί να παραχθεί από ένα στρέμα καλλιέργει­ας δίνεται από το τύπο

Ρ ( ) 2t , , , βδ , δ , t = - , εαν το φυτεμα γινει t ε ομα ες απο 5 την αρχή του έτους και η τιμή πώλησης είναι

τ(t) = � € / κιλό . t + 4

α) Να βρείτε το εισόδημα από ένα στρέμα ως συνάρτηση του t. β) Να βρείτε ποια βδομάδα πρέπει να φυ­τέψει ο γεωργός για να έχει το μέγιστο εισό­δημα, και ποιο θα είναι αυτό από καλλιέρ­γεια 20 στρεμμάτων; γ) Εάν επιλέξει για φύτεμα την τέταρτη εβδομάδα του Ιανουαρίου πόση απώλεια ει­σοδήματος θα έχει;

α) Έστω E(t) το εισόδημα από ένα στρέμα. Τότε 2t 20 8t ' Ε ( t ) = Ρ ( t ) · Τ ( t ) = - · -2- = -2- σε χιλιαδες 5 t + 4 t + 4

ευρώ, Ο � t � 8 .

β) Είναι E' ( t ) = (�)· t +4

(8t) ' ( t2 +4) -8t ( t2 +4) ' ( t2 +4)2

(-',, :'�γ -,-� ι-�.'� � -:.ι \_�, ' - \-1

8 ( t 2 + 4) - 8t · 2t = ----'------'--::---( t 2 + 4 )2

-8 ( t 2 - 4 ) E ' { t ) = Ο <:::> = Ο <:::> t 2 - 4 = Ο <:::> ( t2 + 4 )2 t = t 1 = -2 ή t = t2 = 2 . Δεκτή μόνο η t 2 = 2 .

Ε ' ( t) > Ο <:::> -8 ( t2 - 4) > Ο <:::> t2 - 4 < Ο <:::> Ο < t < 2 .

Αφού η συνάρτηση E(t) είναι γνησίως αύξου­σα στο [0, 2] και γνησίως φθίνουσα στο[2,8] , άρα παρουσιάζει μέγιστο στο t = 2 .

Επομένως για t = 2, δηλαδή η δεύτερη βδομά­δα το έτους είναι η πιο κατάλληλη για φύτεμα.

. t Ό 2 +οο

E' (t) : + ο

E(t) Μεγ.

Το εισόδημα από ένα στρέμμα είναι Ε (2 ) = 2 χιλιάδες ευρώ και από 20 στρέμματα 40.000€ .

' 8 · 4 32 γ) Για t = 4 εχουμε Ε {4) =

-2-=

-χιλ.

4 + 4 20 ευρώ το στρέμα και από 20 στρέματα:

20 . 32 Ι 000 = 32 .000€ . 20

Η απώλεια σ ' αυτήν την περίπτωση είναι: 40.000-32 .000=8.000€.

του Γεώργιου Σ. Τασσόπουλου

Στον απόηχο πλέον των Πανελλαδικών εξετάσεων, χωρίς να προκαλούμε αγωνία ή άγχος στους υπο­ψηφίους, θα εκφράσουμε μερικές απόψεις για τα φετινά θέματα.

Ι . Στα Μαθηματικά Γενικής Παιδείας (Θέμα Ι 0) , ενώ ζητήθηκε η παράγωγος της συνάρτησης

f (χ ) = J;. με τον περιορισμό χ>Ο και βρέθηκε f (χ ) = ΙΓ , αντιθέτως για την παράγωγο της

2ν χ f(x)=xv, νεΝ, δεν δόθηκε κανένας περιορισμός. Οι εξεταστές λοιπόν στις ενδεικτικές λύσεις τους έδωσαν την απάντηση : f(x)=v·xv- I . Όμως γίνεται

εύκολα αντιληπτό ότι για ν= Ι , ο τύπος f' (x)= I ·x0 δεν έχει νόημα όταν χ=Ο, ενώ για ν=Ο δεν ορίζεται ούτε καν η συνάρτηση f(x)=x0, όταν χ=Ο. Η σωστή απάντηση λοιπόν θα έπρεπε να είναι:

ν> Ι � f' ( Χ ) = ν · Χ ν-ι

}

ν= Ι � f { χ ) = χ � f' { χ ) = Ι ή να δοθεί ο περιορισμός χ*Ο

ν=Ο και χ :;t: Ο � f ( χ ) = Ι � f'{ χ ) = Ο

ΕΥΚΛΕΙΔΗΣ Β ' 65 τ. l/63

Page 66: Ευκλειδης Β 65

Μαθηματικά για την Γ Λυκείου

Δεν έκανε άραγε εντύπωση στους εξεταστές για ποιο λόγο το σχολικό βιβλίο εξετάζει χωριστά τις παραγώγους των συναρτήσεων f(x)=C, και f(x)=x και χωριστά την παράγωγο της f(x)=xv, ν εΝ με ν> 1 ( ;)

2 . Στα θέματα πολλαπλής επιλογής ή Σωστού Λάθους θα πρέπει οι εξεταστές να είναι πιο προσεκτικοί, γιατί δημιουργούνται παρανοήσεις. Έχουμε και σε προηγούμενο άρθρο μας αναφέρει (Ευκλείδης Β τεύχος 6 1 ) τη σύγχυση που προκα­λούν οι εξεταστές με το να ζητούν να χαρακτηριστούν ως αληθείς ή ψευδείς εκφράσεις που δεν είναι προτάσεις, αλλά προτασιακοί τύποι. Για παράδειγμα: «Αν xεiR, τότε χ-5>0» ή το φετινό 1 ο Θέμα της Θετικής - Τεχνολογικής κατεύθυν­σης: «Α ν η συνάρτηση f είναι συνεχής στο χ0 και η συνάρτηση g είναι συνεχής στο χ0, τότε η σύνθεση (τους) g ο f είναι συνεχής στο Χο». Οι εκφράσεις αυτές είναι προτασιακοί τύποι που εξαρτώνται ο μεν πρώτος από τη μεταβλητή χ για την οποία απλώς δηλώνεται ότι είναι πραγματικός αριθμός, ενώ ο δεύτερος από τις μεταβλητές g, f, χ0 για τις οποίες απλά δηλώνεται ότι οι g, f είναι συναρτήσεις και το χ0 (εννοείται) ότι είναι πραγμα­τικός αριθμός. Έτσι λοιπόν ο μεν πρώτος προτασιακός τύπος για άλλες τιμές του χ γίνεται αληθής πρόταση (όπως για χ=6) και για άλλες τιμές του χ γίνεται ψευδής πρόταση (όπως για χ=4). Ομοίως ο δεύτερος προτασιακός τύπος για άλλες τριάδες (g, f, χ0) γίνεται αληθής πρόταση (π.χ. όταν x0=f(x0)) και για άλλες ψευδής. Όσο για τον ισχυρισμό ορισμένων, ότι σ' αυτές τις εκφράσεις εννοούμε πάντοτε, για κάθε x εiR. ή για κάθε g, f, χ0 έχουμε να παρατηρήσουμε ότι έτσι οδηγούμαστε σε παράλογα συμπεράσματα, κα­θότι θα πρέπει να χαρακτηρίσουμε ψευδή την θεωρούμενη «πρόταση» (Αν x εiR, τότε χ-5>0) και την «πρόταση» (Αν xεiR., τότε χ-5:::;0), που κανονικά θα έπρεπε να είναι άρνηση της προηγούμενης, δη­λαδή αληθής. Μοιάζει έτσι να αποδεχόμαστε ότι είναι ψευδής και η χ-5>0 και η χ-5:::;0. Δεν μπο­ρούμε λοιπόν να διατυπώνουμε προτάσεις μόνο με τον ποσοδείκτη V(για κάθε), αλλά χρειαζόμαστε και τον ποσοδείκτη Ξ3(υπάρχει) . Το ορθό λοιπόν είναι, να πούμε για την πρώτη : «Για κάθε xεiR. ισχύ­ει χ-5>0» (ψευδής πρόταση) και για τη δεύτερη «Υπάρχει x εiR ώστε χ-5::Ξ;0» (αληθής πρόταση) ή αντιστρόφως. Η μόνη περίπτωση κατά την οποία, υπονοώντας το V(για κάθε) σε ένα προτασιακό τύπο, δεν οδηγούμαστε σε παράλογα συμπεράσματα, είναι όταν ο προτασιακός τύπος p(x) είναι καθο-

λικώς αληθής, οπότε η άρνησή ρ ( χ ) αυτού είναι καθολικώς ψευδής, επομένως πολύ μάλλον

και μερικώς ψευδής, όπως απαιτείται. π. χ. «Αν χ ε R τότε χ2+ 1 >0» (αληθής) και «Αν χ ε R, τότε χ2+ 1:::; 0». (ψευδής) ακόμη και στην περίπτωση που αντί του Ξ3(υπάρχει) εννοούμε (κα­κώς) το V(για κάθε) , και στη δεύτερη πρόταση . Δεν παρουσιάζεται λοιπόν τότε το παράδοξο να χαρακτηρίζονται ως αληθείς ή ως ψευδείς συγχρόνως και οι δυο θεωρούμενες «προτά­

σεις» χ2+ 1 >0 και χ2+ 1 :::;0 .

3 . Τέλος είναι γνωστό ότι για κάθε φθίνουσα και συνεχή συνάρτηση f: [ -1 , 1 ]� IR., το σύνολο τιμών της είναι: f( [-1 , 1 ])=[f( l ), f(- 1 )] . Αντ ' αυτού στις ενδεικτικές λύσεις του 3ου θέματος της Θετικής - Τε­χνολογικής κατεύθυνσης οι εξεταστές έγραψαν: - 1::Ξ;χ::Ξ; 1=>f( 1 )::Ξ;f(χ)::Ξ;f(-1 ) και είχαν την εντύπωση ότι εννοούσαν το ίδιο, ενώ στην ουσία αυτό που έγραψαν σημαίνει ότι: f( [-1 , 1 ] )�[f( l ) , f(- 1 )] . Έτσι παρά το ότι είχαμε f( l )<O<f( -1 ) αυτό δεν σημαίνει ότι η συνάρτηση f(x) παίρνει την τιμή Ο . Άρα δεν αποδεικνύεται έτσι το ζητούμενο, δηλαδή ότι η εξίσωση f(x)=O έχει ρίζα στο (- 1 , 1 ), οπό­τε η λύση που δόθηκε από τους εξεταστές ήταν ελλιπής. Αντιλαμβανόμαστε ότι μετά από τέτοιες διατυπώσεις λύσεων θα ήταν εντελώς άδικο να αφαιρούμε μονάδες από μαθητές οι οποίοι για να δείξουν π.χ. ότι: I z ιzz l = I z ι I I zz l έγραψαν: «( 1 )=> I z ι zz l 2= I z ι 1 2 1 z2 l 2 =>z1z2ZιZz =z1ZιZ2Zz , που ισχύει», αντί του ορθού : «Για να ισχύει η ( 1 ) αρ­κεί I z ι z2 l 2= I z ι 1 2 1 z2 l 2 , ή αρκεί z1z2ZιZz = z1ZιZ2Zz , που ισχύει», δηλαδή συμβολικά:

« ( 1) <= lz1z2 12 = lz1 1 2 · lz2 1

2 <=z1 z2ZιZz = z1Ζι · Z2Zz που ισχύει», ή έστω αντί αυτού που συνηθίζεται

(κακώς) « ( 1) <=> 1z1z2 12 = lz1 12 lz2 1

2 <=> z1z2ZιZz = z1Ζι · Z2Zz που ισχύει» (το κακώς αναφέρεται στο ότι εί­ναι περιττές - πλεονάζουσες οι συνεπαγωγές «=>» ) .

ΕΥΚΛΕΙΔΗΣ Β ' 65 τ.l/64

Page 67: Ευκλειδης Β 65

χ

f(to,Xo)

Χ ο

to

Μαθηματικά για την Γ Λυκείου

Θανάσης Κυριακόπουλος Το φετινό 4° θέμα στα Μαθηματικά Κατεύθυνσης είχε ως αφετηρία τις

χ χ

δυο αρχικές συναρτήσεις F { χ ) = Jf { t ) g ( t ) dt και G { χ ) = Jg ( t ) dt οι α α

οποίες συνθέτουν την ανισότητα Cronwall που χρησιμοποιείται ως εισα­γωγικό Λήμμα για την απόδειξη του θεωρήματος του Μονοσημάντου των λύσεων μιας Δ.Ε. (Οι f, g πληρούν τις υποθέσεις του Λήμματος καθ ' όσον είναι ορισμένες, συνεχείς και θετικές στο [α, β]=[Ο, \ ] ) . Το θέμα αυτό (ανι­σότητα Cronwall) είχε προταθεί στον «Ευκλείδης Β \> στο τεύχος 64/σ. 74 και στη στήλη «0 Ευκλείδης προτείνει . . . » (Άσκηση I 00). Με την ευ και-ρία αυτή σημειώνουμε:

Τα θεωρήματα Ύπαρξης (Peano) και μονοσημάντου των λύσεων αποτελούν τη βάση της θεωρίας των Διαφο­ρικών Εξισώσεων. Λέμε για μια Δ. Ε x '=f(t, χ) ότι ισχύει το μονοσήμαντο (των λύσεων) όταν για δεδομένη αρχική συνθήκη (to, χ0) όλες οι πιθανές λύσεις (ολοκληρωτικές καμπύλες) φ 1 , φ2, φ3 , . • . ταυτίζονται σε μια περιοχή του t0 (Βλ. Σχήμα) . Δηλαδή έχουν το ίδιο εφαπτόμενο διάνυσμα f(t0, χ0) που ταυτίζεται με το αντίστοιχο διάνυσμα του πεδίου διευθύνσεων της Δ.Ε. (Μονοσημάντου των λύσεων μιας Δ.Ε) . Έστω f : U ---* .!R, UCIR2 (υ α-

νοικτό υποσύνολο του JR2) Αν η f είναι συνεχής ως προς (t, χ) και ικανοποιεί τη συνθήκη Lipschitz (ως προς χ) στο

υ, τότε για κάθε (t0, χ0) ε υ ΥΠΆΡΧΕΙ ΑΚΡΙΒΩΣ ΜΙΑ Λ ΥΣΗ φ(t) της Δ.Ε. x '=f(t, χ) που ικανοποιεί την αρχική συνθήκη φ(tο)= Χο

"Εκ παραδρομής στο Άρθρο: 'ΈΠΑΝΑΛΗΠΠΚΑ ΘΕΜΑ ΤΑ ΓΕΝΙΚΗΣ ΠΑΙΔΕΙΑΣ ΤΗΣ Γ Λ ΥΚΕΙ­

Ο Υ"στην άσκηση 6 σελίδα 71, δόθηκε και στην εκφώνηση αλλά και στη λύση το γ ερώτημα το οποίο να δι­

αγραφεί αφού οι μαθητές δεν μπορούν να απαντήσουν στο ερώτημα αυτό. Η κατανομή φυσικά δεν είναι ομοιόμορφη ώστε να ισχύει κάτι τέτοιο. Προφανώς η λέξη ομοιογένεια αντί της ομοιομορφίας οδήγησε

στην λανθασμένη αυτή αντιμετώπιση. Ζητούμε συγνώμη για αυτιί την παρανόηση ".

ΜΑΘΗΜΑJΙΚΑ Α' Γυμνασίου

- 11> ΕΠΙΣΤΗΜΟΝ ΙΚΑ Για ΑΕΙ, ΤΕΙ, ΙΕΚ, ΑΣΕΠ • • 11> ΕΚΠΑΙΔΕΥτΙ ΚΑ Για Δημοτικ6, Γυμνάσιο, λύκειο, ΤΕΕ • • 11> λΟΓΟΤΕΧΝΙΑ, ΜΕΛΕτΕΣ, λΕΥΚΩΜΑΤΑ

ΜΑΘΗΜΑJΙΚΑ Β' Γ υμνnσίου

ΜΑΘΗΜΑJΙΚΑ Γ' Γυμνασίου

Αναλυτική Γεωμετρία

tlf1'ύ.t'lιιιtνnίrιnM ΜαΙJημαi:'κ6.

� ,.. .. .._'::ΑΣΕΠ

Γ ρομμι•ή Άλγεβρα

Νέες Εκδόσεις 2006 γιο το Λύκειο ----, ::�

ΜΑΘΗΜΑJΙΚΑ & ΗΟΙΧΕΙΑ HAJIHIKHI

Γ' Ενιοfου Λυκεfου Διαφορικός Λογισμός.

ΣτατισtιΚJ]. Πιθανότητες

ΜΑθΗΜΑτΙΚΑ θετικής & Τεχνολ. Κατευθ.

Γ' Ενιαίου Λυκείου Τόμος 1: Μιyαδικοi αp�θμοi.

"Ορω & συνέχεια συνάρτησης

ΜΑΘΗΜΑτΙΚΑ Θετικής & Τεχνσλ. Κατευθ.

Γ' Ενιαίου Λυκεfου Τόμος 2: Διαφορικός λογισμός.

Ολοκληρωτικός λογισμός

Εmλεyμένα θέματα από τα Μαθηματικ� Αvόλυση MAΘHMAJIKA Τόμος 1: Ιιινnplήπεις ιιιπς ιιΕ.τπβληlής

Γιu ι uu) διuγωvιuμuύ) ιuu ΑΣΕΠ Τόμος 2: Λuγιuμύ) ιιuλλών με ιuβλτ ι ιών

• ΒΙΒΛIΟΠΩΛΕΙΟ - KENTPIKH ΔΙΑΘΕΣΗ : Αρμεvοπούλου 27 • 546 35 Θεσσαλονίκη Τηλ. 2310·203.720 • Faχ 2310-21 1 .305 • e-mail: saι[email protected]

• ΒIΒΛIΟΠΩΛΕ IΟ ΑθΗΝΩΝ - "ΕΝΩΙΗ ΕΚΔΟΤΩΝ ΒιΒΛΙΟΥ θΕΣΙΑΛΟΝΙΚΗΣ• Στοά του Βιβλfου (Πεσμαζόγλου 5) · 1 05 64 ΑΘΗΝΑ • Τηλ.·Faχ 21 0·321 1 .097

• ΑΠΟΘΗΚΗ ΑΘΗΝΩΝ . ΠΩΛΗΣΗ ΧΟΝΔΡΙΚΗ: Ασκληπιού 60 - Εξάρχεια 1 1 4 71 . Αθήνα Τηλ ·Fax 21 0-381 6 650 • e-mail: athιna@zΠi.Qr

ΕΥΚΛΕΙΔΗΣ Β ' 65 τ. l/65

Page 68: Ευκλειδης Β 65

- "' '"

' � - c " ' ""

Το Βήμα του Εn�(δη π I

Η στήλη αυτή έχει ως στόχο την ανάπτυξη μαθηματικού διαλόγου. Φιλοδοξούμε να συμμετάσχουν όλοι όσοι έχουν ένα γενικότερο ενδιαφέρον για τα Μαθηματικά.

Επιμέλεια: Γιάννης Στρατής - Βαγγέλης Ευσταθίου

Γιώργου Σ. Τασσόπουλου (καθηγητή Βαρβακείου Λυκείου)

« ';4ν δέν fχεις κάνει lρωτα ποτέ σου μέ τά μαθηματικά, δέν θά μπορέσεις ν ' αποδείξεις δrι τά γραφτά σου τούς μοιάζουν»

ΟΔ ΥΣΣΕΑΣ ΕΛ ΥΤΗΣ ('Εκ του πλησίον)

Προκειμένου τα άρθρα των συναδέλφων που απευθύνονται στον Ευκλείδη Β ' να έχουν, κατά το δυνατόν, μία ομοιομορφία ως προς τις Μαθηματικές εκφράσεις και τους συμβολισμούς, χωρίς να χάνουν κάτι από το προσωπικό ύφος του συγγραφέα στις διατυπώσεις των συλλογισμών, τολμάμε να παραθέσουμε μερικές από τις παρατηρήσεις που κάναμε κατά την διάρκεια της μακρόχρονης διδασκαλίας μας.

Εναπόκειται, φυσικά, στην κρίση των συναδέλφων και στη διακριτική τους ευχέρεια η υιοθέτηση ή μή αυτών των παρατηρήσεων.

η Πεδ ίο 'J }' r,σμ.oiJ Σ'..'\''c1[c''"""1 ση ς - �ξ�σtο.'t'Ι'""Ις

Όταν θέλουμε να βρούμε το πεδίο ορισμού

μιας συνάρτησης, π.χ. της f(x) =-2- - � δεν χ- 5 ν χ- 1

λέμε (ή γράφουμε) :«πρέπει x-5:;t:O και χ - 1 > 0 » διότι έτσι αφήνουμε την αμφιβολία ότι πιθανόν αυτό να μην είναι αρκετό !

Για παράδειγμα, αν πούμε: «πρέπει χ :;t: 5 και χ :;t: 1 )) πράγματι αυτό χρειάζεται αλλά δεν είναι αρκετό.

Επίσης, δεν λέμε : «αρκεί χ � 6 )) , διότι αυτό δεν είναι αναγκαίο .

Κακώς, επίσης, λέμε: «Η f(x) ορίζεται για χ � 6 )) ή «η f(x) ορίζεται όταν χ � 6 )) διότι αυ­τό δεν σημαίνει ότι το διάστημα [ 6, +οο) είναι το πεδίο ορισμού Ar της συνάρτησης, αλλά ότι [ 6, +οο) ς Ar .

«Για να ορίζεται η f(x) πρέπει και αρκεί χ - 5 :;t: Ο και χ - 1 > Ο ))

Αναλυτικά, μάλιστα, αναφέρουμε τις σχέσεις με τη σειρά που απαιτείται για το κτίσιμο του τύ-που f(x) "'= -

2- - � . Δηλ. πρέπει και αρκεί:

χ - 5 ν χ - 1 χ - 5 :;t: Ο ( για να υπάρχει το πρώτο κλάσμα), χ - 1 � Ο ( για να υπάρχει το ριζικό) και .Jx - 1 :;t: Ο ( για να υπάρχει το δεύτερο κλάσμα).

Τελικά, είναι: Ar = (1, 5) υ (5, +οο) .

Το ίδιο, φυσικά, λέμε προκειμένου να βρούμε 2 1

το πεδίο ορισμού της εξίσωσης: -- --­

χ - 5 � -Ανάλογα, για να ορίζεται η

2χ - 1 g(x) = 1 - 1og(--)

χ - 2 θα πούμε με την ακόλουθη

σειρά: « πρέπει και αρκεί να ισχύουν οι σχέσεις: 2χ - 1 2χ - 1 χ - 2 :;t: Ο , -- > Ο , 1 - 1og(--) � Ο )) και όχι χ - 2 χ - 2

μόνο η τελευταία σχέση όπως κακώς βλέπουμε σε πολλά συγγράμματα. Κάθε μία σχέση δε από αυτές αποτελεί προϋπόθεση για την επόμενη .

Όταν φυσικά δεν μπορούμε να συναληθεύ­σουμε τις σχέσεις αυτές για να βρούμε τελικά το πεδίο ορισμού με μορφή διαστήματος ή ένωσης διαστημάτων, τότε δίνουμε το πεδίο ορισμού με περιγραφή των στοιχείων του. Π.χ. για την

f(x) = -1 - - .Jx5 - 2χ3 + 4 έχουμε χ - 5

Ar = { χ ε R / x - 5 :;t: Ο και χ5 - 2χ3 + 4 � Ο }

Όταν θέλουμε να δείξουμε μια σχέση, δεν λέμε «πρέπει)) αλλά «αρκεί)) να δείξουμε κάτι, κατά κανόνα ισχυρότερο. π. χ. για να δείξουμε ότι a < 5 αρκεί να δείξουμε ότι a + 1 < 5 . (συμβολικά: a < 5 <::::: a + 1 < 5 ) ή αρκεί να δείξουμε a - 5 < 0 (συμβολικά: a < 5 <::::: a - 5 < Ο ) .

Για να δείξουμε ότι a < 5

ΕΥΚΛΕΙΔΗΣ Β ' 65 τ.l/66

Page 69: Ευκλειδης Β 65

Το Βήμα του Ευκλείδη

πρέπει να δείξουμε ότι a < 6 (συμβολικά: a < 5 => a < 6 ) Ούτε λέμε: Για να δεί­ξουμε ότι a < 5 πρέπει να δείξουμε a - 5 < Ο (συμβολικά: a < 5 => a - 5 < Ο ) παρόλο που ισχύει η ισοδυναμία: a < 5 <:::> a - 5 < Ο .

Εκ πρώτης όψεως φαίνεται παράλογο να επι­διώκουμε την απόδειξη της ισχυρότερης σχέσης a + 1 < 5 αντί της a - 5 < Ο . Όμως, υπάρχει περί­πτωση η ισχυρότερη σχέση να αποδεικνύεται ευκολότερα. Χαρακτηριστικά αναφέρουμε το εξής παράδειγμα από τους μιγαδικούς αριθμούς: Για κάθε z1 , z2 ε C , να δειχτεί ότι:

l z 1 + z2 l 2 � <l z1 1 2 + 1)<l z2 12 + 1 ) (1 )

Αφού o � Ι zl + z2 Ι � Ι zl l + l z2 1 , για να ισχύει η ( 1 )

αρκεί να ισχύει η Qz1 l + lz2 1)2 � Qz1 12 + l)Qz2 12 + 1) (2) Η σχέση (2), αν και ισχυρότερη, αποδεικνύ­

εται ευκολότερα ( όπως εύκολα διαπιστώνεται), κυρίως λόγω του ότι οι μεταβλητές J zι l = a , Ι z21 = b ανήκουν στο [Ο,+οο) και όχι στο C.

Επίσης, για να δείξουμε επαγωγικά ότι : Για κάθε ν ε Ν με ν � 2 ισχύει: 1 1 1 1 3 ' δ 'ξ ' - + --+ . . . + - > - , αρκει να ει ουμε οτι ν ν + 1 2ν 24

1 1 1 1 3 ισχύει : --+--+ . . . +- > -ν + 1 ν + 2 2ν 24

Η πρώτη πρόταση δεν αποδεικνύεται με τον συνήθη επαγωγικό τρόπο, ενώ η δεύτερη - αν και ισχυρότερη - αποδεικνύεται.

Στη Γεωμετρία, για παράδειγμα, για οποιοδή­ποτε ζεύγος τριγώνων ΑΒΓ , Α 'Β 'Γ' δεν είναι σω­στό να λέμε ότι:

«Για να είναι ίσα τα τρίγωνα ΑΒΓ , Α 'Β 'Γ' πρέ-

πει να είναι β = β ' , γ = γ' , Α = Α ι » διότι αυτό που μας ενδιαφέρει είναι αν οι σχέσεις αυτές αρκούν ή όχι. Π.χ. αν λέγαμε: «Για να είναι ίσα τα τρίγωνα ΑΒΓ , Α'Β'Γ' πρέπει να έχουν ίσα εμβαδά», τότε προφανώς αυτό δεν είναι αρκετό οι παραπάνω εκ­φράσεις αποδίδονται συμβολικά ως εξής:

Η πρώτη αποδίδεται με: «τριγ. ΑΒΓ = τριγ Α'ΒΓ=>β=β',γ=γ', Α=Α'» ( 1 )

όπου είναι φανερό ότι ο συλλογισμός δεν οδηγεί στην ισότητα των τριγώνων αλλά εκπορεύεται από την ισότητα αυτή . Το σημαντικότερο όμως εί­ναι ότι και η συνεπαγωγή είναι ψευδής, αφού πι­θανόν να έχουμε για κάποιο ζεύγος σκαληνών τρι-γώνων «τριγ. ΑΒΓ = τριγ Α'Β'Γ' και (β=d,γ=β',Α=Γ) οπότε β =Ι- β ' . Ισχύει λοιπόν η άρνηση της συνε­παγωγής ( 1 ) .

Η άρνηση της συνεπαγωγής (p ::::::> q) είναι η

-

σύζευξη (p Λ q) . Η δεύτερη αποδίδεται με:

«τριγ. ΑΒΓ = τριγ Α'Β'Γ' => (ΑΒΓ) = (Α 'Β 'Γ') » όπου, είναι μεν σωστή η συνεπαγωγή αλλά δεν οδηγεί στην ισότητα των τριγώνων.

Η σωστή έκφραση , λοιπόν, είναι: «Για να είναι τα τρίγωνα ΑΒΓ , Α 'Β 'Γ' ίσα,

αρκεί να είναι: β = β', γ = γ', Α = Α ι )) Συμβολικά

τριγ. ΑΒΓ = τριγ Α'Β'Γ' � β = β' , γ = γ', Α = Α ι Γενικά, λοιπόν, η έκφραση : « Για να ισχύει η

πρόταση p πρέπει να ισχύει η πρόταση φ> απο­δίδεται με (p => q) , ενώ η έκφραση :

«Για να ισχύει η πρόταση p αρκεί να ισχύει η πρόταση q » αποδίδεται με (p � q) .

Πολλές φορές αντί του σωστού (p � q) , γράφουμε (p <:::> q) , παρόλο που η συνεπαγωγή p => q είναι περιττή. Αν μάλιστα η συνεπαγωγή p => q δεν ισχύει, τότε γράφοντας (p <:::> q) οδη­

γούμαστε σε λογικό λάθος εξαιτίας μιας σχέσης που δεν μας χρειάζεται στην αποδεικτική διαδι­κασία ! ! ! . Αναγκαζόμαστε λοιπόν να ελέγχουμε και τις δύο σχέσεις p � q , p => q , ενώ μας χρειά-

ζεται μόνον η πρώτη . Αναφέρουμε χαρακτηριστικά ένα παράδειγμα

από το σχολικό βιβλίο της Β ' Λυκείου . Αν α,β,γ διαδοχικοί όροι Γεωμετρικής προόδου

( συμπληρώνουμε β =1:- -γ ), τότε να δείξετε ότι:

(α + β)2 =α

. ( 1 ) (β + γ)2 γ

Συνθετικά η ( 1 ) αποδεικνύεται εύκολα θέτο­ντας β = αλ ,γ = αλ2 . Το σχολικό βιβλίο γράφει:

(1) <=>γ(α+ β)2 = α(β+γ)2 <::>αγ(α-γ) = β2 (α-γ) (2)

και η (2) ισχύει αφού αγ = β2 . Συμβολικά δηλ.

γράφει: (1) <:::> γ( α + β)2 = α(β + γ)2 <=> <=> αγ(α - γ) = β2 (α - γ) � αγ = β2

έμμεσα δηλ. αποδέχεται ότι οι συνεπαγωγές (1) => γ( α + β)2 = α(β + γ)2 ::::::> αγ(α - γ) = β2 (α - γ) είναι περιττές, αφού διαπιστώνει ότι η συνεπαγω­γή : αγ(α - γ) = β2 (α - γ) => αγ = β2 (που πιθανόν να μην ισχύει όταν α = γ ) δεν του χρειάζεται και γράφει απλώς την αντίστροφη συνεπαγωγή

αγ(α - γ) = β2 (α - γ) � αγ = β2 (που ισχύει) . Παρ όλα αυτά όμως γράφει και τις περιττές συνε­παγωγές. Σε παλιότερη έκδοση μάλιστα, για να ι-σχύει η ισοδυναμία: αγ(α-γ) =β2 (α - γ)<=>αγ = β2

ΕΥΚΛΕΙΔΗΣ Β ' 65 τ.l/67

Page 70: Ευκλειδης Β 65

Το Βήμα του Ευκλείδη

έθετε τον περιορισμό α -:;:. γ , πράγμα που αποτε­λούσε, προφανώς, πλεονασμό. Το σωστό λοιπόν

είναι να γράφουμε: (1 ) <= γ( α + β)2 = α(β + γ )2 <=

<::: αγ(α - γ) = β2 (α - γ) <= αγ = β2 και να διαπι­στώσουμε ότι η τελευταία ισχύει, αφού οι α,β,γ εί­ναι διαδοχικοί όροι Γεωμετρικής προόδου . Με λό­για αυτό αποδίδεται ως εξής:

Για να ισχύει η ( 1 )αρκεί να ισχύει η

γ( α + β)2 = α(β + γ )2 , αρκεί να ισχύει η

αγ(α - γ) = β2 (α - γ) , αρκεί τέλος να ισχύει η

αγ = β2 , που ισχύει, αφού οι α, β, γ είναι διαδοχι­κοί όροι Γεωμετρικής προόδου. Για να μήν επανα­λαμβάνουμε διαρκώς το «αρκεί» μπορούμε με συμφωνία να γράφουμε ως εξής:

Για να ισχύει η ( 1 ) αρκεί να ισχύει η

γ(α + β)2 = α(β + γ)2 , ή αγ(α - γ) = β2 (α - γ) , ή

αγ = β2 (που ισχύει) Π ροσοχή : Ο συμβολισμός (p, η q) διαφέρει

πλέον από τον συμβολισμό (p ή q) . Πράγματι, ο δεύτερος αποδίδει μία διάζευξη ενώ ο πρώτος α­ποδίδει την έκφραση : « Για να ισχύει η p αρκεί να ισχύει η φ>

Τελικά, για να δείξουμε την p, δεν γράφουμε : Ρ <=> Ρ1 <=> Ρ2 <=> Ρ3 <=> p4 και αποδεικνύουμε

την p 4 , όπου πιθανόν να μας ξεφύγει κάποια λαν­θασμένη συνεπαγωγή Pi � Pi+I που ουσιαστικά δεν μας χρειάζεται, ούτε γράφουμε ( ακόμη χει­ρότερα) p <::::> p 1 <= p2 <::::> p3 <= p4 και αποδεικνύ­ουμε την p4 . Ο σωστός συμβολισμός είναι :

Ρ <= Ρ1 <= Ρ2 <= Ρ3 <= Ρ4 , και αποδεικνύουμε την Ρ4 ·

(Η διαδικασία αυτή λέγεται Αναλυτική μέθο­δος απόδειξης )

Π εριφραστικά : Για να ισχύει η p αρκεί να ι­σχύει p1 , ή p2 , ή p3 , ή p4 (με κόμμα πρίν από το ή ) και να αποδεικνύουμε την p4 .

Αν γνωρίζουμε ότι η απόδειξη της p προκύπτει με αφετηρία την αληθή πρόταση p4 , τότε ο παρα­πάνω συλλογισμός αποδίδεται ως εξής:

Ρ4 � Ρ3 � Ρ2 � Ρ1 � Ρ και p4 αληθής. (Η διαδικασία αυτή λέγεται Συνθετική μέθοδος απόδειξης) .

Ακολουθεί ένα παράδειγμα Αναλυτικής και Συνθετικής μεθόδου απόδειξης.

Για να δείξουμε π.χ. την πρόταση : «Για κάθε χ Ε R , ισχύει χ2 + 1 � 2χ (1) », με

την Αναλυτική μέθοδο γράφουμε : «Για να ισχύει

η ( 1 ), αρκεί να ισχύει χ2 + 1 - 2χ � Ο , ή (χ - 1)2 � Ο , που ισχύει για κάθε χ Ε R »

Συμβολικά: (1) <= χ2 + 1 - 2χ � Ο <= (χ - 1)2 � Ο , που ισχύει για κάθε χ Ε R .

Την ίδια πρόταση για να την δείξουμε με τη Συνθετική μέθοδο γράφουμε:

«Για κάθε χ Ε R , ισχύει (χ - 1)2 � Ο (2) και

(2) � χ2 - 2χ + 1 � 0 � χ2 + 1 � 2χ .

Άρα η χ 2 + 1 � 2χ ισχύει για κάθε χ Ε R . (κανόνας αποσπάσεως)»

Ε ίναι σοβαρό λάθος να γράψουμε :

« (l) � x2 + 1 - 2x � O � (x - 1)2 � 0 , που ι­σχύει » αφού έτσι δεν βρίσκουμε ως συμπέρα­σμα την (1), αλλά την θεωρούμε ως υπόθεση .

Επίσης, κακώς γράφουν ορισμένοι: « ( Ι ) <=> χ 2 + Ι - 2χ � Ο <:::::> (χ - 1 )2 � Ο , που ισχύει» αφού, όπως διαπιστώσαμε, οι συνεπαγωγές ( �) είναι περιττές και δεν υπάρχει λόγος να ασχολη­θούμε με το άν είναι ή όχι σωστές για να τις γρά­ψουμε διακινδυνεύοντας κάποιο πιθανό λάθος. Π. χ. είναι σωστό να γράψουμε: ax = ay <= χ = y , ενώ είναι λάΟος να γράψουμε :

ax = ay � χ = y , αφού πιθανόν να είναι a = Ο . Είναι λάθος λοιπόν να γράψουμε

ax = ay <::::> χ = y . Δεν υπάρχει επομένως λόγος να διακινδυνεύσουμε τη συνεπαγωγή ax =ay�x =y , που δεν μας χρειάζεται, όταν θέλουμε να δείξουμε τη σχέση ax = ay .

Τελικά λοιπόν σημασία έχει σε κάθε πρόβλη­μα να γίνεται κατανοητή η φορά της λογικής πο­ρείας, δηλαδή (p1 � p2 ) στη Συνθετική μέθοδο και (p2 <= p1 ) στην Αναλυτική μέθοδο, κάτι που δεν γίνεται φανερό με τη χρήση της ισοδυναμίας (ΡΙ <=> Ρ2 ) ·

ΠΙ) Μέθοδος Απαγωγής στο άτοπο Πολλές φορές, όταν είναι δύσκολο ή ανέφικτο

να ακολουθήσουμε την Συνθετική ή την Αναλυτι­κή μέθοδο απόδειξης μιας πρότασης p, τότε κατα­φεύγουμε στη μέθοδο της απαγωγής σε άτοπο. Δηλ. δεχόμαστε ότι ισχύει η άρνηση p της p και

-με υπόθεση την p καταλήγουμε σε άτοπο, δηλ κα-

-

ταλήγουμε στη σύζευξη r Λ r , που σημαίνει ότι

ισχύει η πρόταση r και η άρνησή της r , πράγμα, προφανώς, παράλογο ( άτοπο ), δηλ. κάτι που δεν έ-κει τόπο να σταθεί λογικά.

Συνήθως ενδιαφερόμαστε για την ισχύ μιας συνεπαγωγής p � q .

Δεχόμαστε, λοιπόν τότε, ότι ισχύει η άρνηση ΕΥΚΛΕΙΔΗΣ Β ' 65 τ. l /68

Page 71: Ευκλειδης Β 65

Το Βήμα του Ευκλείδη

p => q της συνεπαγωγής, δηλ. δεχόμαστε ότι ισχύ--

ει η σύζευξη p Λ q και καταλήγουμε σε άτοπο. Αποδεικνύουμε λοιπόν

- -(p Λ q) => (r Λ r) .

τη συνεπαγωγή

Ο κίνδυνος στην εφαρμογή αυτής της μεθόδου είναι να μην διατυπώσουμε σωστά την άρνηση p

μιας πρότασης p. Γιαυτό είναι απαραίτητο να επι­σημάνουμε τα εξής:

α) Η άρνηση της σύζευξης p Λ q είναι η διά-

ζευξη p ν q , ενώ η άρνηση της διάζευξης p ν q

είναι η σύζευξη p Λ q .

π.χ. Για κάθε α, β Ε R γνωρίζουμε ότι: α� + β� = Ο <=> α = Ο και β = Ο , οπότε α� + β� :;t: Ο <=> α :;t: Ο ή β :;t: Ο , ενώ α β = Ο <=> α = Ο ή β = Ο , οπότε : α β :;t: Ο <=> α :;t: Ο και β :;t: Ο .

β) Η άρνηση της πρότασης: « Για κάθε χ Ε Α ισχύει p( χ) »

(Συμβολικά : Vx Ε A, p(x) ) είναι η πρόταση :

«Υπάρχει χ Ε Α , τέτοιο, ώστε να ισχύει J\x) ».

(Συμβολικά : Ξ3χ Ε A, p(x) ) Συνεπώς, η άρνηση της πρότασης:

«Υπάρχει χ Ε Α ,τέτοιο, ώστε να ισχύει p(x) » είναι η πρόταση : «Για κάθε χ Ε Α ισχύει η p(x) »

π.χ. Η άρνηση της πρότασης: « Για κάθε χ Ε R ισχύει χ� � Ο » (που είναι αληθής πρόταση), είναι η πρόταση :

«Υπάρχει χ Ε R , τέτοιο, ώστε να ισχύει χ2 < 0 » (που είναι ψευδής πρόταση ) . Ανάλογα η άρνηση της πρότασης: «Υπάρχει χ Ε R , τέτοιο, ώστε να ισχύει χ 2 + 1 = Ο » (που είναι ψευδής πρόταση) εί­ναι η πρόταση : «Για κάθε χ Ε R ισχύει χ2 + 1 :;t: Ο » ( που είναι αληθής πρόταση)

Ακολουθούν μερικά χαρακτηριστικά παραδείγ­ματα. 1) Για κάθε a Ε R να δειχθεί ότι:

a3 + 3a < 3 :=> a < l . Απόδειξη : Υπενθυμίζουμε ότι δεχόμενοι την

άρνηση της πρότασής μας, δεν εννοούμε την άρ­νηση του συμπεράσματος, αλλά την άρνηση της συνεπαγωγής, δηλ. δεχόμαστε ότι ισχύει η σύζευ-ξη : a3 + 3a < 3 και a � 1 .

Με υπόθεση λοιπόν αυτό, θα καταλήξουμε σε άτοπο.Πράγματι a � 1 :=>a3 � 1 και 3a�3:=>a3 +3a�4 :=>a3 +3a � 3 ενώ έχουμε και a3 + 3a < 3 (άτοπο )

Παρατηρήστε, λοιπόν, ότι άν είχαμε δεχθεί μόνο την άρνηση του συμπεράσματος, δηλ. a � 1 ,

χωρίς την a 3 + 3a < 3 , δεν θα καταλήγαμε σε άτο­πο. Ας προσέξουμε ότι το άτοπο, δεν αναφέρεται πάντα στη <ruζευξη της υπόθεσης με την άρνησή -της, αλλά σε οποιαδήποτε σύζευξη r Λ r . π. χ. 2>0 και 2::::;0, όπως στο ακόλουθο παράδειγμα.

2) Για κάθε α, β, χ Ε R να δειχθεί ότι:

2(χ - 1) = αβ} z => 2xz * az + 4χ

β = 2 { 2(χ - 1) = αβ (1) Απόδειξη : Έστω β2 = 2 (2) τότε:

2χ 2 = α2 + 4χ (3) ( 2 )

(1) => α2β2 = 4(χ - 1)2 => α2 · 2 = 4χ2 - 8χ + 4 (3 ) :=> α2 = 2χ 2 - 4χ + 2=> α2 = α2 + 2 => 0 = 2

ενώ Ο :;t: 2 , πράγμα άτοπο. Μπορεί να γίνει φυσι­κά και απόδειξη με τη συνθετική μέθοδο.

3) Να δειχθεί ότι: Για κάθε � > Ο , υπάρχει

χ Ε R τέτοιο, ώστε χ3 + 2χ + 3 > � , ( χωρίς την έννοια του ορίου )

Απόδειξη : Έστω, ότι: Υπάρχει � > Ο , τέτοιο, ώστε για κάθε χ Ε R να ισχύει χ3 + 2χ + 3 ::::; � . Τό­τε και για κάθε χ > Ο θα έχουμε χ < χ3 +2�+3 ::::; � , δηλ. χ < � . Επομένως για χ = � > Ο θα έχουμε � < � ενώ είναι � � � , πράγμα άτοπο.

Π ΡΟΣΟΧ Η ! ! ! . Δέν πρέπει να γίνεται σύγχυ­ση της μεθόδου αυτής ( απαγωγής σε άτοπο ) με την μέθοδο της αντίθετοαντιστροφής, η οποία, ως γνωστόν, βασίζεται στην ισοδυναμία των προτάσε-ων: (p => q) και ( q => p) . Για παράδειγμα, σε κά-

θε τρίγωνο ΑΒΓ η πρόταση (Β :;t: Γ => β :;t: γ) είναι

ισοδύναμη με την πρόταση : (β = γ => Β = Γ) η ο­ποία, προφανώς, είναι πολύ απλούστερη .

Ομοίως, για να δείξουμε ότι μία συνάρτηση f : Α � R είναι 1 - 1 , αντί του ορισμού που λέει ότι: Η f είναι 1 - 1 όταν και μόνο: Για κάθε χ1 ,χ2 ΕΑ ισχύει η συνεπαγωγή χ1 *Xz :=>f(x1 ) :;t:f(Xz) αρκεί, σύμφωνα με τη μέθοδο της αντίθετοαντι­στροφής, να δείξουμε ότι: Για κάθε χ 1 , χ2 Ε Α ι-σχύει η συνεπαγωγή : f(x , ) = f(x2 ) => χ , = χ 2 • Έ­τσι, αποφεύγουμε την ανισότητα α :;t: β , που ουσι­αστικά κρύβει δύο ανισότητες (α < β ή α > β) και ασχολούμαστε με την αντίστοιχη ισότητα α = β .

IV) Σωστή Ανάγνωση Συνεπαγωγής - Ισο­δυναμίας

Η έκφραση (p => q) διαβάζεται: «αν p τότε q>>. Είναι λάθος λοιπόν να γράφουμε: (αν p => q )

ή ( από την p => q ) διότι τότε ουσιαστικά γράφουμε:

ΕΥΚΛΕΙΔΗΣ Β ' 65 τ.l/69

Page 72: Ευκλειδης Β 65

Το Βήμα του Ευκλείδη

(αν αν p τότε q) ή ( από την αν p τότε q) Εκτός από το αντιαισθητικό των δύο συνεχόμενων «αν» η επε­ξήγηση είναι ότι : το πρώτο «αν», δηλ το «αν» στην έκφραση (αν p � q ),δεν αναφέρεται στην πρόταση

p σJ..λά στην συνεπαγωγή p � q . Επίσης στην έκ­

φραση: ( από την p � q ) το «από την » δεν αναφέ­

ρεται στην p σJ..λά στην συνεπαγωγή ( p � q ). Δη­

λαδή είναι σαν να γράφουμε: «Α ν ισχύει η συνεπαγωγή p � q » ή « από την

συνεπαγωγή p � q »

Αυτές είναι συντακτικά ελλιπείς προτάσεις. Κανονικά, θα πρέπει να τις συμπληρώσουμε ως εξής: « αν ισχύει η συνεπαγωγή p � q , τότε συμ-

βαίνει κάτυ> ή « από την συνεπαγωγή p � q προ­

κύπτει κάτι» π.χ. Για κάθε χ ε R , αν (χ > 0 �3 > 5) ,τότε χ :s; Ο (Ι)

ή ισοδύναμα: Από την συνεπαγωγή (χ > ο � 3 > 5) προκύπτει χ ::; ο .

Παρατηρούμε δηλαδή ότι, όταν αποδεικνύου­με μία συνεπαγωγή p � q , δεν έχουμε εξασφαλί-

σει την αλήθεια κάποιας από τις προτάσεις p, q. π.χ. και οι τρείς συνεπαγωγές που ακολουθούν

είναι σωστές 2 < 0 �3 > 5 , 2 < 0�3 < 5 , 2 > 0 � 5 > 3

Για να ισχύει, λοιπόν η q,αρκεί να ισχύει η συ­νεπαγωγή (p � q) και να είναι αληθής η p (κανό-

νας αποσπάσεως) . Ανάλογα, όταν αποδεικνύουμε μια ισοδυναμία

(p <::::> q) δεν έχουμε εξασφαλίσει την αλήθεια ού-

τε της p, ούτε της q. π. χ. 2 < Ο <::::> 3 > 5 . Όπως παρατηρήσαμε στη σχέση (1), αν ισχύει

η συνεπαγωγή (p � q) και η q είναι ψευδής, τότε

συμπεραίνουμε ότι και η p είναι ψευδής. Επίσης, όταν η q είναι ψευδής και θέλουμε να

ισχύει η συνεπαγωγή (p � q) τότε πρέπει και

αρκεί να είναι η p ψευδής. Αναφέρουμε χαρακτηριστικά δύο παραδείγματα.

.:1� Για κάθε χ ε (Ο, +οο) να δείξετε ότι:

1 χ + - < 2 � η μ2χ + συν2χ = 2

χ Προφανώς, για κάθε χ ε (0, +οο) ισχύει

ημ2χ + συν2χ = 1 -:;; 2

(ψευδές συμπέρασμα), οπότε για να ισχύει η συνε­παγωγή αυτή αρκεί να δείξουμε ότι: Για κάθε

χ ε (0, +οο) ισχύει χ + _!_ � 2 (δηλ. ότι και η υπό-χ

Οπότε χ + _!_ � 2 . χ

�� Να βρείτε τις τιμές του λ ε R , ώστε για κά­θε χ ε R να ισχύει η συνεπαγωγή :

χ2 - 2χ + 5 < λ � χ2 + 1 :::;; Ο ( I ) Παρατηρούμε ότι για κάθε χ ε R ισχύει

χ2 + 1 > Ο (ψευδές συμπέρασμα). Για να ισχύει

λοιπόν η ( Ι ) πρέπει και αρκεί: χ 2 - 2χ + 5 � λ , για κάθε χ ε R ( 11 ) (ψευδής υπόθεση). Έχουμε:

(11) <::::> χ2 - 2χ + 5 - λ � Ο , για κάθε χ ε R <::::> Δ ::; Ο <::::> 4-4(5-λ) :s; Ο <::::> -4 + λ ::; Ο <::::> λ ::; 4 ,

όπου Δ η διακρίνουσα του τριωνύμου

f(x) = χ2 - 2χ + 5 - λ .

Αν γράψουμε :

Πρέπει και αρκεί: χ 2 - 2χ + 5 � λ , για κάθε

χ ε R <::::> χ 2 - 2χ + 5 - λ � Ο , για κάθε χ ε R <::::> Δ ::; Ο <::::> 4 - 4( 5 - λ) ::; Ο <::::> -4 + λ ::; Ο <::::> λ ::; 4

τότε ουσιαστικά δεν απαιτούμε την ισχύ της ( 11 ) αλλά απαιτούμε να είναι σωστές οι παραπάνω ισο­δυναμίες, οι οποίες προφανώς είναι σωστές. Ουσι­αστικά,λοιπόν, δεν απαιτούμε τίποτα ! ! !

Πρέπει και αρκεί

Ρι <=> Ρ2 <=> p3 ,που σημαίνει ουσιαστικά να είναι

ισοδύναμες οι προτάσεις p 1 , p2 , p3 , αλλά πρέπει

και αρκεί να ισχύει η p1 • Στη συνέχεια, αν θέλου­

με να αντικαταστήσουμε την p1 με κάποια ισοδύ­

ναμη σχέση γράφουμε Ρι <=> Ρ2 <=> Ρ3 •

Επίσης, Για κάθε χ ε R συνε-

πάγεται χ 2 � Ο

(Συμβολικά: Vx ε R � χ2 � Ο ), αλλά γράφουμε:

Για κάθε χ ε R ισχύει χ2 � Ο . (Συμβολικά:

Vx ε R, χ2 � Ο )

Επίσης, γράφουμε : Για κάθε χ ε R ισχύει η

συνεπαγωγή χ > 3 � χ2 > 9 (Συμβολικά: Vx ε R , (x > 3 � χ2 > 9)

Για κάθε χ > 3 συνεπάγεται χ 2 > 9 (Συμβολικά: Vx > 3 � χ2 > 9 ) .

(Άρα: η Α = Β ) (Άρα: το 1im .Jx - 1 = +οο ) .

χ ---++οο

Έχουμε τότε την εντύπωση ότι το «η>> είναι άρ­

θρο της γωνίας Α καθώς και ότι το άρθρο «το» εί­

ναι το άρθρο του ορίου lim .Jx - 1 . Σκεφθείτε ότι χ ---++οο

θεση είναι ψευδής) . αν γράφαμε αντί της έκφρασης (η Α = Β ) την έκ-

Πράγματι: χ +_!_

- 2 = χ 2 + 1 - 2χ (χ - 1)2 � Ο . φραση (η Β = Α ), που δηλώνει το ίδιο ακριβώς, τό-

χ χ χ Λ

τε το «η>> θα ήταν άρθρο της γωνίας Β ! ! ! . Ουσια-στικά, λοιπόν, το «η>> είναι άρθρο της ισότητας

ΕΥΚΛΕΙΔΗΣ Β ' 65 τ. l/70

Page 73: Ευκλειδης Β 65

Το Βήμα του Ευκλείδη

Α = Β , οπότε η έκφραση (Άρα: η Α = Β ) , δηλαδή (Άρα η ισότητα Α = Β ) είναι ελλιπής. Μία πλήρης έκφραση π. χ. θα ήταν η εξής:

«Άρα, η Α = Β συνεπάγεται την α=β» δηλαδή «Άρα, η ισότητα Α = Β συνεπάγεται την ι­

σότητα α=β» Ακόμη χειρότερα, στην δεύτερη έκφραση, το

άρθρο «το» είναι τελείως ατυχές καθώς είναι άρθρο της ισότητας, επομένως έπρεπε να αντικατασταθεί με «η» δηλαδή να γράψουμε: (Άρα, η ισότητα lim � = +οο ) , που είναι πάλι ελλιπής έκφραση.

Χ -Η«:

Μία πλήρης έκφραση θα ήταν π. χ. (Άρα, η ισότητα lim .Jx - 1 = +οο συνεπάγε-

·�-

ται την ανισότητα .Jx - 1 > 1000 , για τιμές του χ κοντά στο +οο)

(Σύμφωνα με το σχολικό βιβλίο, λέγοντας χ κοντά στο +οο, εννοούμε χ>δ για κατάλληλο δ>Ο)

" " ' ' . . .,,,. � ' "' ',, , \· .,_:� ..._ .. .. \_ιf �.ι

Όταν θέλουμε να λύσουμε μία εξίσωση με την χρήση βοηθητικού αγνώστου π.χ. την 2η μ: χ - 3η μχ + 1 = Ο (1) , γράφουμε :

( 1 ) <=> J , ημχ = Υ ή - εκφραζόμενοι περι-l2y - - 3y + 1 = 0

φραστικά -μπορούμε να γράψουμε: Θέτουμε η μχ = �- ( i) οπότε η επιλύουσα της ( 1 ) είναι η 2 y: - 3)" + 1 = 0 (2) .

«Θέτουμε η μχ = y οπότε η ( 1 ) γίνεται 2y2 - 3y + 1 = Ο (3) », αφού η (3) έχει

δύο μόνο ρίζες και δεν μπορεί να είναι ισοδύνα­μη με την (1), που έχει άπειρες ρίζες.

Επίσης, δεν είναι απαραίτητο να γνωρίζουμε εκ των προτέρων το σύνολο τιμών της συνάρτησης y. Στην περίπτωσή μας, προφανώς, το σύνολο τι­μών της y είναι το [- 1 , 1 ] . Αν όμως είχαμε την ε-

3 2 3 ξίσωση : 2(ημχ - -) - 3(ημχ - -) + 1 = 0 ,

ημχ η μχ

θα θέταμε y = ημχ - -3- (ii) .

η μχ Τότε δεν θα ήταν εύκολο να βρούμε το σύνολο

τιμών της y, αλλά ούτε το χρειαζόμαστε. Το πρό­βλημα λοιπόν είναι απλώς να δούμε αν οι εξισώ­σεις (i), (ii) λύνονται όταν y = y, ή y = y2 ( όπου y1 , y2 είναι οι ρίζες της(2)). Ανάλογα, φυσικά, ερ­γαζόμαστε για την ανίσωση 2ημ2χ - 3ημχ + 1 > Ο .

Όταν ζητάμε την ικανή και αναγκαία συνθήκη για να ισχύει μία σχέση, τότε την aντικαθιστούμε με μία ισοδύναμη σχέση, εf.i:αοντας προσεκτικά την

ισχύ της ισοδυναμίας. π.χ. Για να είναι αχ2 + βχ + γ > Ο , για κάθε χ Ε R , πρέπει και αρκεί (α>Ο] ή (α = β = ΟJ , όπου Δ=β2 -4αy , α, β, γ Ε IR. Δ<Ο) γ > Ο j

Για να εξασφαλίσουμε την συνέχεια μιας συ­νάρτησης f στη θέση χ0 Ε Ar γράφουμε:

«Πρέπει και αρκεί lim f(x) = f(x0 ) », και Χ � Χ ο

«Πρέπει και αρκεί lim. f(x) = lim_ f(x) = f(x0 ) », Χ � Χ ο Χ ---+ Χο

αφού η εύρεση του ορίου στο χ0 δεν γίνεται πά­ντοτε μέσω των πλευρικών ορίων, διότι μπορεί η f να μήν αλλάζει τύπο εκατέρωθεν του χ0 ή να μήν ορίζεται αριστερά του χ0 ή δεξιά του χ0 •

Ανάλογα, για να εξασφαλίσουμε την παραγω­γισιμότητα μιας συνάρτησης f στη θέση χ0 Ε Ar , το σωστό είναι να γράφουμε:

«Πρέπει και αρκεί lim λ( χ) Ε R , όπου Χ -+ Χ ο

λ( ) _ f(x) - f(x0 ) Α - { } Χ - με Χ Ε f Xa • Χ - Χ0

Συνηθίζεται όμως σε πολλά ευρετικά προβλή­ματα ( Λύση εξίσωσης, κατασκευή τριγώνου κλπ) να βρίσκουμε πρώτα κάποιες αναγκαίες συνθήκες και μετά να ελέγχουμε αν είναι και ικανές.

Για παράδειγμα, το σχολικό βιβλίο ( αλλά και τα περισσότερα βοηθήματα) για να εξασφαλίσει την παραγωγισιμότητα της συνάρτησης { x2 + x + a2 , χ < Ο , f(x) = 3 στο χ0 = Ο , γραφει:

χ + ax + 1 , χ � Ο Για να είναι η f παραγωγίσιμη στο χ0 = Ο

πρέπει ( χωρίς να αρκεί πάντα ) η f να είναι συνε­χής στο χ0 = Ο , υπονοώντας δηλ. ότι: f παραγωγί-σιμη στο χ0 = Ο ::::} f συνεχής στο χ0 = Ο .

Βρίσκει, έτσι ότι a Ε { -1 , 1 } . Ελέγχει μετά για κάθε μία απο αυτές τις τιμές αν ισχύει η σχέση lim λ (χ) Ε R ( 1 ) και βρίσκει ότι δεκτή τιμή είναι

Χ -+ Χ ο

μόνο η a = 1 . Πιθανόν όμως να βρούμε περισσό­τερες τιμές του a, οπότε είναι ασύμφορο να εξετά­ζουμε την ισχύ της ( 1 ) για κάθε μία από αυτές τις τιμές του a που καθιστουν την f συνεχή ( φαντα­σθείτε π.χ. 5 τιμές του a), ενώ μπορούμε να ελέγ­ξουμε εφάπαξ πότε ισχύει η ( 1 ) .

Στο συγκεκριμένο πρόβλημα θα βρούμε ότι: lim λ( χ) = 1 , ενώ lim λ( χ) = a μόνο όταν χ-+0+ x-+0-

a2 - 1 = Ο , οπότε (1) <=> <=> a = 1 . {a2 - 1 = Ο a = 1

Αποφεύγουμε, έτσι τις δοκιμές.

ΕΥΚΛΕΙΔΗΣ Β ' 65 τ.l/71

Page 74: Ευκλειδης Β 65

Το Βήμα του Ευκλείδη

Επίσης, βλέπουμε να γράφεται η συν-θήκη ως εξής: { lim λ( χ) ε R (1) f παραγωγίσιμη στο Χ0 <=>

};x;(x) = f(xo ) (2)

δίνουμε δηλ. την εντύπωση ότι εκτός από τη σχέση ( 1 ) που είναι ικανή και αναγκαία χρειαζόμαστε και άλλη μία σχέση (2) για να την κάνουμε πιό ικανή και πιό αναγκαία. Η σχέση (2) λοιπόν εί­ναι,προφανώς, πλεονασμός. Ανάλογη διαδικασία ευρετική ακολουθούμε και για την επίλυση εξισώ­σεων. Για παράδειγμα ας θεωρήσουμε την εξίσω-ση J2 - x = x (1)

Το πεδίο ορισμού της ( 1 ) είναι το Α = { χ ε R I 2 - χ � Ο (2) } .

Χωρίς να λύσουμε την (2), η οποία πιθανόν να είναι μεγάλου βαθμού και να μήν μπορεί να λυθεί, γράφουμε : Στο Α έχουμε : (l) => (J2 - x )2 = χ 2 ::::> 2 - χ = χ2 => χ2 + χ - 2 = 0

=> χ ε { 1 , -2} Όπως και στην περίπτωση της συνέχειας (που δεν εξασφαλίζει πάντα την παραγωγισιμότητα) έτσι και εδώ ελέγχουμε αν οι τιμές αυτές επαληθεύουν την ( 1 ). Αρχικά εξετάζουμε άν επαληθεύουν τη σχέση (2), που πράγματι συμβαίνει και στη συνέ­χεια διαπιστώνουμε ότι μόνο η τιμή Χ ι = 1 επαληθεύει την ( 1 ) . Τελικά λοιπόν: (1) <=> χ = 1 . Η διαδικασία αυτή βέβαια, δεν ενδείκνυται αν οι τι­μές του χ που βρίσκουμε είναι δύσχρηστες για α-

7 ± Jl3 ' ντικατάσταση . π.χ. Χ ι ? = . Σαυτη τηνπε-· - 1 5

ρίπτωση συμφέρει να έχουμε το πεδίο ορισμού Α, με τη μορφή διαστήματος ή ένωσης διαστημάτων, δηλ. να είναι η (2) επιλύσιμη, όπως στην περίπτω­σή μας όπου Α = ( -οο, 2] και να γράψουμε:

Στο Α = ( -οο, 2] έχουμε :

(l) � ι.,�2::)� � χ' � {χ,:�� χ

<=> {χ2 +χχ�� = Ο

6 {χ :{�.�2} <=> x = l

που είναι δεκτή τιμή, αφού 1 ε Α . Έτσι λοιπόν, αποφεύγουμε την αντικατάσταση

και ελέγχουμε μόνο αν οι τιμές ανήκουν στο Α.

Θα προσπαθήσουμε να aποσαφηνίσουμε τί εν­νοούμε όταν ζητάμε τον γεωμετρικό τόπο σημείων στην Αναλυτική Γεωμετρία, δηλαδή τον γ. τόπο των σημείων M(x, y) τα οποία είναι τέτοια, ώστε

χ = f(λ) , y = g(λ) , όπου λ ε R . Το πρόβλημα λοιπόν τότε είναι να βρούμε ένα σχήμα C : h(x, y) = O τέτοιο, ώστε για κάθε σημείο M(x0 , y0 ) να ισχύει η ισοδυναμία:

Μ ε C <=> Υπάρχει λ ε R , ώστε χ0 = f(λ) , y0 = g(λ) . Δηλ. h(x0 , y0 ) = 0 <=> {χ0 = f(λ) Υπάρχει λ ε R , ώστε Υ

ο = g(λ) Ουσιαστικά, ζητάμε την ικανή και αναγκαία

συνθήκη μεταξύ των χ0 , y 0 , ώστε να έχει λύση ως

{χ0 = f(λ) προς λ ε R το σύστημα: Υ

ο = g(λ) δηλ. να είναι ΣΥΜΒΙΒΑΣΤΟ το σύστημα αυτό.

Η απλούστερη περίπτωση είναι όταν οι f(λ), g(λ) είναι πολυωνυμικές συναρτήσεις 1 ου

{χ0 = 2λ + 3 βαθμού. Π. χ το (Σ) _ . Τότε

y0 - 1 - 3λ

το (Σ) είναι συμβιβαστό <=>{λ - '\-

}

συμβιβαστό λ =

1 -yo 3

χ -3 1-y: <=>-ο- = __ ο <=>3Χσ -9=2-2y0 <::::>3χ0 +2y0 - 1 1 = 0

2 3 και διαφορετικά:

(Σ) συμβιβαστό <=> { λ = Χ ο 2- 3

συμβιβαστό y0 = 1 - 3λ

χ - 3 <=> y0 = 1 - 3(-0 -) <::::> 3χ0 + 2y0 - 1 1 = 0 .

2 Αρα, ο γ. τ. Είναι η ευθεία (ε) : 3χ + 2y- 1 1 = 0 . Αντ αυτού,

{χ=2λ+3 {λ=

χ�3 χ-3 _ 1 -y 3 2 1 1 -0 => =>-- -- => χ+ y- -

y= 1-3λ λ =1-y 2 3 3

Έτσι δεν είμαστε βέβαιοι ότι ο γ.τόπος είναι ολόκληρη η ευθεία. Επίσης,

{χ = 2λ + 3 <=> {

λ = χ � 3

<=> λ = χ - 3 =

1 - Υ y = 1 - 3λ λ = 1 - y 2 3

3

<::::> χ - 3 =

1 - y <::::> 3x + 2y - 1 1 = 0 2 3

(Ετσι, φαίνεται μία σχέση με 3 αγνώστους χ, y, λ, να ισοδυναμεί με μία σχέση με δύο αγνώ­στους x,y. Δίνεται δηλ. η εντύπωση ότι δύο ίσα

ΕΥΚΛΕΙΔΗΣ Β' 65 τ. l/72

Page 75: Ευκλειδης Β 65

Το Βήμα του Ευκλείδη

χ - 3 1 - y κλάσματα -2- , -3- μπορούμε να τα εξισώσου-με με οποιοδήποτε λ Ε R , π. χ. χ-3 1 -y χ-3 1 -y --=-- �-- =-= 7 , που είναι παράλογο . 2 3 2 3

Μια τέτοια παρανόηση οφείλεται στην μή α­ποσαφήνιση του ρόλου του λ)

Προσέξτε επί πλέον ότι στις δύο τελευταίες διατυπώσεις, γίνεται και σύγχυση της συνάρτησης h(x, y) = 3x + 2y - 1 1 με την αριθμητική τιμή της h(x0 , y0 ) = 3χ0 + 2y0 - 1 1 . Αυτή η σύγχυση γίνε­ται εμφανέστατη στο ακόλουθο παράδειγμα.

Για την εύρεση του γ.τ. των σημείων Μ, που απέχουν από την ευθεία (ε) : 3χ - 4y + 5 = 0 από-σταση ίση με 2, γρ άφουν κακώς:

Το Μ(χ, y) είναι σημείο του τόπου l 3x - 4y + 5l <:::::> = 2 <:::::> 3χ - 4y + 5 = ± 1 0 κ.λ.π . .J32 + 42 Η σχέση αυτή είναι ακατανόητη , αφού δεν έχει

νόημα να μιλάμε για απόσταση του σημείου Μ(χ . :γ ) από την ευθεία 3χ - 4y + 5 = Ο , αλλά για απόσταση του σημείου M(x0 , y0 ) από την ευθεία 3χ - 4�- - 5 = Ο . Κανονικά λοιπόν πρέπει να γρά­φουμε : Το M(x0 , y0 ) είναι σημείο του τόπου

i 3 x ,, - 4yo + 5 1 <:::::> I • = 2 <:::::> '\/ 3 : + 42 3 χ .:, - 4y0 + 5 = ± 1 0 <=:> 3χ0 - 4y0 - 5 = 0 ή 3 χ 0 - 4y0 + 1 5 = Ο <:::::> Μ Ε (ε 1 ) υ (ε2 ) , όπου ( ε , ) : 3χ - 4y - 5 = 0 , (ε2 ) : 3χ - 4y + 1 5 = 0 .

Σ το ακόλουθο δυσκολότερο παράδειγμα γίνε­ται πιό κατανοητή η διαδικασία

Να βρεθεί ο γ. τ. των σημείων M(z) , με 5λ + i '

z = -- , οπου λ Ε R . 1 + λi Έστω z = χ0 + y0i . Τότε για να είναι το M(z)

σημείο του τόπου πρέπει και αρκεί να υπάρχει λ R ' . 5λ + i (1) Ε , ωστε: χ0 + y0ι = --. . 1 + λι

Όμως (1) <:::::> Χ0 + λχ0i + y0i - λy0 = 5λ + i <:::::> {Χ0 - λy0 =:λ <:::::> {Χ0 =�Υ ο + 5) (Σ)

λχο + Υ ο - 1 λχο - 1 - Υ ο α) Αν χ0 ::F Ο , τότε (Σ) συμβιβαστό <:::::> {Χο = λ(Υο + 5)

<=> λ = 1 - Υ ο συμβιβαστό<=:> χ0 =

1 :Υ ο (y0 + 5) χ ο ο

<:::::> χ � - �-; + 4y0 = 5 <:::::> χ� + (y0 + 2/ = 9 <:::::> Μ Ε C

' {χ2 + (y + 2)2 = 9 ' οπου (C) : , δηλ. τα σχημα (C) x ::F O

είναι ο κύκλος με κέντρο Κ(Ο, -2) και ακτίνα ρ = 3 , εκτός από τα σημεία τομής του με την ευ­θεία χ = Ο , που είναι ο άξονας y'y , δηλ. εκτός από τα σημεία Α(Ο, 1) , Β(Ο, -5) .

β) Αν χ0 = 0 , τότε: {Ο = λ(y0 + 5) (Σ) συμβιβαστό <:::::> λ · 0 = 1 - y0 συμβιβαστό {0 = 6λ <:::::> συμβιβαστό <:::::> Υ ο = 1 , άρα το ση-Υ ο = 1

μείο Α(Ο, 1) του άξονα y'y είναι σημείο του τό­που. Τελικά, ο γ.τ. είναι το σχήμα (γ) = (C) υ {Α} δηλ. ο κύκλος με κέντρο Κ(Ο, -2) και ακτίνα ρ = 3 , εκτός από το σημείο Β( Ο, -5) .

Υ Α(ο,γ � ΧΊ \\

χ

- -

Κ(Ο, -2)

� --//

Β(Ο, -5) y = -5 β τροπος : α) Αν y0 ::F -5 , τότε: (Σ) συμβιβαστό <:::::> { λ - Χο

<:::::> - Υ ο + 5 συμβιβαστό <:::::> �- Χο = 1-y0 <:::::> Υο +5 λχο = 1 - yo <:::::> χ� + (y0 + 2)2 = 9 <:::::> Μ Ε (C')

' cc· ') {χ2 + (y + 2)2 = 9 δ λ δ'

' οπου : η α η το σχημα y ::F -5 (C') είναι ο κύκλος με κέντρο Κ(Ο, -2) και ακτί­να ρ = 3 , εκτός από τα σημεία τομής του με την ευθεία y = -5 , δηλ. εκτός από το σημείο Β( Ο, -5) . , <=>{Χο =λ· Ο<=>{ Χο =0 β) Αν y0 = -5 , τοτε: (Σ) , λΧο =6 λ · 0=6 που είναι αδύνατο. Άρα το σημείο Β(Ο,-5) δεν εί­ναι σημείο του τόπου .

Ο γ. τόπος λοιπόν είναι το σχήμα (C') . Και πάλι δηλ. πρόκειται για τον κύκλο με κέντρο Κ(Ο, -2) και ακτίνα ρ = 3 εκτός του σημείου Β(Ο, -5) .

ΕΥΚΛΕΙΔΗΣ Β ' 65 τ. l/73

Page 76: Ευκλειδης Β 65

Ο Ε υ κλεί δ ης

π ροτείνει .. .. .

«Η καρδιά των μαθηματικών είναι τα προβλήματα και οι λύσεις και ο κύριος λόγος ύπαρξης του μαθηματικού είναι να λύνει προβλήματω>.

P. R. HALMOS

Επιμέλεια: Γ. Τριάντος, Ν. Αντωνόπουλος, Θ. Κυριακόπουλος

1 02 . Δίνονται δυο σταθερά σημεία Α, Β και μεταβλητό σημείο Ν. Αν Νι είναι το συμμετρικό του Ν ως προς το Α, Ν2 στην προέκταση της Ν.Β ώστε να ισχύει ΒΝι=2ΒΝ2 και Μι το μέσο της ΒΝ, να αποδείξετε ότι το κέντρο βάρους του τριγώνου ΝιΝ2Μι διατηρείται σταθερό, ανεξάρτητα από τη θέση του σημείου Ν.

ΛiJ ση

(Επροτάθη από το χημικό Καρβελά Δημήτρη - Πεύκη Αττικής)

Από τον κ. Άνδρη Ιωάννη Από τα δεδομένα του προβλήματος προκύπτει

ότι οι ΒΑ και Μι είναι διάμεσοι στο τρίγωνο ΒΝΝι , οπότε το σημείο τομής τους G είναι το βαρύκεντρο του τριγώνου αυτού, οπότε είναι σταθερό σημείο επί του ΑΒ.

Ν Λ,

I ''··

Επίσης Νρ = 2 = Ν,Β , οπότε Μ1Ν2 Il GB . GM, ΒΝ2

Τότε όμως το σημείο θ είναι το μέσο του τμήματος GB και επιπλέον χωρίζει τη διάμεσο ΝιΖ σε λόγο 2 : 1 , οπότε είναι το βαρύκεντρο του τριγώνου ΝιΝ2Μ ι . Επομένως το βαρύκεντρο του τριγώνου ΝιΝ2Μι διατηρείται σταθερό ανεξάρτητα από τη θέση του σημείου Ν (είναι το σημείο του τμήματος ΑΒ το οποίο καθορίζεται από την ισότητα ΑΘ=2ΘΒ).

Λ ύσt:ις έση:ιλαν �:π

ίσης ο ι συνάδελφο ι Π ι:τρολέκας Στέλιος- Λραπετσιί>να, Λου κάς

Χυτή ρ η ς - Κ έ ρ κυ ρ α , Χ ατζηελανίδ η ς Π έτρος -Θεσσαλονίκη κ α ι ο Γιώργος Μπατέλης, φοιτητής Μ αθ η μ ατι κοiJ - Π άτρ α

1 03. Στις πλευρές ΔΓ και ΓΒ τετραγώνου ΑΒΓ Δ θεωρούμε τα σημεία Κ, Λ και Μ, Ν αντίστοιχα,

ώστε να ισχύουν ΔΚ=ΚΛ=ΛΓ=ΓΜ=ΜΝ=ΝΒ. Αν οι ευθείες ΑΚ, ΚΛ τέμνονται από τις ΔΝ, ΔΜ στα σημεία Ε, Ζ και Η, Θ αντίστοιχα, να αποδείξετε ότι α) ΕΗ//ΔΓ β) 13(ΕΖΗ)=20(ΕΘΗ)

(Επροτάθη από το συνάδελφο Γιώργο Αποστολόπουλο - Μεσολόγγι)

Λύση (Από το συνάδελφο Γ. Σταματογιάννη -Δροσιά Αττικής) α) Αν α είναι η πλευρά του τετραγώνου, τότε

, λ β , , ΑΛ αJU ΑΗ α.JW ευκο α ρισκουμε οτι = -- και =-- .

3 3 Α Β

Δ Κ Λ Γ Από την ισότητα των τριγώνων ΑΔΛ και ΔΓΝ

συμπεραίνουμε ότι Ζ = 1 80° - ( φ + ω) = 1 80° - 90° = 90°

Ομοίως από την ισότητα των τριγώνων ΑΔΚ και ΔΓΜ βρίσκουμε ότι Θ = 90°

Το τετράπλευρο ΕΘΗΖ είναι εγγράψιμο, οπότε ΑΖ ΑΕ - = -ΑΘ ΑΗ

Στο ορθογώνιο τρίγωνο ΑΔΛ ισχύει:

ΑΖ = ΑΔ2

=> ΑΖ = 3αJ13

ΑΛ 1 3

( 1 )

(2)

Ομοίως από το ορθογώνιο

ΑΘ = 3α.Jl0 1 0

τρίγωνο ΑΔΚ

βρίσκουμε ότι: (3)

Από τις ισότητες ( 1 ), (2), (3) συμπεραίνουμε

ότι: ΑΕ = {10 = ΑΚ οπότε ΕΗIΙΔΓ. ΑΗ Vl3 ΑΛ

β) Από την ομοιότητα των τριγώνων ΖΕΗ και ΖΔΛ έχουμε: Από τις ( 4 ), ( 5) έχουμε :

ΕΥΚΛΕΙΔΗΣ 8 '65 τ. l /74

Page 77: Ευκλειδης Β 65

Ο Ευιώ:ίδης προτείνει ...

Ομοίως

(ΖΕΗ) = ( ΕΗ )2

(ΖΔΛ) ΔΑ

(ΕΘΗ) = ( ΕΗ )2

(ΚΘΔ) ΔΚ Δ Δ

Επίσης Δ Θ Κ � Α Δ Κ οπότε

(4)

(ΔΘΚ) ( ΔΚ )2 1 α2 --'------� = - = - => (ΔΘΚ) = - (5) (ΑΔΚ) ΑΚ 1 0 60

Από τις (4), (5) έχουμε:

(ΕΘΗ) = � · _2._(ΕΗ)2 = _2._(ΕΗ)2

60 α2 60

Επίσης ι� (�)2 => (ΖΔΛ) = :� και

( ΖΕΗ) = 4α2

· _2_(ΕΗ)2 = _2._(ΕΗ)2

39 4α2 39

Επομένως 60

(ΕΘΗ) = (ΕΗ)2 = 39 (ΖΕΗ) => 9 9

=> 60{ΕΘΗ) = 39( ΖΕΗ) => 1 3(ΕΖΗ) = 20{ΕΘΗ) που είναι τα ο ζητούμενο

· . ι ' c;· ,� ι , ; . . .. J ν επ[cηιr,; Ο ί συνάδt:λφ ο ι ? , - VJ.cγ. 'η , 1{ ·G�J �Y'ψ; Χ μ τιJσόσ'rο μος ·-

1 U6 . Αν για τους μη αρνητικούς αριθμούς χ, y, z ισχύει x+y+z=1 να βρείτε τη μέγιστη τιμή του αθροίσματος.

S = xy (x + y) + yz (y + z) + zx (z + χ) (Επροτάθη από το συνάδελφο Γιιοργο

Τσαπι1κίδι1- Αγρίνιο)

Είναι: S=xy( 1 -z )+yz( 1-χ )+zx( 1-y )=xy+yz+zx-3xyz

Από την ταυτότητα του Euler έχουμε:

x3+y3+z3 -3xyz=(x+y+z)(x2+y2+z2 -xy-yz-zx)<=> -3xyz= x2+y2+z2 -xy-yz-zx-x3 -y3 -z3

οπότε η ( 1 ) γίνεται: S=xy+yz+zx+ x2+y2+z2 -xy-yz-zx-x3 -y3 -z3

= x2+y2+z2-x3-y3-z3 =x(x-x2)+y(y-z2)+z(z-z2)

Εξάλλου χ2 - χ + _!_ � Ο => χ - χ2 � _!_ . 4 4 ο ' 2 1 2 1 ' μοιως y - y � - και z - z � - οποτε 4 4

1 1 S � - (x + y + z ) = -4 4 Επομένως S � _!_ με την ισότητα να ισχύει αν 4

και μόνο αν οι δυο από τους αριθμούς χ, y είναι ίσοι με Υ2 και ο άλλος είναι ίσος με μηδέν, που σημαίνει ότι η μέγιστη τιμή του αθροίσματος είναι Υ4 και επιτυγχάνεται αν και μόνο αν (χ = y = ± και z = Ο) ή (χ = y = � και y = Ο) ή

( y = z = � και χ = Ο) . :c_�η μ:; ( ιιJση : Η στήλη αισθάνεται την ανάγκη να ζητήσει για μια ακόμη φορά συγγνώμη από τους φίλους που προσπάθησαν να λύσουν το παραπάνω θέμα με την εσφαλμένη αρχική διατύπωση .

1 1 0. Έστω δύο περιττοί πρώτοι ακέραιοι pι , Ρ2 και α, η ακέραιοι με α>1 και η>1 . Αν η εξίσωση ( Ρ2

2

- 1 )Ρ' + ( p2

2

+ 1 )Ρ' = απ δεν έχει ακέραιες

λύσεις στην περίπτωση pι=Ρ2 , τότε δεν έχει ακέραιες λύσεις και στην περίπτωση ΡιW2

(Επροτάθη από τον φοιτητή του Ε.Μ.Π. Μ ιχαήλ Θ. Ρασσιά)

ΛίJση (Από τον ίδιο) Από τα δεδομένα αποκλείονται οι περιπτώσεις

p1 = 2 και p2 = 2 , οπότε θεωρούμε p2 =2χ+1 ,χ εΝ. Υποθέτουμε τώρα ότι η εξίσωση δέχεται

τουλάχιστον μια ακέραια λύση .

Τότε, η εξίσωση (Ρ22- 1τ + ( Ρ2

2+ 1τ = aπ

λαμβάνει τη μορφή χΡ' + (χ + 1)Ρ' = a" (1) Επειδή ο αριθμός p1 είναι περιττός, προκύπτει ότι:

aπ = χΡι + (χ + 1)Ρι = (χ + χ + 1)Α = {2χ + 1)Α, Α ε Ν .

Άρα 2 χ + 1 Ιa π • Επειδή ο 2 χ + 1 είναι πρώτος

αριθμός προκύπτει ότι 2χ + 1 ia . Άρα

(2χ + 1)2 1a2 . Όμως η > 1 , οπότε (2χ + 1)2 1a"

ή (2χ + 1)2 ΙχΡ' + (χ + 1)Ρ' (2)

Επομένως:

ΧΡι + (Χ + 1)Ρ' = ΧΡι + [(2χ + 1) _ χ]Ρι

= χΡ' + (2χ + 1)Ρ' + (�1}2x + 1)p' -1 (-x) + . . .

+ ( ΡΙ ) (2χ + 1)(-χ)p, -Ι + (-χ)Ρ' Ρ Ι - 1

= χΡ' + (2χ + 1)2 Β + ( ΡΙ ) (2χ + 1)χp, -Ι _ χΡ' Ρ Ι - 1

ΕΥΚΛΕΙΔΗΣ Β '65 τ.l/75

Page 78: Ευκλειδης Β 65

Ο Ευκλείδης προτείνει .. .

= ( p1 ) (2χ + 1)χΡι - 1 + (2χ + 1)2 Β , Β ε Ζ . Ρ1 - 1

Οπότε :

χΡι + (χ + 1ΥΙ = ( p1 ) c2x + 1)χΡι -1 + (2χ + 1)2 Β . Ρ1 - 1

Από τη σχέση (2) έχουμε ότι

(2χ + 1)2 ( Ρ1 ) (2χ + 1)χΡι -1 Ρ1 - 1

ή (2χ + 1) ( Ρ1 ) xp1 - 1 , ή (2χ + l) lp1 . χ Ρι -1 . Ρ 1 - 1

Επειδή ο αριθμός 2χ + 1 είναι πρώτος, ισχύει ότι (2χ + 1) lp1 ή (2χ + l) lx . Η δεύτερη περίπτωση αποκλείεται αφού 2χ + 1 > χ .

Τελικά (2χ + l) lp1 , δηλ. p1 = 2χ + 1 αφού ο αριθμός p1 είναι πρώτος. Άρα p1 = p2 . Όμως, σύμφωνα με την υπόθεση , η εξίσωση δεν επιδέχεται λύση για p1 = p2 , οπότε, η εξίσωση ( 1 ) (p2 -1)1'1 (p2 + 1)Ρι _ η και επομένως η εξίσωση -- + -- - a

2 2 δεν έχει ακέραιες λύσεις.

Λύση 2η (Από τον συνάδελφο Ροδόλφο Μπόρρη , χωρίς την προϋπόθεση ότι η εξίσωση είναι αδύνατη για Ρ 1 = Ρ2 )

Έστω p 1=2μ+ 1 και p2=2κ+ 1 ακέραιες λύσεις της εξίσωσης. Τότε έχουμε κ2μ+ 1+(κ+ 1 )2μ+ 1=α"=>(2κ+ 1 )Q(κ)=α", ( 1 ) όπου Q(κ) ιiκέραιος (Αριθμητική Τιμή πολυωνύμου φ(χ) με ακέραιους συντελεστές) . Από την τελευταία

Στο τεύχος 64 δημοσιεύθηκε λύση της άσκησης 1 04 από τον συνάδελφο Χρυσόστομο Κούρτη, που προτάθηκε από τον συνάδελφο Θανάση Κυριακόπουλο. Ενδιαφέρον παρουσιάζει η καθαρά γεωμετρική προσέγγιση της άσκησης αυτής από τον συνάδελφο Γιώργο Τασσόπουλο την οποία δίνουμε στη συνέχεια.

104. Σε τρίγωνο ΑΒΓ η γωνία Β είναι διπλάσια από τη γωνία Γ. Να βρείτε τη σχέση που συνδέει τις πλευρές του τριγώνου.

Λύση : �----------------------.

' /

' .... .... _ _ , , �

ι I

Λ Λ Λ Λ

Είναι Β = 2 Γ . Άν Γ = ω , τότε Β = 2ω .

ισότητα προκύπτει ότι 2κ+ 1/α" . Όμως 2κ+ 1 Λ Λ Λ Λ

αποτέλεσμα ΒΖΓ = ΒΓΖ = ω , δηλ. ΒΓΑ=ΒΖΓ=ω ,

Προεκτείνουμε την ΑΒ κατά τμήμα �Η"' =α. Τότε το τρίγωνο ΒΖΓ είναι ισοσκελές με

πρώτος, οπότε 2κ+ 1 /α =>α"=(2κ+ 1 )" ·λ, λ εΖ. που σημαίνει ότι η πλευρά ΑΓ είναι εφαπτομένη Άρα: ( 1 )=>(2κ+ l )Q(κ)=(2κ+ 1 )"·λ απ' όπου του περιγεγραμμένου κύκλου του τριγώνου ΒΖΓ

προκύπτει ότι το πολυώνυμο Q(x) έχει παράγοντα στο Γ. Από το θεώρημα της εφαπτομένης το (2χ+ 1 ) . Επομένως ο αριθμός παίρνουμε : ΑΓ2 = ΑΒ · ΑΖ ή β2 = γ(γ + α) ή Ρ(κ)=κ2μ+ 1+(κ+ 1 )2μ+ Ι μπορεί να γραφεί και ως

2 β2 - γ2 = αγ , που αποτελεί και τη ζητούμενη Ρ( κ)=(2κ+ 1 ) R( κ), πράγμα που σημαίνει ότι το πολυώνυμο Ρ(χ) έχει παράγοντα το (2χ+ 1 ? . σχέση .

Επομένως θα έχουμε Ρ '(-�) = Ο , άτοπο,

αφού για το πολυώνυμο Ρ(χ) ισχύει Ρ '(χ)=(2μ+ 1 ) [χ2μ+(χ+ 1 )2μ] και

p ·( -Ξ) � (2μ + Ι ) [( �τ + (Η} ο

Τελικά η εξίσωση δεν έχει ακέραιες λύσεις

Λύση έστειλε επίσης ο Πολιτικός Μηχανικός Μηλιάκος Γεώργιος - Σπάρτη

'

Προτεινόμενες Ασκήσεις

127. Αν α,β είναι θετικοί πραγματικοί αριθμοί να αποδείξετε ότι (8α2 + β2 )2 > 1 6α3β

(Προτείνεται από τον Θανάση Κυριακόπουλο) 128. Δίνεται η εξίσωση αχ2 + βχ + γ = Ο (1) ,

όπου α,β,γ ε R με lαyi > O (2) και 1�-1� +1� <0 (3). Να αποδείξετε ότι η εξίσωση (1)έχει το πολύ μία ακέραιη ρίζα.

(Προτείνεται από 'fΟν Θανάση Κυριακόπουλο)

ΕΥΚΛΕΙΔΗΣ Β '65 τ. l /76

Page 79: Ευκλειδης Β 65

Τα Μαθηματικά μας διασκεδάζουν Τα μαθηματικά αν και είναι επιστήμη που απαιτεί αυστηρή διατύπωση, έχουν τη μαγεία

να αποσπούν το ενδιαφέρον όλων των ανθρώπων. Επινοήσεις σε προβλήματα ή ασκήσεις με κατάλληλο τρόπο διατυπωμένα εξάπτουν το πνεύμα, διεγείρουν τη φαντασία και κεντρίζουν την περιέργεια. Πρώτοι οι Αρχαίοι Έλληνες όπως ο Δ ιόφαντος, ο Ζήνωνας κ. ά. μας δίδαξαν αυτά τα μαθηματικά.

Στη στήλη αυτή θα παρουσιάζουμε θέματα τα οποία δεν απαιτούν ιδιαίτερες μαθηματικές

γνώσεις αλλά μας διασκεδάζουν με τη ν εκφώνησή τους ή τη λύση τους και είναι μια ευχάρι­στη και συναρπαστική ασχολία .

Επιμέλεια : Παναγιώτης Χριστόπουλος

Σ η μ ε ί ωση : .-:1. πό το τε�;ο; αυτό θα δημοσιεύουμε στο ίδιο τεύχος σε άλλη σελίδα τις απαντήσεις τω , · ;φοβί.ηιιcί. π·j \ ' ιj rc•J \ ' ; ·ρ ioc•j \ ' και όχι στο επόμενο τεύχος όπως γινόταν μέχρι σήμερα.

' Ι αντέψτε το τηίi:φω\'Ο του φίλου σας Πείτε σ:ο l'ίi.o σα� : " Γ ι=-άvε τα τρία πρώτα ψηφία του τη ί.ει..:>e:)\Όυ σου :τ .;ι: . στο 2 1 03 6 1 6532 γράφου­με .3 6 1 ό;ι:ι :ο .:::: I ΓΙ τi στο 2 76 1 06 1 299 γράφουμε 1 06 ό;ι:ι :ο .:::: - 6 . Π οί.ί.α:rί.ασίασε τον 3ψήφιο αυτό αριθ uό μc: :ο 5 (Ί χαι πρόσθεσε τη μονάδα. Το απο­τέλεσμα ;:οί.λα:rί.ασίασέ το με 250 . Στη συνέχεια πρόσΗc:σε οι�ο οορέ� τα 4 τελευταία ψηφία του τη­ί.εοC:)\Όυ σου. Αοαίρεσε τώρα το 250 και πές μου το\ ' α �'!Ι=ι u ό :του ;φοκύπτει» . Ποιό είναι το τηλεφω­\'ό του : ι δε� τι� απαντήσεις)

Π α ιπίδι με τι.: ευθείες Σ;ι:εδιάστε ένα μεγάλο κύκλο και με ένα μοιρο­γ\'C·ψόηο σημειώστε πάνω του ανά Ι Ο μοίρες τους αριθμού.; I ,2,3 ,4, . . . , 36 . α ) Ε\'6)στε τώρα το σημείο με αριθμό ν με το ση­μείο ν+ ι Ο. Όταν το ν+ ι Ο είναι μεγαλύτερο από 36 θ α αφαιρέσετε το 36 για να βρείτε το σωστό ση­μείο . Για παράδειγμα 27 και 27+ Ι Ο θα γίνει 27 και 27+ 1 0-36 . Αν αυτό γίνει για όλα τα σημεία τι σχή­μα προκύπτει από τα ευθ . τμήματα; β) Σε έναν άλλο κύκλο ενώστε τώρα το σημείο με αριθμό ν με το σημείο ν+5 . γ) Το ν με το ν+ Ι 5 δ) Δοκιμάστε και άλλες σχέσεις ε) Δοκιμάστε και το ν με το 2ν. στ) Συνέχεια το ν με το 3ν κλπ. Τι σχήματα προέκυψαν;

Πρό)τοι αριθμοί Γ ράψτε αριθμούς που να έχουν από μια φορά όλα τα ψηφία. Ποιοι από αυτούς είναι πρώτοι; (δηλαδή διαιρούνται μόνο με τον εαυτό τους και τη μονά­δω

Μονοκοντυλιά Με μια τεθλασμένη (πολυγωνική) γραμμή που έχει 6 ευθύγραμμα τμήματα ενώστε 4 τετράδες σημεί­ων που είναι σε σχήμα τετραγώνου μπορείτε;

• • • •

• • • •

• • • •

• • • •

Το β ιβλίο περιδιαβαίνει το χώρο των

Μαθηματ ικών Κατεύθυνσης της Γ' Λυ κείου και

φιλοδοξεί να αποτελέσει ουσιαστ ικό βοήθημα για

τους μαθητές-υποψήφιους και τους καθηγητές.

Παράλληλα, επιδιώκει να συστήσει τη βάση

πάνω στην οποία θα αναπτυχθεί ένας γόνιμος

προβληματισμός γύρω από τα θέματα και το

πνεύ μ α των Πανελλαδικών Εξετάσεων.

Ε ΚΔΟΣΕΙΣ ΜΑΥΡΙΔΗ Τηλ. : 231 0 228009 • Fax: 231 0 287097

ΕΥΚΛΕΙΔΗΣ Β' 65 τ. Ι /77

Page 80: Ευκλειδης Β 65

------------- Τα Μαθηματικά μας Διασκεδάζουν -------------

ΊΊ:τραγω\' ική ρίl:ο

Η ρίζες των αριθμών 2χ3χ4χ5+ 1 , 3χ4χ5χ6+ 1 , 4χ5χ6χ7+ 1 , 5χ6χ7χ8+ 1 είναι ακέραιοι αριθμοί για­τί;

λλλο.: τρόπος πολλοπλοσ ι ασμοίJ

Έχω να πολλαπλασιάσω 3 7χ29 και κάνω τα εξής βήματα: Διαιρώ συνεχώς το 37 με το 2 και γράφω κάθε φορά μόνο το πηλίκο . Ταυτόχρονα πολλα­πλασιάζω τον δεύτερο αριθμό 29 επί 2 συνεχώς τόσες φορές όσες συνεχίζεται η διαίρεση . Το απο­τέλεσμα είναι το άθροισμα των γινομένων που α­ντιστοιχούν στα πηλίκα που είναι μονοί αριθμοί. 37χ29

37 29

1 8 58

9 1 1 6

4 232

2 464

1 928

Αποτέλεσμα 29+ 1 1 6+928 = 1 073

Ο κυρ Φά\·ης και το κρασί

Φημιζόταν για το καλό κρασί που είχε ο κυρ Φά­νης και όλοι έτρεχαν στην ταβέρνα του να γεμί­σουν το κανάτι τους από το βαρέλι του . Για να δί­νει τις ποσότητες που του ζητούσαν είχε δύο δο­χεία το ένα χωρούσε 7 λίτρα και το άλλο 4 λίτρα που τα χρησιμοποιούσε με μεγάλη επιδεξιότητα. Μπορούσε να υπολογίσει οποιαδήποτε ποσότητα λίτρων χωρίς να χαθεί σταγόνα κρασί ενώ αν πε­ρίσσευε κάποια ποσότητα την έριχνε πίσω στο βα­ρέλι. . Θα μπορούσατε εσείς να εξυπηρετήσετε έ­ναν πελάτη που ζήτησε 2 λίτρα;

ΊΌ χαλιtσμ έ\'Ο κομπιουτεριΊ κ ι

Ένας μαθητής θέλει να πολλαπλασιάσει δυο αριθ­μούς αλλά τα πλήκτρα που λειτουργούν στο κο-

μπιουτεράκι του είναι το @ το � και το � Μπορείτε να τον βοηθήστε να κάνει τον πολλα­

πλασιασμό;

λλλο.: τρόπος :-\φιι ίρι:ση.:

Θέλουμε να αφαιρέσουμε 357 από τον 439 . Κά­νουμε αφαίρεση του 357 από το 999 και ύστερα στη διαφορά τους προσθέτουμε τον 439. Από τον αριθμό που προκύπτει αγνοούμε το 1 από τη θέση των χιλιάδων και προσθέτουμε 1 στις μονάδες και έχουμε το αποτέλεσμα. Είναι σωστό; γιατί;

Η συ,·ιιυλίιt

Ο Πέτρος και ο Μιχάλης θέλουν να παρακολου­θήσουν συναυλία σε γειτονική πόλη που απέχει 2 1 km. Το μόνο μέσο που διαθέτουν είναι το ποδήλα­το του Πέτρου. Με το ποδήλατο ο Πέτρος τρέχει 1 6 km/h ενώ με αυτό το ποδήλατο ο Μιχάλης τρέ­χει 1 2 km/h. Επίσης ο Πέτρος περπατά 6 km/h ενώ ο Μιχάλης 8 km/h. Σε 2ώρες και 1 6 λεπτά αρχίζει η συναυλία. Πώς πρέπει να κινηθούν για να φτά­σουν μαζί στη συναυλία και χωρίς καθυστέρηση ;

Π ρό βλημιt του Ν είJτ(Ι)\'ΙΙ 75 βόδια κατανάλωσαν σε 1 2 μέρες το χορτάρι που υπήρχε σε ένα λιβάδι 60 στρεμμάτων, καθώς και το χορτάρι που φύτρωσε κατά τη διάρκεια αυ­τών των 1 2 ημερών. Πόσα βόδια θα κατανάλωναν το αντίστοιχο χορτάρι 96 στρεμμάτων σε 1 8 μέ­ρες; Το χορτάρι φυτρώνει ομοιόμορφα και τα βό­δια τρώνε εξ ίσου και κάθε μέρα ίδια ποσότητα.

Οι οχη\> υπουργοί

Οχτώ υπουργοί συμφώνησαν να συνεδριάζουν για το ασφαλιστικό κάθε μέρα γύρω από ένα στρογγυ­λό τραπέζι, αλλά κάθε φορά θα κάθονται με δια­φορετική σειρά. Όταν δεν υπάρχει πλέον άλλος δι­αφορετικός τρόπος να καθίσουν τότε θα σταματή­σουν τις συνεδριάσεις και θα ψηφιστεί ο νόμος στον οποίο θα έχουν καταλήξει. Πότε θα ψηφιστεί ο νόμος;

Οι απαντήσεις είναι στη σελίδα 81.

ΕΥΚΛΕΙΔΗΣ Β' 65 τ.l/78

Page 81: Ευκλειδης Β 65

/1 Νίκος Α νδρουλακάκης

Περιμένοντας ερωτήσεις και απορίες σας γιά τη μουσική, θα σας ενημερώσουμε γιά τα βασικά. Συνηθέστεροι τύποι μουσικών αρχείων στους υπολογιστές είναι : 1 .cda (=CD Audio) π.χ. Track 1 5 .cda : συντομεύσεις ψηφιακών μουσικών αρχείων μεγέθους lΚΒ ,

που τα συναντάμε στον Windows Explorer, όταν βάλουμε ένα μουσικό CD στον υπολογιστή . 2 .wma (=Windows Media Audio) format, ψηφιακών μουσικών αρχείων της Microsoft. 3 .waν (=WAVeform=KYMATOMOPΦΉ) γενικό format, ψηφιακών μουσικών αρχείων. 4 .mp3 (=Moνing fictures Expert GroupJ) format συμπυκνωμένων ψηφιακών μουσικών αρχείων. 5 .mid (=Musical !nstrument Digital Interface=Moυσικoύ Οργάνου Ψηφιακός Διασυνδετής). Με διπλό κλικ επάνω τους , ανοίγει ο Media player και τα ακούμε. Όλα , περιέχουν πληροφορίες από την ψηφιακή ΗΧΟΓΡΑΦΗΣΗ που κάναμε,

εκτός του MIDI, που περιέχει πληροφορίες γιά τη μουσική που ΠΑΙΞΑΜΕ όταν το δημιουργούσαμε. f ] 'i · . :. 1 c , _, (; Ι

Ο πιανίστας, πατώντας τα πλήκτρα του παραδοσιακού πιάνου, αναγκάζει τα αντίστοιχα "σφυράκια" να χτυπήσουν τις αντίστοιχες χορδές. Αυτό που ακούει από τις παλλόμενες χορδές, εάν προηγουμένως έχει πατήσει RECORD στο πρόγραμμα ΗΧΟΓΡΑΦΗΣΕΩΣ, (εισαγόμενο από το μικρόφωνο), θα καταγραφεί και σ' ένα . wav αρχείο. Όμως, πατώντας τα πλήκτρα ενός ηλεκτρονικού MIDI μουσικού οργάνου, (συνδεδεμένου με τον υπολογιστή) , απλώς "ανοιγοκλείνει διακόπτες" που καθοδηγούν την κάρτα ήχου, να παράγει τη μουσική (και μάλιστα, εάν διαλέξει, και με διαφορετικό ήχο από του πιάνου). Πάλι το ίδιο θα ακούει, (τώρα όμως από την κάρτα ήχου) αλλά εάν προηγουμένως, επιπλέον έχει πατήσει RECORD σ 'ένα πρόγραμμα SEQLΈNSER (=σειριακός καταγραφέας), θα καταγραφεί και σ' ένα .mid αρχείο, όχι ήχος, αλλά με

ακριβή χρονική σειρά, το πότε, με πόση διάρκεια και ποιούς διακόπτες-πλήκτρα πάτησε. Ακόμα και το pedal, διακόπτης είναι ! Τώρα με PLAY, και τα δύο αρχεία θ ' αποδώσουν ΤΗΝ ΙΔΙΑ ΜΟΥΣΙΚΉ. Μόνο που το .wav θα παίξει την ηχογραφημένη μουσική , ενώ το .mid θα στέλνει ΤΩΡΑ στην κάρτα ήχου , τις ίδιες εντολές με αυτές που έστελνε ο πιανίστας πριν, παίζοντας ζωντανά. Διαφορές: Για ένα τρίλεπτο κομμάτι, το .wav θα έχει καταλάβει περίπου 30 MBytes μνήμης, ενώ το .mid μονάχα . . . . 30 ΚBytes ! Βέβαια , η ηχογράφηση, θα περιελάμβανε , εκτός απ' το θόρυβο της εξατμίσεως του διερχόμενου δίκυκλου (δηλ. χρειάζεται Studio) ΚΑΙ φωνή, εάν συγχρόνως τραγουδούσε ο πιανίστας, ο . . . Πάριος ή και μιά χορωδία , ενώ το .mid θα είναι μόνο ορχηστρικό (instrumental). Η . . . επανάσταση οφείλεται στο ότι, αφού ΔΕΝ πρόκειται για ηχογράφηση, αλλά γιά ταξινομημένη καταγραφή μουσικών πληροφοριών, μπορούμε να δούμε την παρτιτούρα ολόκληρης της ενορχηστρώσεως, ν' απομονώσουμε π.χ. το μέρος του μπασίστα, θαυμάζοντας το πώς συνοδεύει τους υπόλοιπους, ν' αναλύσουμε το ρυθμό και ν ' ανακαλύψουμε ένα ένα τα κρουστά που χρησιμοποιεί ο ντραμίστας, να μάθουμε ποιές πολύπλοκες συγχορδίες έχει διαλέξει ο κιθαρίστας και βάζοντας πιό αργά το τέμπο, να προσπαθήσουμε να μιμηθούμε το δεξιοτεχνικό σόλο του σαξοφωνίστα ! Εδώ μπορούμε να κάνουμε και μιά πονηριά. Να παίξουμε σε αργό τέμπο, ένα περίπλοκο σόλο και μετά να το παρουσιάσουμε στους . . . . θαυμαστές μας, στο κανονικό, το γρήγορο ! Φαντάζεστε πόσο εύκολα και γρήγορα μπορείτε να ενορχηστρώσετε ένα γνωστό σας κομμάτι ; Παίζετε το σόλο. Ακούγοντάς το, προσθέτετε το μπάσο. Ακούγοντας σόλο+μπάσο προσθέτετε ντράμς. Κατόπιν μιά χορωδία, μιά κιθάρα, ένα πνευστό κλπ. Ότι δεν σας αρέσει, το διαγράφετε. Και πάλι . . . . Εκτός λοιπόν από τ ο ότι μπορούμε ν ' ανακαλύψουμε τους κρυμμένους θησαυρούς στις ενορχηστρώσεις

ΕΥΚΛΕΙΔΗΣ Β ' 65 τ. l /79

Page 82: Ευκλειδης Β 65

Η επανάσταση του MIDI

των αγαπημένων μας κομματιών, κατεβάζοντάς τα απ' το intemet (υπάρχουν όλα τα είδη μουσικής,

από κλασικά και τζαζ, μέχρι έθνικ, χορευτικά, έντεχνα, λαϊκά, ποπ, ροκ, κλπ), ακόμα και εάν ΔΕΝ είμαστε μουσικοί, αξίζει να "παίξουμε" με τη μουσική . Να μεταφέρουμε π.χ. τη drums από ένα

τσιφτετέλι σ' ένα κλασικό κομμάτι του Μπετόβεν και . . . να το διασκευάσουμε. Αυτές είναι μερικές από τις δυνατότητες που προσφέρει η μουσική τεχνολογία , σήμερα .

J.. s i n x = ? 1 1 •

sίχ = 6

Ας παρατηρήσουμε από μία οθόνη του προγράμματος που μοιάζει με λατέρνα (ονομάζεται ΚΕΥ ΕDΠ), χωρίς να χρειάζεται να ξέρουμε νότες , τρία κομμάτια πασίγνωστα και ας παρακολουθήσουμε πώς ανεβοκατεβαίνουν οι νότες τους . Το Ν ο 7 είναι τα Χριστουγεννιάτικα κάλαντα (Καλήν εσπέραν άρχοντες) . Το Νο 8 είναι το Κερκυραϊκό (Μεσ' αυτή τη γειτονιά στην πάρα πάνω ρούγα). Το Ν ο 9 είναι ένα retro της παλιάς Αθήνας (Στο Ζάππειο μια μέρα, περιπατούσα) . Για τους τελείως άσχετους : Τα πλήκτρα , όσο πηγαίνουμε προς τ ' αριστερά , αντιστοιχούν σε πιό μπάσα ! Προς τα δεξιά , οι νότες «ανεβαίνουν» στα πρίμα. Αν σας βρίσκεται κάποιο πληκτροφόρο όργανο , δοκιμάστε με το ένα δάχτυλο να παίξετε αυτά τα εύκολα και γνωστά κομμάτια . Όπως λέει η πείρα μου, μετά το τρίτο κομμάτι , το χέρι σας θ ' αρχίσει πηγαίνει μόνο του όλο και πιο εύκολα . Θα παίζετε «με τ ' αυτί», όπως και πάρα πολλοί άλλοι . Πολύ θα βοηθηθείτε στην αρχή , αν παίζετε αργά και τραγουδάτε συγχρόνως. Όσο για το αριστερό χέρι , που είναι υπεύθυνο για τις συγχορδίες και τους ρυθμούς . . . . έχουμε καιρό .

Άντε και καλή επιτυχία.

Ευπρόσδεκτα σχόλια και αιτήματα στην ηλεκτρονική διεύθυνση into@hιηs .gr ή

[email protected]

Και γιά να μην νομίσετε ότι είμαστε μονομανείς

μουσικομαθηματικοί, κλείνουμε το άρθρο με δύο

μαθηματικές γελοιογραφίες.

ΕΥΚΛΕΙΔΗΣ Β' 65 τ. l/80

Page 83: Ευκλειδης Β 65

Απαντήσεις στα Μαθηματικά μας δισκεδάζουν Μαντέψτε το τηλέφωνο του φίλου σας

Διαιρέστε τον αριθμό που σας είπε ο φίλος σας ότι προέκυψε, με το 2 και θα έχετε τον αριθμό του τηλεφώνου του . Π α ιπίδι με τι.; ευθείες

Στην πρώτη περίπτωση θα πάρετε εσωτερικά έναν κύκλο. Στην ε) σχή­μα σαν καρδιά στην στ) σχήμα νε­φρού κλπ. Μονοκοντυλιά Πρώτοι αριθμοί

Κανένας. Γιατί το άθροισμα των

ψηφίων Ο+ I + 2+ 3+ . . . +9=45 που δι­αιρείται με το 3 και το 5 άρα κανέ­νας δεν είναι πρώτος.

Τετραγωνική ρίζα Διότι ν( ν+ Ι )(ν+2)(ν+3 }+ Ι= (ν � -3ν- Ι ) 2 . .\ί.λος τρόπος πολλαπλασιασμού Γίνεται διότι μετατρέπεται ο ;τρώτο; αριθμό; σε δυαδική μορσή και έzουμε 37 = � -. :--Οz� Ό-Ι-Οz� - 3- Ι z� · �--οz� Ί - Ι = 1 00 1 0 1 λρα 3 7χ29 = I � . 5 /� 9 ι-ι � . �z�9 Ι-Ι ! z)9 I Ο ι.:ι ρ Φά\·η.: ι.:αι το ι.:ρα σ ί Γεμίστε ϊ Ο -ί.ιτρο και ύστερα με α·,,: ο · . � ,...::-:c: :ο 4ί.c:fΧ'.λL::.ειάv.ε στο βαρέλι το 4λιτρο και ι:-:�: Ξ :-Ξ ::·_ : :· :ο κ:αΎ. ι 3 ί.ί:ι:-α 1 ;του έzει το 7λιτρο. Ξαναγε­,.:::-:.Ξ :: - �·. : ::c ::.::ό :ί' 3αι:-έi.ι και από αυτό aπογεμίστε το ..:.··.: :.-.: .:. :: :: ::.' .:: ·. -::-.ω ,.ι: 1 ί.ίφο ι . λδειάστε πάλι το 4λιτρο στο Ξ-::: :.··.-. •: :: . ::�:c: α::ό -ί.ιτρο να γεμίσει και πάλι; έτσι στο - -._ : : :·: -:-· :: _.:: :-. -ο'Λ :-ιό\'ο � ί.ίτρα για τον πελάτη . Τ ο ιa�.α σuηο ιωμπιουτερά κι

' , 2 Έ Ί_-: -. _.: :-:> = ι z-υ ι - - z - -ψ Το μισό είναι το χψ.

ΜΑΘΗΜΑΤΙΚΕΣ ΠΡΟΚΛΗΣΕΙΣ

ΠροJλήματα για την ανάnτu�η της μαβηματικής crκίψης ΚΥΚΛΟtΟΡΕΙ ΣΤΑ ΑΓΓΛΙΚΑ ΚΑΙ ΣΥΑ ΓΑΜΙΚΑ

Άλλο�: τρόπος Αφαίρεσης Η μέθοδος είναι ανάλογη αυτής που χρησιμοποιούν οι υπολογιστές αλλά εκεί οι αριθμοί είναι σε δυαδική μορφή

439-357=43 9+(999+ 1 - 1 000)-3 5 7= 439+(999-35 7)+ 1 - 1 000 Η συναυλία Ο Πέτρος πρέπει να διανύσει 12 km με το πο­δήλατο, να το αφήσει και να συνεχίσει με τα πόδια. Χρόνος 2 ώρες και 15 λεπτά. Ο Μιχάλης ξεκινάει με τα πόδια και στα 1 2 km βρίσκει το ποδήλατο το παίρνει και διανύει τα υπόλοι­πα 9. Χρόνος 2 ώρες και 15 λεπτά. Π ρόβλημα του Ν εύτωνα Έστω χ η ποσότητα του χορταριού που φυτρώνει κάθε μέρα σε κάθε στρέμμα. Δηλαδή 75 βόδια τρώνε σε 1 2 ημέρες χορτάρι 60 στρεμμάτων + 720χ (1) Τα 81 βόδια τρώνε σε 15 ημέρες χορτάρι 72 στρεμμάτων + I 080χ (2) Πρέπει να βρούμε πόσα βόδια τρώνε χορτάρι 96στρεμμάτων + 1 728χ σε 1 8 μέρες. Από (1) 75 β σε 1 5 μ τρώνε 75 στ + 900χ Άρα I β σε 1 5 μ τρώει I στ + 1 2χ (3)

8 1 β σε 1 5 μ τρώνε 8 1 στ + 972χ (4) Από (2) και (4) προκύπτει ότι χορτάρι 8 1 στ + 972χ είναι ίσο με χορτάρι 72στ + 1 080χ και χορτάρι 1 στ = με 12χ συνεπώς

χορτάρι 9 στ = με I 08χ χορτάρι 60 στ = με 720χ χορτάρι 1 44 στ = με 1 728χ Από ( I ) 75 β σε 12 μ τρώνε 60 στ+60 στ Δηλαδή 75 β σε 1 8 μ τρώνε ( 1 20. 1 8) : 1 2 = 1 80 στ Άρα σε 1 8 μέρες χορτάρι 96 στ + 1 72 8χ = 16 στ + 1 44 στ θα το φάνε (75 . 240) : 1 80 = 100 βόδια. Οι οχτώ υπουργοί Ποτέ ! Οι δυνατές διαφορετικές θέσεις εί­ναι Ι χ2χ3χ4χ5χ6χ7χ8 δηλαδή 40320 ημέρες. Είναι I 1 0 χρόνια και 1 42 ημέρες.

ΚmWιιΛο νιο uoBnιtς που ouuu&ιtxouv σιοuς διανωvισuούς mς ΕΜnvικ�ς ΜαΒnuαιικQς

το εf&/Ιιο nΕΡιεχει :

MEPOl 1, Α. ΠΕΡt APtθMDN �� TQN MVεrHPtDN ΤΜ ΜΕΡΟΕ 1• Β. tεrOPttΆ εrotXEtA ΑΠΟ ΤΑ εΛΠΗΝf� θΕΗεΛtΑ

TQN kAθHMA1tΚDN

ΜΕΡΟΕ 2• Α. ΠΡ�ΗΜΑ1Α ΜΕ lXHI1A1A, NOktEHA1A, Ent/1/A � ΜΕΡΟΞ 2- Β. r�MErPtllOt �tΦOf

ΜΕΡΟΕ 2- r. �tΦOt nonι<HE

kEPOE 2- Δ. APfθMOθ�Pf.Πtt<Ot �tΦOt

ΜΕΡΟΕ 2- Ε. �tΦOt εYNΔVAErff(HΞ

kEPOE 2- εr. �tΦOt ΕΙΡΑ'1ΗΠΙ(ΗΕ

kΕΡΟΞ ;.. Α. ΑΝθΟΠΟΠΟ θΕΗΑ1DΝ kAθHkA1tΚDN ΔtAr�NtEkDN

ΤΗΕ εΛΠΗΝtf(ΗΞ MAθHkA1tf(Hε ErAtPEJA!

ΜΕΡΟΕ ;.. Β. APXEtO θEkA1DN MAθHkA1tΚDN

ΔtAr'QNtEHQN AMQN �QN

ΗΕΡΟΕ tι. AnANrHWE

Α. rvMNAEtO

B. nwo

ΕΥΚΛΕΙΔΗΣ Β ' 65 τ.l/81

Page 84: Ευκλειδης Β 65

...J

ιι::ι:: ιι::ι:: z u ιι::ι:: ­w ι­

c ιι::ι:: :Ξ

w w :Ξ ι­w ιι::ι:: ...J :Ξ cιι::::ι σ ι::ι::: Ω..

I

από r ιs εκδόσε ιs <<Δ Ι Ο ΦΑΝΤΟΣ>>

Γιώργος Μ. Μιχαηλfδης

ΜΑθΗΜΑΤΙΚΑ r ΛΥΚΕΙΟΥ

ΤΟΜΟΣ Β'

Θετική - Τεχνολογική Κατεύθυνση

ΟΛΟΚΛΗΡΩΜΑΤΑ

8 θεuιιΙ(α • MεOoboi.oy(�

• ΙΙαQfΙ 'rηQήιιη;: - Σχιiλια 8 Λυμtνι.ι ΙΙσ.�αδιίyμιιτα

8 Ε:�ωηjuεις: Κστανόησηi • Αιrχήσε•• - Ηέμutιι

Γιώργος Μ. Μιχαηλίδης

ΜΑΘΗΜΑΤΙΚΑ ΓΛΥΚΕΙΟΥ

θετική - Τεχνολογική Κατεύθυνση

eξε;-ι;�σe ι ς 2008 - 2009

Πιθανά Θeμa,;a

Γιώργος Μ. Μιχαηλίδης Octavian Ν. Stanasila

(Prot. Univ. Polltehnlca8uchaι'es�

Γιωργος Μ Μιχαηλίδης

ΔΙΟΦΑΝΤΟΣ ΕΚΔΟΣΕΙΣ